סיקור מקיף

התגלה החור השחור הסופר-מאסיבי הרחוק ביותר עד כה

תגלית מרתקת מהיקום הצעיר: חוקרים גילו את החור השחור הסופר-מאסיבי המרוחק ביותר שנצפה עד כה, בתקופה של 690 מיליון שנה בלבד לאחר המפץ הגדול. החוקרים הופתעו מהמסה העצומה שלו, שעומדת על פי 800 מיליון ממסת השמש, ועדיין לא מבינים כיצד יכל לאסוף מסה רבה כל כך בתקופה כה קדומה.

הדמיה של הקוואזר J1342+0928, הגלין הגלקטי הפעיל שמכיל במרכזו חור שחור סופר-מסיבי, עם מסה של פי 800 מיליון זו של השמש שלנו, הזולל חומר רב מדיסקה לוהטת הסובבת אותו. מקור: Robin Dienel/Carnegie Institution for Science.
הדמיה של הקוואזר J1342+0928, הגלעין הגלקטי הפעיל שמכיל במרכזו חור שחור סופר-מסיבי, עם מסה של פי 800 מיליון זו של השמש שלנו, הזולל חומר רב מדיסקה לוהטת הסובבת אותו. מקור: Robin Dienel/Carnegie Institution for Science.

צוות חוקרים בינלאומי הודיע אתמול על גילוי החור השחור הסופר-מאסיבי המרוחק ביותר המוכר עד כה, שלאורו נדרשו יותר מ-13 מיליארד שנות אור להגיע אלינו, מתקופה בה היקום היה בן 690 מיליון שנה בלבד, או 5% מגילו הנוכחי (13.8 מיליארד שנה). ממצאי המחקר פורסמו אתמול בכתב העת Nature.

לחור השחור, שכינויו J1342+0928 (שם קוד המעיד על מיקומו בשמיים), מסה עצומה של פי 800 מיליון מהמסה של השמש שלנו. החור השחור הסופר-מאסיבי סופח חומר רב מדיסקה לוהטת המסתחררת סביבו במרכזה של גלקסיה – תופעה המכונה קוואזר.

“קוואזרים הם מבין גרמי השמיים הבהירים והמרוחקים ביותר, וקריטיים עבור הבנה של היקום הצעיר”, אמר החוקר בראם ונמאנס, ממרכז מקס פלאנק לאסטרונומיה בגרמניה. לקוואזר הספציפי שבו שוכן החור השחור הסופר-מאסיבי, בהירות המשתווה לפי 40 טריליון מבהירות השמש שלנו.

מרחקו העצום של הקוואזר והחור השחור במרכזו נקבע באמצעות מדידת ההסחה לאדום של אורו. בתופעה המכונה אפקט דופלר, אורך גל של אור ממקור המתרחק מהצופה, “נמתך” ומשתנה לאורך גל ארוך יותר. בעקבות ההאצה בהתפשטות היקום, ככל שהעצם מרוחק יותר מאיתנו, כך הוא גם מתרחק יותר מאיתנו, ואורו מוסח יותר לאדום. ההסחה לאדום של הקוואזר המרוחק עומדת על כ-7.5, כשמשמעות הדבר היא שאורך הגל שלו כפי שנצפה מכדור הארץ הוא פי 8.5 (7.5+1) ממה שהיה כשנפלט ממנו במקור לפני יותר מ-13 מיליארד שנים.

תעלומת המסה של החור השחור הקדום

המסה העצומה של החור השחור המרוחק והקדום הפתיעה את החוקרים. תיאוריות קיימות המסבירות כיצד נוצרים ומתפתחים חורים שחורים סופר-מאסיביים, שסופחים חומר מגז וכוכבים במרכזן של גלקסיות, לא מצליחים להסביר כיצד החור השחור גדל כה מהר, זמן קצר כל כך, במונחים קוסמולוגיים, אחרי המפץ הגדול.

“איסוף כל המסה הזאת בפחות מ-690 מיליון שנים הוא אתגר עצום עבור תיאוריות של גדילת חורים שחורים סופר-מאסיביים”, אמר אדוארדו בניידוס, ממרכז קרנגי בארה”ב, שהוביל את המחקר.

אילוסטרציה המנסה להמחיש את מרחקו העצום של החור השחור הסופר-מסיבי, שלאורו לקח יותר מ-13 מיליארד שנות אור להגיע לאור שני טלסקופי מגלן בצ'ילה, שאחד מהם מוצג במתחית האילוסטרציה. מקור: Robin Dienel, courtesy of the Carnegie Institution for Science.
אילוסטרציה הממחישה את מרחקו העצום של החור השחור הסופר-מאסיבי, שלאורו לקח יותר מ-13 מיליארד שנות אור להגיע לטלסקופי מגלן בצ’ילה, שאחד מהם מוצג בתחתית האיור. מקור: Robin Dienel, courtesy of the Carnegie Institution for Science.

“אם אתה מתחיל עם זרע כמו כוכב גדול, ונותן לו לגדול בקצב המקסימלי האפשרי, ומתחיל מרגע המפץ הגדול, אתה לא מסוגל ליצור משהו עם 800 מיליון מסות שמש – זה לא ריאלי”, הסביר רוברט סימקו, פרופסור לפיזיקה מ-MIT, שהשתתף במחקר. “לכן חייבת להיות דרך אחרת שבה הוא נוצר. וכיצד בדיוק זה קרה, אף אחד לא יודע”.

לפי החוקרים, כדי להסביר את אופן היווצרותו של חור שחור כה מאסיבי בתקופה קדומה כל כך של היקום, מודלים להיווצרות חורים שחורים סופר-מאסיביים יצטרכו להראות כי ביקום הצעיר היו יכולים להתקיים “זרעים” של חורים שחורים עם מסה של עד כ-10,000 מסות שמש, בתקופה של 65 מיליון שנה בלבד לאחר המפץ הגדול, או, לחלופין, להסביר כיצד החורים השחורים המוקדמים ביותר גדלו בקצב רב יותר מההערכה המקובלת כיום.

עדות מהיקום הצעיר

ייחודיותו של החור השחור נובעת גם מהתקופה שבה החוקרים משערים שבה התקיים. לאור מהקוואזר לקח יותר מ-13 מיליארד שנים להגיע לכדור הארץ, והוא נפלט ממנו כשגיל היקום היה רק בן 690 מיליון שנה, בתקופה המכונה “שלב הריוניזציה” (Reionization), או “היינון מחדש” בעברית.

עד כ-380,000 שנה לאחר המפץ הגדול, היקום היה חשוך לחלוטין, מפני שפוטונים היו כלואים בתוך הפלזמה הצפופה והלוהטת של חלקיקים ואלקטרונים. אולם גם לאחר שהיקום התקרר מספיק על מנת לאפשר הופעת אטומי מימן בעלי מטען חשמלי נייטרלי, והפך לשקוף לאור, הוא נותר חשוך עד אשר זרחו הכוכבים הראשונים. אז החלה התקופה המכונה “שלב הריוניזציה”, שהתקיים לפני בין 13.5 ל-12.5 מיליארד שנים, כאשר האנרגיה שפלטו הכוכבים והגלקסיות הראשונים גרמה ליינון רוב אטומי המימן ביקום הצעיר – לאבד את האלקטרון שלהם.

“שלב הריוניזציה היה השינוי המשמעותי האחרון ביקום, והוא נחשב לאחד מבין חזיתות המחקר האסטרופיזי”, אמר בניידוס.

באמצעות ניתוח ספקטרום האור שנפלט מהסביבה הסמוכה לקוואזר, החוקרים הצליחו לקבוע שבין 38 ל-77 אחוז מהמימן בסביבתו נמצא בצורה של אטומים נייטרליים, ולא של יונים. משמעות הדבר היא שסביבת הקוואזר עדיין לא הספיקה להפוך למיוננת ברובה, ושהוא התקיים בתקופת הריוניזציה – העדות היחידה שהתגלתה מתקופה קדומה זו של היקום.

בתרשים השוואה בין הקוואזרים המאסיביים ביותר והמרוחקים ביותר שידועים נכון להיום. בעוד J1342+0928 (הכוכב הצהוב) אינו הקוואזר המאסיבי ביותר, הוא הרחוק ביותר שהתגלה עד היום. מקור: Jinyi Yang/UA; Reidar Hahn/Fermilab; M. Newhouse/NOAO/AURA/NSF.
בתרשים השוואה בין החורים השחורים הסופר-מסיביים בעלי המסה והמרחק הגדולים ביותר, שמוכרים נכון להיום. בעוד J1342+0928 (הכוכב הצהוב) אינו המסיבי ביותר, הוא הרחוק ביותר שהתגלה עד היום. מקור: Jinyi Yang/UA; Reidar Hahn/Fermilab; M. Newhouse/NOAO/AURA/NSF.

יש כיום מספר תיאוריות מתחרות בנוגע למתי בדיוק התרחש שלב הריוניזציה של היקום. לפי החוקרים, העדות החדשה מצביעה על כך שהריוניזציה התרחשה מאוחר יחסית בכרונולוגיה הקוסמולוגית.

“זה מוסיף להבנתנו הכללית של היקום, מכיוון שזיהינו את הנקודה בזמן שבה היקום נמצא בחצי הדרך של השינוי המהיר מאד ממצב נייטרלי למצב מיונן”, אמר סימקו. “יש לנו כעת את המדידה המדויקת ביותר של התאריך בו הכוכבים הראשונים נדלקו”.

החיפוש אחר הקוואזר המרוחק ביותר

קוואזרים מרוחקים הינם קשים לזיהוי. בשנים האחרונות, במיוחד מאז 2010, אסטרונומים הצליחו להכפיל את מספר הקוואזרים הידועים שמקורם מתקופה של כ-850 עד מיליארד שנים לאחר המפץ הגדול.

כדי “לשבור את השיא” ולאתר קוואזר מרוחק מתקופה קדומה אף יותר, צוות החוקרים בהובלת אדוארדו בניידו השתמש בשיטה מורכבת ושיטתית למיפוי מיליוני מועמדים.

כדי לזהות מועמדים לקוואזרים, החוקרים השתמשו בנתונים מאורכבים של מספר סקרי שמיים נרחבים, במטרה לאתר עצמים שמצד אחד הינם בהירים בתחום התת-אדום, ומצד שני בלתי ניתנים לזיהוי בתחום האור הנראה – זאת מפני שבשל אפקט דופלר, ההסחה לאדום של אור הקוואזרים המרוחקים הופכת אותם לבהירים יותר בתת-אדום.

סקרי השמיים שבהם השתמשו החוקרים כללו את אלו שהפיקו טלסקופ החלל WISE, “הטלסקופ התת-אדום של הממלכה המאוחדת” (UKIRT) במאונה קאה בהוואי, ו”מצלמת האנרגיה האפלה” (DECam) בצ’ילה.

מתוך מאות מיליוני היעדים שנצפו בסקרי השמיים הללו, החוקרים הצליחו להגיע למספר מאות מועמדים לקוואזרים. מועמדים אלו זכו לתצפיות מעמיקות יותר באמצעות מספר טלסקופים, ובמיוחד שני טלסקופי מגלן בצ’ילה.

המאמץ הסיזיפי הזה הניב בסופו של דבר גילוי של קוואזר יחיד מתקופה קדומה זו של היקום, אולם, בהתחשב בכך שהחוקרים צופים שיש רק בין 20 ל-100 קוואזרים רחוקים ובהירים כמו J1342+0928 בכל רחבי השמיים, מדובר בתגלית משמעותית ביותר.

בשני הצילומים הקטנים במעלה התמונה מראים את ההבדל בבהירות הקוואזר באור נראה ובתת אדום. בעוד בתת-אדום (ימין) הקוואזר נראה בברור, באור נראה לא ניתן להבחין בו כלל, מכיוון שאפקט דופלר גרם להסחתם האור הנראה שלו לאורכי גל ארוכים יותר - תת-אדום. בתחתית התמונה מוצג הספקטרום של הקוואזר, כפי שנרשם על ידי ספקטרומטרים בשני טלסקופי מגלן בצקילה.באמצעות הספקטרום של העצם, החוקרים יכלו למדוד את המסה העצומה של החור השחור הסופר-מאסיבי ששוכן במרכז הקוואזר. מקור: Eduardo Bañados/Carnegie Observatories and Xiaohui Fan/UA.
שני הצילומים הקטנים במעלה התמונה מראים את ההבדל בבהירות הקוואזר באור נראה ובתת אדום. בעוד בצילום בתת-אדום של טלסקופ מגלן (ימין) הקוואזר נראה בברור, באור נראה לא ניתן להבחין בו כלל, מכיוון שאפקט דופלר גרם להסחת האור הנראה שלו לאורכי גל ארוכים יותר – כגון תת-אדום. בתחתית התמונה מוצג הספקטרום של הקוואזר, כפי שנרשם על ידי ספקטרומטרים בשני טלסקופי מגלן בצ’ילה. באמצעות ניתוח הספקטרום שלו, החוקרים יכלו למדוד את המסה העצומה של החור השחור הסופר-מאסיבי של הקוואזר. מקור: Eduardo Bañados/Carnegie Observatories and Xiaohui Fan/UA.

גם הגלקסיה בה שוכן החור השחור מפתיעה את החוקרים

באמצעות מערך טלסקופי הרדיו NOEMA בהרי האלפים הצרפתים והמערך הגדול מאד (VLT) בניו מקסיקו בארה”ב, צוות חוקרים גרמני ממכון מקס פלאנק לאסטרונומיה בחן את מאפיינה של הגלקסיה בה שוכן החור השחור הסופר-מאסיבי. הם פרסמו את ממצאיהם במאמר נלווה למאמר הראשי בכתב העת The Astrophysical Journal Letters.

החוקרים גילו להפתעתם שהגלקסיה, שגם היא כמובן מאותה תקופה קדומה של 690 מיליון שנה לאחר המפץ הגדול, מכילה כמויות גבוהות מאד של אבק ויסודות כבדים. עקב בהירותו העצומה של הקוואזר במרכז הגלקסיה, החוקרים לא מסוגלים לצפות באור הכוכבים שלה ישירות, לפחות לא בטלסקופים קיימים. למרות זאת, העובדה שהיא מכילה כמות גבוה יותר של יסודות כבדים, שנוצרים במרכזיהם של כוכבים, מעידה שהגלקסיה מכילה כנראה כמות גבוהה של כוכבים.

מסת כלל הכוכבים של הגלקסיה המרוחקת משתווה, כך לפי הערכה שהחוקרים מזהירים כי אינה מדויקת לגמרי, לכ-20 מיליארד פעמים המסה של השמש שלנו. אם משווים זאת לגלקסיית שביל החלב, שמסת כלל הכוכבים בה עומדת על בין 40 ל-60 מיליארד פעמים מסת השמש, מדובר בכמות כוכבים רבה מאד, בוודאי בתקופה כל כך צעירה של היקום.

תגלית זו בנוגע לגלקסיה המכילה את הקוואזר המרוחק תאפשר, לפי החוקרים, לחדד ולדייק מודלים קיימים בנוגע להיווצרות גלקסיות.

“מודלים של אבולוציית גלקסיות יצטרכו להסביר כיצד גלקסיה מסוגלת ליצור את הכוכבים הנדרשים כדי להפיק את הכמויות הנצפות של אבק ויסודות כימיים כבדים במשך זמן קצר כל כך יחסית”, אמר החוקר בראם ונמאנס ממכון מקס פלאנק לאסטרונומיה.

מה הלאה?

החור השחור הסופר-מאסיבי, כמו גם הגלקסיה בה הוא שוכן, יהפכו כעת ליעד לתצפיות המשך. כבר עתה אושרו מספר תצפיות, באמצעות טלסקופ החלל האבל שיצפה בקוואזר בתחום התת-אדום הקרוב, טלסקופ החלל שפיצר בתחום התת-אדום וטלסקופ החלל צ’נדרה שיצפה בו בקרני X. בנוסף, אושרו תצפיות במערך טלסקופי הרדיו ALMA במדבר אטקמה בצ’ילה, על מנת להמשיך ולחקור את הגלקסיה המארחת של הקוואזר.

מצפה החלל שנחשב ליורשו העתידי של טלסקופ החלל האבל, טלסקופ החלל ג’יימס ווב, שצפוי להיות משוגר בשנת 2019 לחלל, יסייע רבות במחקר העתידי הן של הקוואזר הספציפי הזה והן של היקום הצעיר. הטלסקופ יצפה בחלל בתחום התת-אדום במטרה לאפשר לו להסתכל רחוק ככל האפשר, ולחשוף עדויות מכוכבים וגלקסיות מתקופה קדומה אף יותר של היקום, של עד לפני כ-13.5 מיליארד שנים.

ראו עוד בנושא באתר הידען:

319 תגובות

  1. המידע שיש חור שחור שכזה זה ממש מפחיד לחשוב מה הגודל שלו עכשו כי זה קרה יחסית בתחילת היקום אז עכשו כשעברו מליארדי שנים לאחר מיכן הוא הספיק לאכול עוד כמה כוכבים (כשחור שחור אוכל כוכב אז המסה שלו גדלה) אז עכשו הוא חור שחור עם מסה עצומה בגודלה כך

  2. המידע שיש חור שחור שכזה זה ממש מפחיד לחשוב מה הגודל שלו עכשו כי זה קרה יחסית בתחילת היקום אז עכשו כשעברו מליארדי שנים לאחר מיכן הוא הספיק לאכול עוד כמה כוכבים (כשחור שחור אוכל כוכב אז המסה שלו גדלה) אז עכשו הוא חור שחור עם מסה עצומה בגודלה כך שכדור בארץ שלנו זה כלום בשבילו ?

  3. חזרנו מסנטה ברברה.

    סיכום דיון.

    בכתבה ״איום קוואנטי על תורת היחסות״ של סיינטיפיק אמריקן, נטען שאי לוקליות בשזירה קוונטית הכוללת השפעה מיידית מרחוק, סותרת את תורת היחסות הפרטית על פיה שום השפעה לא יכולה לעבור מהר מהאור.

    בכתבה נאמר שלמרות שבמשך שנים ארוכות חשבו הפיזיקאים והפילוסופים שהסתירה כביכול אינה קיימת, בשנים האחרונות נשמעים קולות מתרבים והולכים לכך שהסתירה אכן קיימת.

    ההסבר ללמה הוא לדעתי כתבה אחרת שבא מתואר ניסוי וויילר, על פיו ניתן להשפיע על העבר מהעתיד.

    על פי תורת היחסות הפרטית, מהירות על אורית פירושה מסע אל העבר. אמנם אין בכך סתירה ישירה לקוזיליות, אך עדיין מסע אל העבר הוא משהו שקשה לקבל מהבחינה הפיזיקלית. אילי פאת – מאת.

    אני העלתי את הרעיון שלא צריך להזקק לשזירה ואי לוקליות כדי לראות שקוונטים ויחסות לא מסתדרות. בשורש הניסוי המחשבתי שממנו התפתחה היחסות כפי שנכתב במאמר היחסות המקורי של איינשטיין, נמצא פוטון שלו תנע ומיקום מוגדרים ברגע מוגדר. למרות שרעיון זה הוא הגיוני לגמרי ב1905, הוא סותר את עיקרון אי הודאות, העיקרון היסודי של מכניקת הקוונטים.

    הבעיה מוצתה בפסקה הבאה:

    ״אתה יכול לפלוט פוטון מנקודה A וכעבור זמן מתאים לקלוט אותו ב-B.

    אבל אם הוא מרוח במרחב לפני כן, ועוד בהסתברות שווה, אז יש שתי אפשרוריות:

    1. מציאות הפוטון מעבר לB אינה משהו פיזי – זו רק פונקציית הגל או הפילוסופיה או הזברבירולוגיה, אך לא הפוטון עצמו.

    (קצת מוזר אם נחשוב על כך שריבוע פונקציית הגל היא ההסתברות למצוא את החלקיק בנקודה מסויימת).

    2. החלקיק עצמו נמצא גם מעבר לB.

    במקרה 1, אז אין כל ממשות לפוטון שנע מהר מc, איינשטיין צודק וקוונטים חרטה. אך להבנתי, לא זה המצב.

    במקרה 2, אם הפוטון עצמו נמצא מעבר לB בזמן פחות מct, אז אין מנוס מהמסקנה שהוא נע מהר מc ולמעשה בכל מהירות שהיא״.

    עדיין לא קיבלתי תשובה מאף אחד פה על השאלה המובעת בשורה האחרונה:

    ״במקרה 2, אם הפוטון עצמו נמצא מעבר לB בזמן פחות מct, אז אין מנוס מהמסקנה שהוא נע מהר מc ולמעשה בכל מהירות שהיא״.

    אז מישהו פה מתנדב להסביר?

    אריאל, כשעברתי על התגובות גיליתי את תגובתך שהומתנה בזמנו. אולי תנסה לענות ותיווכח במו עיניך שיש דברים שפשוט אין להם עדיין תשובה חד משמעית?

    נערך דיון בין ניסים לביני על מוזרותה של היחסות ובעיקר התארכות האורך, אך לא המשכנו לנקודות האחרות:

    אם מודל מקסוול שגוי ואין אתר – איך הוא הצליח למצוא באמצעותו את מהירות האור? הרי הוא איננו קיים!

    בפרדוקס התאומים המקורי, כל תאום יכול לטעון שהזמן שלו הוא הזמן האמיתי. אך אם נכונה תיאוריית המפץ, יש זמן מוחלט שמתבטא גם בטמפרטורת היקום על פי נוסחאת פרידמן. לכן כאשר נפגשים התאומים בחזרה, זמנו של התאום הנשאר הוא זמן היקום ואילו זמנו של המטייל הוא זמן מלאכותי.

    פיתוח של רעיון זה מביא לכך שאם נהפוך את היוצרות והתאום המטייל נייח ביחס לקרינה, אז זמנו של התאום הנשאר גבוה יותר מגיל הקרינה ולכן גם מגיל היקום.

    אז איך יכול זמנו להיות גבוה יותר מגיל היקום שהוא חלק ממנו?

    לסיום נקודה אישית שלשמחתי התבהרה בכתבה הזו.

    במשך שנים הואשמתי עיי אלבנצו בטענות שונות ומשונות שהראשית בהן היא שאני שקרן חסר בושה.

    ביקשתי ממנו להתנצל, אך הוא סירב וגם הביא בכתבה זו 3 דוגמאות לכך שאני שקרן:

    ״כן, אתה בהחלט שיקרת בדיונים הרבים שהיו לנו. מצחיק מצידך לצפות שאדע עכשיו לצטט לך בדיוק איפה, כמה למה ואיך בהתחשב בזה שמדובר בדיונים שחלקם היו לפני 3 או 4 שנים, אבל יש לי כמה דוגמאות בראש. אחת למשל היא שבמשך תקופה ארוכה היית חוזר וכותב שוב ושוב ציטוט שלי, אשר באופן פלאי תמיד היה נחתך לך בול באמצע המשפט. הסברתי לך פעמים רבות שאתה מביא חצי ציטוט ובכך משנה לגמרי את מה שאמרתי, אבל לך זה לא הפריע. חלקו הראשון של המשפט נגע לכך שיש אנשים שלא מבינים את מכניקת הקוונטים ולכן טוענים שהיא שגויה – ושלך יש אותה בעיה. החלק שהיית משמיט הבהיר במפורש שאני לא מאשים אותך בהכחשת מכניקת הקוונטים אלא באותו סוג של טעות – אתה לא מבין משהו ולכן מסיק שהוא כנראה שגוי. דוגמא נוספת היא שבמהלך דיון ארוך על פתרון חידת מטבעות כזו או אחרת בעזרת שזירה, טענת שאני אומר שיש לה פתרון קלאסי למרות שכתבתי במפורש פעמים רבות שאין לה פתרון קלאסי. רק ניסיתי להסביר לך שהפתרון הקוונטי לא כולל העברת מידע כי הוא דורש סנכרון מראש של מספר עצום של זוגות שזורים – שזו המקבילה הקוונטית לתיאום מוקדם של קוד, רמאות לפי חוקי המשחק. ואתה בכל זאת האשמת אותי שוב ושוב בכך שאני טוען שיש פתרון קלאסי״.

    קצת קשה לדעת למה הוא מתכוון כי בניגוד אלי שתמיד מביא ציטוטים במקור, אין כאן ציטוטים וצריך לסמוך על הכותב שהוא יודע מה הוא אומר. אבל אם מישהו יבקש, אשמח להביא את אותם ״ציטוטים אשר באופן פלאי תמיד היה נחתך לך בול באמצע המשפט״. אני זוכר אותם ויודע גם איפה הם נמצאים ומאמין שהם רק יחמירו את מצבו של אלבנצו וטענות השקרן שלו.

    ״דוגמא נוספת היא שבמהלך דיון ארוך על פתרון חידת מטבעות כזו או אחרת בעזרת שזירה, טענת שאני אומר שיש לה פתרון קלאסי למרות שכתבתי במפורש פעמים רבות שאין לה פתרון קלאסי. רק ניסיתי להסביר לך שהפתרון הקוונטי לא כולל העברת מידע כי הוא דורש סנכרון מראש של מספר עצום של זוגות שזורים – שזו המקבילה הקוונטית לתיאום מוקדם של קוד, רמאות לפי חוקי המשחק. ואתה בכל זאת האשמת אותי שוב ושוב בכך שאני טוען שיש פתרון קלאסי״.

    והנה מה שאלבנצו כתב:

    ״אין שום הבדל בין פתרון זה לבין כל פתרון אחר של קוד קלאסי מתואם מראש (כפי שניסו חלק מהמגיבים פה להציע), מלבד זה שהקוד הזה פועל לפי חוקים קוונטים, ובפרט הוא מצוי בסופרפוזיציה (כידוע סוד הקסם של שזירה היא שהיא מצב סופרפוזיציה, ולא סתם אלא סופרפוזיציה מיוחדת שהיא לא ספרבילית לתתי מערכות טהורות)״.

    אני מוכן להתערב עם מי שרק רוצה על כל סכום שהוא שאם ניקח מדגם מקרי של 100 אנשים ונקריא להם את המשפט האחרון, הרוב הגדול יחשוב שהכוונה היא לכך שאין שום הבדל בין קוד קלאסי מתואם מראש לפיתרון הקוונטי, מלבד כמה פרטים שוליים. בודאי שלא בעיקר כפי שגם אלבנצו מסכים – שבאמצעות הפיתרון הקוונטי ניתן להשפיע מרחוק באפס זמן, שזה נושא הדיון.

    ניסוח גרוע של אלבנצו, לא שקר שלי. אם כדבריו ״ הבנת הנקרא שלך לוקה. כשאומרים “אין הבדל בין משה לדני מלבד זה שמשה בלונדיני ודני ג’ינג’י”, האם זה אומר שמשה ודני הם זהים? האם כשתפגשו אותם תדע להבדיל ביניהם או לא? ״ אבל אם שכחת לציין שהג׳ינג׳ם ידועים ביכולתם להשפיע מרחוק באפס זמן שזהו נושא הדיון, אז הבנת הנקרא שלך דוקטור לוקה. זה כמו להגיד אין הבדל בין הסרט טיטניק מ1953 לסרט טיטניק מ1997 חוץ מזה שבסרט מ1997 הגיבורים הם בלונדיני וג׳ינג׳ית. הגיבורים האלו – ג׳ק ורוז – הם כמעט הדבר החשוב ביותר בסרט.

    ״הנה דוגמא מעולה לעיוות מטורף של דברי ואפילו המצאה: “אתה טוען שבסבטקסט הם התכוונו בעצם להיפך ושאין איום ושהן מסתדרות מצויין ושכותבי המאמר פברקו משהו מדומיין כדי להגדיל את התפוצה?”. אתה יודע היטב שמעולם לא אמרתי שום דבר כזה. לא כתבתי שהם פברקו כלום. לא כתבתי שהם התכוונו ההיפך״.

    והנה מה שכתב אלבנצו כמה ימים קודם:

    ״קודם כל, צריך לזכור שכל מאמר במגזין – גם מדעי, ובטח שבסיינטיפיק אמריקן שפונה לקהל הרחב ויש לו אינטרס ענק למכור עותקים – יכול ליפול לנטייה טבעית לכותרות סנסציוניות או בומבסטיות״.

    לא מאוד דומה?

    ״אתה שיקרת פעמים רבות, כי לרוב הטיעונים שלך ריקים מתוכן, בגלל הנטייה החזקה שלך לדבר על דברים שאין לך מושג בהם. בדיוק כמו שקורה עכשיו, כאשר אתה כותב עוד ועוד דברים על אינפורמציה כאשר ברור לכולם – כולל לך – שאתה פשוט לא יודע כלום על תורת האינפורמציה, כולל ההגדרות הבסיסיות ביותר של מה היא אינפורמציה או מה הוא מעבר אינפורמציה״.

    האם טענתי אי פעם שאני מכיר את תורת האינפורמציה? או שבכלל ידעתי שקיימת תורת האינפורמציה לפני שאלבנצו הזכיר אותה? אני דיברתי על אינפורמציה כמו שכל אחד אחר פה מדבר עליה.

    ומכיוון שגם סקוראי וביולוגים ועוד רבים אחרים משתמשים במילה אינפורמציה בלי להביא את ההגדרה המדוייקת שלה, איני מרגיש יוצא דופן במיוחד, או שאולי כולנו בעצם שקרנים חסרי בושה.

    בשורה התחתונה, אדם יכול להגיד משפט שאינו אמת וזה לא הופך אותו לשקרן. שקרן הוא זה שבתנאים נורמטיביים נכשל במכונת שקר. אלבנצו אמר משפט שאינו אמת – ״שזה שלא עוברת אינפורמציה בין חלקיקים שזורים זו לא דיעה. יש הוכחה מתמטית״. האם זה הופך אותו לשקרן? לדעתי לא, כי זה מה שהוא מאמין, או לפחות האמין בזמנו.

    ויש גם את הכתבה בסיינטיפיק, שאומרת בדיוק את מה שאני אומר תמיד, אז אם אני שקרן כנראה שגם הם..

    בקיצור – לא שקרן ולא בטיח. איני מחפש מריבות, ומקווה שבזאת נסגר הנושא אחת ולתמיד.

  4. ישראל
    זה בדיוק מה שאני אומר – אין קשר בין פשטות וצדק. מה שאני מנסה להגיד שגם התורה של ניוטון לא תמיד פשוטה.

  5. היו לנו הרבה כאילו בקיבוץ. גם תרנגולות מרובעות וכבשים משולשות.

    תורת היחסות דווקא מאד יפה והקוונטים לא, אז מי יותר נכונה?

  6. ישראל
    כתבתי. הפיסיקה של ניוטון לא ידעה לחשב מסלול של 2 פלנטות.
    אני לא מצליח להבין מה טוב התיאוריה פשוטה, הגיונית ושגויה…

  7. ישראל
    רצוי שתורה פיסיקלית תהיה פשוטה? אנחנו לא יודעים לחשב את המסלולים של 2 פלנות ושמש …. מה יותר פשוט מזה?

  8. אמרתי שאין עדות ישירה להתקצרות האורך, וזאת בניגוד לעדות ישירה שקיימת להתארכות הזמנים.

    בעצם לא אני אמרתי את זה – ויקי אומרת.

    “וכמו שאמרת “שורש של הסתברות” זה משהו מוזר”.

    אמרתי ריבוע וזה לא מוזר, ככה מוצאים הסתברות.

    “הבאג שאני מנסה לפתור עכשיו הוא מוזר – זה לא אומר שהוא לא קיים”.

    זה אכן מוזר, אפילו ההסבר שלך על אלפא קאונטרי. לא אמרתי שזה לא קיים, אבל רצוי שתורה פיזיקלית תהיה ברורה ופשוטה, ראה תאוריות ניוטון ומקסוול.

    “זה כן אומר ש”מידע לא שימושי” יכול לנוע במהירות אינסופית”.

    התפקיד שלנו בכוח זה להפוך אותו לשימושי. רוצה – תצטרף. לא – נסתדר לבד.

    נוסעים לסנטה ברברה.

  9. ישראל
    התחלנו בזה שטענת שאחת הניבויים של תורת היחסות הפרטית – כיווץ האורך – הוא מוזר, ולא מוכח ניסויית.
    הראתי לך כמה תופעות שאנחנו לא יודעים להסביר, ללא התכווצות האורך. האמרירה “זה מוזר” זה לא משהו שאני יודע להתמודד איתו. הבאג שאני מנסה לפתור עכשיו הוא מוזר – זה לא אומר שהוא לא קיים.

    אתה לא מסכים עם מה שאיינשטיין טוען לגבי תורת היחסות הפרטית, אבל כן מסכים עם דעתו כשזה נוח לך. לי נראה שההבנה שלו לגבי משמעות EPR הוא שגוי. הסתברויות יכולות להשתנות במהירות אינסופית. יש הסתברות מסויימת שהמפתחות שלי על השולחן – כי אני מניח אותם לפעמים שם ולפעמים במגירה. בדקתי – והנה הם במגירה. ב-0 זמן, ההסתברות שהם על השולחן ירדה ל-0 🙂

    אני יודע שזה לא כל ההסבר, וכמו שאמרת “שורש של הסתברות” זה משהו מוזר. אבל, זה כן אומר ש”מידע לא שימושי” יכול לנוע במהירות אינסופית.

  10. אצלי? איפה, במחסן? למה מי אני, רמי לוי?

    אבל כן, יש לה קיום עצמאי גם ללא היחסות, ועוד מלפני 1905.

    https://en.wikipedia.org/wiki/Olinto_De_Pretto

    איני בטוח שאנחנו מסונכרנים לגבי התפקיד שלנו בכוח.

    זה לא התפקיד שלנו להסביר כל דבר. אין לנו את הכלים, ההכשרה, הנסיון, הזמן, או הכישרון לכך.

    הבעיה שלנו פשוטה בהרבה: איינשטיין כותב מאמר – EPR – שבו הוא טוען שקוונטים חרטה בגלל שעל פיה חלקיקים משפיעים זה על זה מהר מהאור. הוא מציע ניסוי מחשבתי כדי להוכיח את טענתו.

    וריאציה של הניסוי מבוצעת וקוונטים מנצחת.

    על פי קוונטים, לפוטון אין מיקום לפני המדידה והוא מרוח בהסתברות שווה ביקום.

    ניס שואל:

    ״האם אתה מקבל שאני יכול לפלוט פוטון מנקודה A וכעבור זמן מתאים לקלוט אותו ב-B?״

    ישראל עונה:

    ״ללא ספק. אתה יכול לפלוט פוטון מנקודה A וכעבור זמן מתאים לקלוט אותו ב-B.

    אבל אם הוא מרוח במרחב לפני כן, ועוד בהסתברות שווה, אז יש שתי אפשרוריות:

    1. מציאות הפוטון מעבר לB אינה משהו פיזי – זו רק פונקציית הגל או הפילוסופיה או הזברבירולוגיה, אך לא הפוטון עצמו.

    (קצת מוזר אם נחשוב על כך שריבוע פונקציית הגל היא ההסתברות למצוא את החלקיק בנקודה מסויימת).

    2. החלקיק עצמו נמצא גם מעבר לB.

    במקרה 1, אז אין כל ממשות לפוטון שנע מהר מc, איינשטיין צודק וקוונטים חרטה. אך להבנתי, לא זה המצב.

    במקרה 2, אם הפוטון עצמו נמצא מעבר לB בזמן פחות מct, אז אין מנוס מהמסקנה שהוא נע מהר מc ולמעשה בכל מהירות שהיא״.

    התפקיד שלנו הוא פשוט יחסית: לגלות את החלק של הפוטון שנמצא מעבר לB.

    קפיש?

  11. ישראל
    חובב פשטות? מה ההסבר הפשוט למיואונים? למה כשאני מעלה שעון על בניין גבוה הוא רץ יותר מהר? למה האור מתעקם ליד השמש? למה המסה גדלה עם המהירות?

  12. ברור שזה לא פוגע בקוזיליות, אבל השפעה על העבר? לא אמרנו שאילי פאת – מאת?

    אתה לא רואה שזה רק מסתבך והולך, הולך ומסתבך, מסתבך והולך ומסתבך?

    ובתור חובב פשטות, אינך רואה שפוטון הנע בכל המהירויות פותר את רוב הבעיות כולל השפעה על העבר?

  13. ישראל
    או שעבר ב-0 זמן או שהחלקיק נמצא בו”ז בשתי מקומות. מה שעבר, אם עבר, אינו חלקיק, ובכלל לא בתחום עניינו של תורת היחסות.

  14. https://www.hayadan.org.il/quantun-philospy-part-b-07121

    והמשהו הזה שמשותף לחלקיקים לא עובר מאחד לשני באפס זמן? הרי הוא התחיל בחלקיק אותו מדדנו ועבר לשני מיד.

    פיינמן כבר אמר שאף אחד לא מבין את קוואנטים, אבל לפחות בנושא הזה של השפעה מידית מרחוק היא הוכחה כנכונה וזאת בניגוד (לפחות על פי ויקי) ליחסות.

  15. ישראל
    יש משהו משותף בין החלקיקים. אבל, איפה השפעת העתיד על העבר?

    אני לא מבין איך משהו “פשוט” כמו ניסוי שני החריצים כן מסתדר לך….

  16. יש גם את ההסבר הפשוט של המגנטיות שמופיע בלינק שהבאתי.

    ובקשר לאלפא – בזום רגיל בשלב מסויים יעלמו רוב הכוכבים מהתמונה, אבל כאן זה לא יכול לקרות.

    אגב, אם תרצה אני מנוי של טלסקופ אונליין שתוכל לראות בשידור חי כל כוכב וגלקסיה לפי בחירה. אוכל לתת לך את הפרטים, זה די נחמד.

    אבל עדיין איני מבין: אתה לא מקבל שבשזירה חלקיק א׳ משפיע על חלקיק ב׳ ב0 זמן? אפילו אם לא נקרא לזה העברת אינפורמציה, למרות שאיך שהוא הוא מסדר את חלקיק ב׳ באותו מצב קוונטי כמו שלו, האינך רואה עד כמה זה מאתגר את היחסות שעל פיה שום דבר לא יכול לנוע מהר מהאור (נעזוב טכיונים עכשיו).

    ואינך רואה שזה מוביל להשפעה על העבר מהעתיד? אתה לא מאמין שאילי פאת – מאת?

  17. ישראל
    הנה עוד הסבר:
    https://readingfeynman.org/2015/09/06/magnetism-and-relativity/

    אני למדתי את זה מזמן, מהספרים של ברקלי. אין לי את הספרים איתי….

    בקשר למצלמה. אני לא מבין איך הרעיון שלך נותן תוצאות שונות. נניח אני מתקרב במהירות קרובה ל-c לירח. נניח שיש שם משדר שמשדר כל שנייה את הזמן. זה שקול לפוטון עם שעון, נכון? גם לפי השיטה שלך, הירח והפוטונים נעים במהירויות קרובות. כלומר, אני אקלוט את הפוטון של שעה 12:00 מאד קרוב לשעת ירח 12:00. מה המסקנה שלי? שהירח קרוב מאד.
    כלומר – גם בגישה שלך יש כיווץ מרחק…

  18. הבעיה היא שעל פי הוידאו שהבאת רק מנקודת מבטי המטענים מצטופפים..

    וגם לא הבנתי איך מצלמת הוידאו שבאייפון שלי יודעת שדווקא את אלפא קאונטרי אני רוצה לצלם ולכן מגדילה רק אותו ולא איזה 1000 כוכבים אחרים בסביבה..

  19. ישראל
    נקודת ראותך לא משפיעה על ערך הכוח. תחשוב על המקרה ההפוך – תאר לך שני זרמים של אלקטרונים בריק. למה אין ביניהם משיכה מגנטית (להבדיל מהזרם בחוטים)?

  20. ישראל
    המצלמה לא יודעת כלום. היא קולטת פוטונים שנעים במהירות האור. אם הם היו נעים מעל למהירות האור, היינו מקבלים סתירה.

    נניח שני תילים מקבילים במישור, בכיוון צפון. נניח שבאחד מהם זורם זרם קבוע לכיוון צפון. כלומר האלקטרונים נעים דרומה.

    1) אין זרם בתיל השני. במקרה הזה הן הפרוטונים והן האלקטרונים רואים מטען שלילי בתיל הראשון. סכום הכוחות – 0.

    2) בתיל השני גם זרם בכיוון צפון. בשני התילים – האלקטרונים רואים יותר פרוטונים, וזה יוצר משיכה. וגם, הפרוטונים רואים רואים יותר אלקטרונים, שוב משיכה.

    3) בתיל השני זרם הפוך. האלקטרונים רואים יותר פרוטונים, אבל עוד יותר אלקטרונים, ולכן דחיה. הפרוטונים רואים יותר אלקטרונים וזה יוצר משיכה מסויימת. בסה”כ יש יותר דחיה, ולכן התילים דוחים אחד את השני.

    צריך להדגיש שזה לא אומר שאין שדות מגנטיים. שני סוגי השדה אמיתיים, אבל יש תלות במערכת היחוס.

  21. בעיני זה מוזר.

    מאיפה המצלמה יודעת שאני רוצה דווקא את הכוכב הזה בתמונה שמרוחק ממני 4 שנות אור? למה לא את זה שלידו?

    ומה זה משנה לכוח מהמטענים בתיל המוליך שאני נע יחסית אליהם? בסדר, מנקודת ראותי הם מצטופפים, אבל נקודת ראותי אמורה להשפיע על הכוח הפיזי?

    לא בדקתי, אבל זה בדוק שאם לדוגמה כיוון התנועה של המטען או כיוון הזרם יתהפכו אז הכוח יתהפך גם הוא ממשיכה לדחיה?

  22. ישראל
    אני מניח שבזה אתה צודק. רק שים לב שזה אומר שבתנאים “רגילים”, תורת היחסות הפרטית תשאר נכונה (כמו שתורת ניוטון נכונה בתנאים מסויימים).

    דרך אגב – אני לא מבין מה מוזר בעיוות של התמונה. מאד הגיוני לחשוב שאם אתה נע קרוב למהירות האור אז תראה דברים מוזרים. גם לגבי קול זה ככה. התסכל על גלים של ספינה איטית, שנראות כמו אדוה מאבן שנזרקה, לעומת ספינה מהירה שיוצרת V על המים. כנ”ל לגבי מטוסי קרב – רואים בעיניים דברים מאד מוזרים באוויר על-קולי, כמו יהלומים בגזי הפליטה, או קווי שבר באוויר שליד הקונסים. טיסה על-קולית יוצרת בום מוזר, וחלקיקים שנעים מעל למהירות האור (באותו תווך) יוצרים קרינה מוזרה. מוזר לא אומר שמשהו לא קיים.

  23. אין פה טענה לחוסר סתירה, וכפי שכתוב בסיינטיפיק, האיום הקוואנטי על היחסות התעורר הרבה שנים אחרי דיראק.

    שים לב גם שמה שאני אומר הוא שהיחסות תצטרך כנראה לעבור רוויזיה ותיהפך למקרה פרטי של תיאוריה כללית יותר שתכלול אי לוקליות כמו שקרה לתאוריית ניוטון שהיא מקרה פרטי של היחסות.

  24. ישראל
    אצטט שוב –

    In fact, QM would make grossly inaccurate predictions if Dirac hadn’t shown up and tied QM together with special relativity to create “relativistic QM”.

    General relativity, on the other hand

    זה מוזר, אבל זה לא אומר שזה לא נכון. תחשוב שאתה מתקרב לפוסטר גדול. תוכן התמונה לא משתנה, אבל היא מאד מתעוות. זה לא שונה בהרבה מצילום במהירות גבוהה.

  25. ״המאמר טוען שאין שום סתירה בין תורת היחסות הפרטית לתורת הקוונטים, אלא להיפך״.

    איפה, איפה הוא טוען את זה?

    אז למי יש מאמר יותר גדול?״

    לסיינטיפיק. בערך פי 30.

    ״אתה תראה את כל הכוכבים שראית לפני כן, ולא תראה אף כוכב שלא ראית לפני כן״.

    אבל מקודם אמרת שאם גאמה שווה 10 אז הירח יתפוס פי 10 מהתמונה..

    אז אם גאמה שווה מיליון הוא לא ימלא את כל התמונה?

    מה זה, כל הכוכבים יתכווצו לזווית התמונה?

    כפי שאמרתי, מוזר מאוד.

  26. ישראל
    כבר אמרתי – המאמר מדגיש שהבעיה היא בתורת היחסות הכללית ולא בפרטית. אם אתה מתעקש לקרוא כל משפט בהנחה שמה שאתה מאמין בו הוא נכון, אז אין טעם לדון על זה.

  27. ישראל
    כתבת:
    “ובוא נאמר שאתה מצלם את הירח כשגאמא שווה מיליון.
    מה תראה בתמונה עם הזום היחסותי הזה? רק את הלוע של טיכו?
    אז לאן נעלמו כל יתר הכוכבים שברקע? הרי פוטונים מהם מגיעים לעדשת המצלמה גם כן, אז לאן הם נעלמו?”

    אתה תראה את כל הכוכבים שראית לפני כן, ולא תראה אף כוכב שלא ראית לפני כן.

  28. רק את הכותרת? קראתי את כל הפסקה הראשונה:

    Physicist: Quantum Mechanics (QM) and relativity are both 100% accurate, so far as we have been able to measure (and our measurements are really, really good). The incompatibility shows up when both QM effects and relativistic effects are large enough to be detected and then disagree. This condition is strictly theoretical today, but in the next few years our observations of Sagittarius A*, and at CERN should bring the problems between QM and relativity into sharp focus.

    במיוחד המשפט האחרון:

    should bring the problems between QM and relativity into sharp focus.

    מי זה הפיזיקאי הזה אגב?

    קרא עוד הפעם את המאמר בסיינטיפיק, הוא הרבה פחות מבולבל מהכתבה הזו. הוא מסביר שבזמנו באמת חשבו שאין יותר בעיה בין היחסות לקוונטים, אבל בשנים האחרונות (עשרות שנים אחרי דיראק) ההכרה שכנראה יש בעיה הולכת ומתחזקת אצל פילוסופים ופיזיקאים.

  29. ישראל
    מה יהיה? תפסת מילה אחת בכותרת המאמר, ואתה אוחז בה כמו בולדוג…

    כנראה שאני צריך לתרגם לך את הקטע שהבאתי. ובכן – כתוב בפרוש שאין סתירה בין תורת היחסות הפרטית לתורת הקוונטים. כתוב שלהיפך! פול דיראק הראה שחייבים את תורת היחסות הפרטית כדי להסביר תופעות קוונטיות.

    איפה כן יש בעיה? אם תורת היחסות הכללית. דוגמה אחת – איבוד מידע בחור שחור (מלחמת סוסקינד-הוקינג). דוגמה שניה היא מה שהסביר אלבנצו – בעיות הסינגולריות.

    ישראל, באמת, תתיחס למה שכתוב. אל תיתפס לכל מילה כי אתה חושב שהיא מחזקת את דעתך.

  30. ישראל
    בודאי שקראתי. ציטוט:
    In fact, QM would make grossly inaccurate predictions if Dirac hadn’t shown up and tied QM together with special relativity to create “relativistic QM”.

    General relativity, on the other hand,

  31. נו ניסים באמת, אתה חייב לשים לב למה שאני כותב לפני שאתה מגיב, אחרת זה נשמע קצת מאותגר..

    ״עכשיו אתה כן אומר שיש סתירה? חשבתי שסיכמנו שסעיף 1 הוא רק מוזר…״

    סעיף 1 – כיווצוץ – מוזר.

    סעיף 5 – קוואנטים ואי לוקליות – סתירה כביכול.

    קפיש?

  32. ניסים

    עוד הפעם גלוג גלוג?

    רק על סעיף 1 אותו הזכרת – התקצרות האורך – אמרתי מוזר, אבל ללא סתירה פנימית.

    היו עוד כמה סעיפים כולל הסעיף ממנו אתה מתעלם כל הזמן: המאמר בסיינטיפיק, שאומר משום מה בדיוק מה שאני.

    כל מה שעשיתי זה להראות את שורש הסתירה כביכול, והפעם ללא שזירה ומהירות האור.

  33. ישראל
    איפה דברת שטויות? איפה לא? ?

    לסיכום, כל מה שיש לך להגיד זה “זה מוזר”. ולא משנה מה יגידו לך, כולל ניסויים שמאששים בצורה מדהימה את תורת היחסות.

    אני מבין שאתה מנסה לעלות היפותיזה שמתאימה גם לתצפיות יחסותיות וגם לתצפיות קוונטיות. אתה לא היחיד – שמוליק תיאר היפותיזה של פנרוז, ואלבנצו הזכיר את נושא המיתרים.

    הבעיה היא – שמה שאתה טוען נגד תורת היחסות הוא, לדעתי, לא נכון. אתה נותן ניסוי מחשבתי הזוי, ואתה מופתע שהניסוי מראה משהו הזוי. אתה טוען שאין כל ראיות לכיווץ האורך, ומתעלם ממספר ראיות לכך, אחד מהם היא ראיה מאד חזקה (מיואונים מהחלל). ויש עוד.

  34. ניסים, אתה חייב להתכנס.

    עבור על כל התגובות – איפה אמרתי שאתה טועה? אתה זה שאמרת שמה שאני אומר שגוי אך כשהבנת הסכמת שהכוכב יראה גדול וקרוב.

    בוודאי שקוונטים מוזרה, שאל את איינשטיין, הוא נלחם בה כל חייו – והפסיד.

  35. ישראל
    כן, מוזר. תורת הקוונטים הרבה יותר מוזרה לדעתי.

    אבל, עוד לא הסברתי לי למה אני טועה בסעיף הראשון שלי.

  36. ישראל
    בוא נניח שהירח ב-90 מעלות בצד. אלפא סנטאורי יראה ענק. הירח יראה מאד צר (כי הוא יראה מקווץ רק בציר 1).
    כמו שתארתי קודם – צייר את זה על חתיכת גומי, ואז תמתח/תכווץ כמה שתרצה.

    לא תגיע לשום דבר לא הגיוני.

  37. ובוא נאמר שאתה מצלם את הירח כשגאמא שווה מיליון.

    מה תראה בתמונה עם הזום היחסותי הזה? רק את הלוע של טיכו?

    אז לאן נעלמו כל יתר הכוכבים שברקע? הרי פוטונים מהם מגיעים לעדשת המצלמה גם כן, אז לאן הם נעלמו?

    ???

  38. אבל למה פי 10? הרי כתבתי:

    the star Alpha centauri (4 light years away) fast enough so that gamma factor:

    is equal to 1000,000, and you take a picture of the star from earth. Since length contraction is only in the direction of your movement and not in the perpendicular coordinates, what will your picture show? — a huge star closer than the moon?

    לא כתוב מיליון?

  39. שלילי.

    אני מצלם את הירח עכשיו מהארץ, הוא ממלא מאית מהתמונה.

    מה יהיה גודלו אם אצלם אותו במצלמה זהה מחללית החולפת על פני הארץ כשגאמא שווה ל10? כמה מהתמונה הוא ימלא?

  40. ישראל
    גם הכוכב מתקרב וגם הירח מתקרב. דמיין שאתה מצייר ליקוי חמה על משטח גומי. אתה מצייר את הקונוס שמראה את ההסתרה. עכשיו – תמתח את הגומי בצורה אחידה. לא חשוב איך תעשה את זה, קווים ישרים ישארו ישרים.

    קל להוכיח אפיניות – קח 3 נקודות על קו ישר ותבצע את הטרנספורמציה. אבל ……. הנה ציטוט מויקיפדיה (https://en.wikipedia.org/wiki/Length_contraction)

    In special relativity, Poincaré transformations are a class of affine transformations which can be characterized as the transformations between alternative Cartesian coordinate charts on Minkowski spacetime corresponding to alternative states of inertial motion (and different choices of an origin). Lorentz transformations are Poincaré transformations which are linear transformations (preserve the origin)

    כלומר – לא רק אפיני, גם לינארי.

    אפשר לסכם את נקודה 1?

  41. אז בוא נגיד שהטרנספורמציה היא אפינית. הכוכב קרוב יותר או לא?

    כי בד”כ כלל כשעצם קרוב יותר אז הוא נראה בצילום גדול יותר. לזה אתה מסכים? אז למה אם כל המרחבים האוקלידים הם אפינים זה יהיה שונה במקרה הזה?

    אז אתה טוען שבמקרה הזה בגלל האפיניות של המצב הוא לא יראה גדול יותר? וזה בעיניך לא מוזר ביותר? ולמה איינשטיין לא מציין את האפיניות?

    תוכל להביא קישור בין התקצרות האורך ואפיניות?

  42. ישראל
    אז זהו, שלא. הסברתי כבר כמה פעמים למה, אז אנסה שוב, בדרך אחרת. בוא נניח שהתנועה היא לאורך ציר x, במהירות גאמא = 2. יש לנו כאן טרנספורמציה:

    x => x/2
    y => y
    z => z

    הטרנספורמציה הזו היא אפינית. זה אומר שמתקיימים כאן כמה דברים. למשל – קווים מקבילים נשארים מקבילים. למשל – נקודות מישוריות נשארות משוריות. למשל – וזה מה שחשוב לנו – נקודות קוויות נשארות קוויות.

    נתתי את הסיפור של החללית וליקוי החמה. במקרה של ליקוי, אם נסתכל מנקודת מבטו של הצופה בליקוי, אז כל נקודה בהיקף הירח מסתיר נקודה מתימה על השמש. כלומר – 3 נקודות על קו ישר.
    ולכן – כשהחללית ליד הצופה, עדיין יהיה ליקוי.

    QED

    אז, האם טענתך שטרנספורמצית לורנץ אינו אפיני?

    ישראל – שים לב. אני לא טוען שיש כיווץ במרחק. אני כן טוען שמה שסיפרת על הכוכב שנהיה גדול מהירח הוא שגוי.

  43. ניסים

    הלשגעני אתה אומר כפי ששגעת את המצרי?

    מה שגוי? כאשר אתה נע לכיווו כוכב המרחק אליו אינו קצר יותר מבחינתך? ואם המרחק קצר יותר, האין הכוכב קרוב יותר? והאין עצם קרוב נראה בצילום גדול יותר? רוצה לנסות עם הבניין ממול?

  44. אוף ניסים פעם אחרונה, אם תבין – תבין. אם לא אז לא.

    אין סתירות ביחסות פרופר – אך המסקנות שלה מוזרות ביותר והתקצרות האורך היא אולי המוזרה מכולן.

    אם אתה נע מהר לכיוון כוכב אז המרחק אליו מתקצר, הרוחב לא.

    לכן, אופטית, מה שקרוב יותר נראה גדול יותר, ועל פניו נראה את הכוכב גדול יותר.

    האם זה כך? יתכן. אולי לא. מה זה משנה? זה מוזר ביותר, בניגוד לתאוריות ניוטון מקסוול בולצמן וברנולי שאינן מוזרות. זו כל הפואנטה בנקודה המסויימת הזו. זו אינה סתירה כמו הקוואנטים, רק מוזרות.

    קפיש?

  45. ישראל
    אני ממש מצטער, אבל לא הבנתי. האם אתה טוען שלפי תורת היחסות הפרטית אזי הכוכב צריך להראות יותר גדול מהירח?

  46. ניסים

    נסה לזכור מדי פעם שיש קשר בין המילים למציאות..

    ״עכשיו אתה אומר שלא כתבת את זה??״

    בודאי שכתבתי את זה. אחרת מאיפה עשית העתק הדבק, לא מאיפה שכתבתי?

    וגם כתבתי:

    what will your picture show? — a huge star closer than the moon?”

    זו שאלה, והיא מראה את מה שאני טוען – זה מוזר מאוד.

    אם אתה חושב שההסבר שהבאת ״הטרנספורמציה היא אפינית. בפרט, זה אומר ששלוש נקודות שהן על קו ישר לפני הטרנספורמציה תשארנה על קו ישר גם אחרי״ היא הנכונה, אז זה לא ההסבר היחיד. גם ההסבר שהבאת על המטענים שהמרחק ביניהם מתקצר מה שגורם לכוח על גוף טעון בתנועה אינו מלא, וזה גם מופיע בקישור שהבאתי.

    עבודה.

  47. ישראל
    כתבת
    ” Imagine that you move toward the the star Alpha centauri (4 light years away) fast enough so that gamma factor:

    is equal to 1000,000, and you take a picture of the star from earth. Since length contraction is only in the direction of your movement and not in the perpendicular coordinates, what will your picture show? — a huge star closer than the moon?”

    עכשיו אתה אומר שלא כתבת את זה??

  48. לא.

    אם זה היה נכון היינו מקבלים סתירה ביחסות עצמה, לא?

    נראה לך שאיינשטיין היה מפספס סתירה ביחסות פרופר?

    ואולי תענה כבר על השאלה בקשר למאמר? איך זה ששנינו אומרים את אותו הדבר?

  49. ישראל
    מה שאני אומר זה הדבר הבא:
    בוא נניח שיש ליקוי חמה. חללית מתקרבת במהירות קרובה ועוברת ליד כדור הארץ, כשהיא טסה לכיוון הליקוי. אני טוען שהחללית גם תראה את הליקוי ברגע המעבר ליד כדור הארץ. ממה שהבנתי – אתה טוען אחרת.

    האם הבנתי אותך נכון?

  50. בוא באמת נעבוד מסודר, ותשתדל לראות את כל התמונה.

    אז אתה כותב:

    1) אתה כותב שכוכב מרוחק אמור להראות גדול יותר מהירח. זה פשוט לא נכון. הטרנספורמציה היא אפינית. בפרט, זה אומר ששלוש נקודות שהן על קו ישר לפני הטרנספורמציה תשארנה על קו ישר גם אחרי.

    ומכאן ניתן להסיק שאני טוען שיש בעיה עם התקצרות האורך, לזה התכוונת, נכון?

    אז הנה מה שכן כתבתי:

    Even though there are nomathematical inconsistencies within the theory, physically relativity is so strange that many physicists found it very difficult to accept. Theoretical physicists such as Abraham, Lorentz, Poincaré, and Langevin still believed in the existence of an ether.
    For example, Lorenz length contraction is especially difficult to comprehend and has no direct experimental support; http://math.ucr.edu/home/baez/physics/Relativity/SR/experiments.html#Length_Contraction
    Tests of length contraction
    At this time there are no direct tests of length contraction.

    תרגום: למרות שאין סתירות מתמטיות בתורת היחסות עצמה, היא מאוד מוזרה פיזיקלית.

    ואז אני עובר לדבר על התקצרות האורך, מזכיר את העובדה (מויקיפדיה) שאין לה עדות נסיונית ישירה, ונותן את הדוגמה שבה אתה נע מול אלפא קאונטרי במהירות ושואל האם הוא יראה גדול מהירח.

    האם אני טוען כאן שהתקצרות האורך אינה קיימת? בודאי שלא, הרי אמרתי כבר קודם שאין סתירות ביחסות כשלעצמה, הבעיה היא רק העימות עם תאוריות אחרות.

    התקצרות האורך הובאה רק כדי להראות שהיחסות מביאה למסקנות שעל פניהן הן מוזרות ונראות קשות מאוד לעיכול. ראה כמה הסתבכנו אנחנו עם פרדוקס פרדוקס התאומים ועד כמה הסתבכו איתו 4 פרופסורים וכל אחד הביא לבסוף הסבר הפוך מחברו.

  51. ניס, עצור בסוסיך..

    אולי במקום לצעוק ״אתה לא מבין, אתה לא מבין!״ ולהגיד שישראל מבין יותר טוב מאיינשטיין, קיימת גם האפשרות שניסים לא לגמרי מבין מה אומר ישראל? 🙂

    איפה דברת שטויות? איפה לא? 🙁

    מפתיחת ההרצאה:

    At the end of the presentation you will realize I believe, that if Einstein was wrong at all, it was a very reasonable mistake that he made.

    I also must add that chances are that he was not wrong at all, except maybe on the subject of illocality, and I simply don’t understand the issue good enough.

    יעזור אם תענה על השאלה ששאלתי קודם:

    ״אז אולי אתה, המסביר הלאומי, תסביר לי איך זה מתפרסם בסיינטיפיק מאמר שאומר בדיוק מה שאני אומר? צירוף מקרים? מקרה ממוזל? קסם?״

    איני שואל אם זה נכון או לא, או אם אתה מסכים או לא, רק איך זה קרה ששנינו אומרים בדיוק את אותו הדבר? אני לא קראתי את המאמר עד לפני כמה שנים ויש לי הרגשה שהם גם כן לא קראו את מה שאני אומר בעיון..

    אז איך זה יצא בול?

  52. ישראל
    1) אתה כותב שכוכב מרוחק אמור להראות גדול יותר מהירח. זה פשוט לא נכון. הטרנספורמציה היא אפינית. בפרט, זה אומר ששלוש נקודות שהן על קו ישר לפני הטרנספורמציה תשארנה על קו ישר גם אחרי.

    2) אתה כותב שאין ראיות לקיווץ המרחק. גם זה לא נכון. בויקיפדיה יש תיאור של 5 מיקרים שיש לכך ראיות. ברור שאין מדידה “ישירה” – אין לנו היום דרך להאיץ גופים מדידים למהירויות יחסותיות.

    3) החישובים של מקסוול נכונים כי הוא משתמש בשני קבועים שלכאורה הם בלתי תלויים. בתורת היחסות מראים שאחד מהם ניתן לחישוב מהשני.

    4) אתה כותב שהנחה שחוקי הפיסיקה זהים בכל מערכת אינרציאלית אינה נכונה בגלל קרינת הרקע הקוסמית. אני לא מבין את הסתירה כאן. תמיד תוכל לבחור מערכת שלישית כרפרנס – איך זה משפיע על הנחות היסוד של תורת היחסות? יש לי שעון הל היד – אז בגלל זה כל תורת היחסות הפרוית אינה נכונה? ברצינות? 🙂

    5) אתה ממש לא מצליח להבין את “פרדוקס” התאומים. בבקשה – נסה להבין מה שאני מתאר עכשיו. אני על כדור הארץ ואחי על הירח – לשנינו שעונים מתואמים. חללית במהירות גאמא = 10 עוברת את הכדור הארץ ואחרי זה את הירח. נניח המרחק לירח 10 שניות אור.
    1) החללית עוברת אותי בזמן ואת אחי בזמן t+10.
    2) החללית תראה זמן u כשהיא חוצה אותי, וזמן u+1 כשהיא מגיעה לירח.

    מבחינת טמפרטורה – קרינת הרקע הקוסמית מספקת מערכת יחוס נוספת. אנחנו יודעים שהמהירות שלנו יחסית לקרינה הזו היא 370 ק”מ לשנייה, לכיוון מזל אריה. החללית תגלה שהיא נעה יחסית לקרינה הקוסמית במהירות עצומה לכיוון מסויים. כלומר – היא תראה הסחה גדולה לכחול בכיוון התנועה שלה.

    6) בעניין השזירה. תורת היחסות לא אוסרת תנועה מעל למהירות האור. היא אומרת שלא ניתן להאיץ מסה ממשית מעל למהירות האור (כי היא גדלה לאינסוף), וכמובן שאסור שיהיו סתירות (מעבר אינפורמציה שמשפיעה על העבר.

    וכן – אני חושב שאנחנו מבינים ביחסות היום יותר מאשר איינשטיין הבין. אלבנצו נתן את הדוגמה של דארווין. מה הבעיה בזה? אתה הרי טוען שישראל מבין יותר טוב מאיינשטיין…..

  53. ישראל, מהו התחום ההבנה שלך בפיזיקה?
    לפי מה שאני רואה, אוליי כמה קורסים בסיסיים בפיזיקה קלאסית ואוליי אחד במבוא למכניקת קוונטים.
    האם אתה חוקר בנושא? האם פתחת איזשהו ספר שהוא לא ספר מדע פופלרי?
    לעניות דעתי (התבססות על שירשורים במשך כמה שנים וגם מאתרים שהם לא הידען שגם שמה אתה מגיב)
    התשובה היא לא חד-משמעית.
    גם כמוך, אני מתפלא מנסתרות הקוונטים והיחסות, ולא מבין למה האינטוציה שלי שגויה. ההבדל ביננו הוא, שאני מניח שההנחות שלי שגויות, ושואל למה ההנחות שלי שגויות. קל הרבה יותר ללמוד משהו אם אתה מניח שאתה טועה, ולנסות למצוא למה אתה טועה. אבל התפיסה שלך היא שהאינטואיציה שלך היא זאת שצודקת ולא המחקרים/התיאוריות שפותחו במאה האחרונה וממשיכות להתפתח היום.

    בכל מקרה אני מאוד ממליץ לך לא לנסות לערבב נושאים כמו יחסות וקוונטים כי גם לפיזיקאים שחוקרים את התחומים הללו עדיין מנסים לחפש תשובות לשאלות שאנשים כמוך שואלים. ההבדל הוא שהאנשים האלו עושים זאת למחייתם. הם מנוסים.

    הפסקה האחרונה לא נועדה לצנן את ההתלהבות והסקרנות שלך מהפיזיקה, ובטח לא להעליב אותך בשום צורה כלשהי. הסקרנות שלך היא ברוכה, אבל על מנת להשביע אותה, תצטרך לעשות טיפה מעבר לשאול אנשים באינטרנט שאלות על נושאים מורכבים כמו יחסות וקוונטים המצריכים בסיס יחסית מעמיק של מתמטיקה ופיזיקה קלאסית. תנסה להתחיל עם Berkeley Mechanics volume 1. בפרקים האחרונים יש כיסוי מעמיק על יחסות פרטית. תמשיך משם לכרכים 2 ו3. תסמוך עליי שלא תתאכזב.

    אני אומנם עדיין סטודנט לפיזיקה, אך אני מקווה שבשנים הקרובות התואר הזה יהפוך לטיפה מעבר. אבל בבירור, הדרך לשם היא בשאילת שאלות על נושאים שמעבר להבנתי, אלא במסע ארוך וממושך.

  54. אם אתה לא מבין אז כנראה שזה לא חשוב..

    השתחררה הממתינה. למזלך, הוורדפרס חתך את כל הנוסחאות האיורים והקישורים כך שיקח לך רק שעתיים לקרוא אותה.

    הרים, כלאבי.

  55. ישראל
    איזה ניסוי מראה אי התאמה? עזוב אותי ממאמרים – תשאל את כותבי המאמר אם משהו לא ברור. רק ניסויים וחישובים מעניינים כאן.

    אני לא מבין למה אתה מערבב את הקבוע הקוסמולוגי עם המיואונים המסכנים שלנו.

  56. התארכות הזמנים במיואונים:

    ״תורת היחסות הפרטית טוענת ששעונו של צופה הנע במהירות גבוהה מתקדם בקצב נמוך יחסית לצופה במנוחה. לתופעה זו קוראים התארכות זמן יחסותית, וניתן להדגים אותה בעזרת חלקיקים הקרויים מיואונים״.

    ״הקבוע הקוסמולוגי הוצג על ידי אלברט איינשטיין זמן קצר לאחר גיבושה של תורת היחסות הכללית על מנת לאפשר פתרון שלה שמתאר יקום סטטי״.

    מה שמראה שניתן הסבר שגוי כדי להצדיק תיאוריה קיימת וזה לא אומר שהתאוריה שגויה. זו הפואנטה – ואיני אומר שעקה אינה אולי ההסבר הנכון להתארכות הזמנים, אלא שאין לי צורך להסביר אותה. התפקיד שלי הוא רק להצביע על אי הקונסיסטנטיות של היחסות עם הקוואנטים.

    מה שמזכיר לי – עדיין לא ענית על השאלה:

    ״אז אולי אתה, המסביר הלאומי, תסביר לי איך זה מתפרסם בסיינטיפיק מאמר שאומר בדיוק מה שאני אומר? צירוף מקרים? מקרה ממוזל? קסם?״

    ובנוגע לאטמוספירה – קו המשווה מסתובב מהר מהקטבים. האם גם האוויר בשכבות העליונות של האטמוספירה מסתובב מהר מהאוויר בשכבות העליונות מעל לקטבים?

    לילה טוב.

  57. ישראל
    לא – זה ההסבר מהצד של צופה קרקעי. לגבי המיואון – ההסבר הוא כיווץ המרחק. מבחינתו – אורך חייו 2 מיקרו, ועובי האטמוספירה 600 מטר (ולא 6000).

    מה שייך הקבוע הקוסמולוגי? אין לזה קשר לתורת היחסות הפרטית.

    בעקרון, האטמוספירה מסתובבת עם כדור הארץ. כשיורים טיל לחלל, רואים את העשן שלו לגובה רב מאד. ראיתי מספר שיגורים בפלורידה ובקליפורניה, ובכולם העשן נשאר בערך במקום.
    רק אל תשכח שבגובה רב, אין הרבה אטמוספירה.

    בנוסף – האוויר בגובה משפיע על מסלולים של לווינים נמוכים. זה נראה שבגובה של לווינים האטמוסיפרה נעה מאד מהר בגלל הגובה, אבל …. הלווינים נעים הרבה יותר מהר. תחשוב על האבל. הטלסקופ בגובה 600 ק”מ, כלומר האטמוספירה נעה ב 10% יותר מאשר בגובה נמוך. מצד שני, האבל מקיף את כדור הארץ תוך שעה וחצי.

  58. ניסים

    ההסבר של הזמן מואט הוא גם ההסבר לאורך חיי המיואונים המופלג גם ביחסות.

    איינשטיין הביא את הקבוע הקוסמולוגי כדי להסביר את אי ההתאמה למודל שלו ואח״כ התלונן שזו הטעות הגדולה בחייו. האם זה אומר שהסבר לא קיים? (מפץ גדול).

    ומכיוון שהוזכרו מיואונים ואטמוספירה: ידוע למישהו אם האקסוספירה (החלק החיצוני באטמוספירה) מסתובבת ביחד עם הארץ? האם הגזים באקסוספירה מעל אנטארקטיקה מסתובבים לאט יותר מבחינתו של מיואון מאשר אילו מעל לקו המשווה?

  59. ישראל
    ההסבר שלך לא יותר טוב מ”למה – ככה”. בעיני, הוא הרבה פחות טוב. הוא לא מסביר למה אור מתעקם ליד מסה, הוא לא מסביר למה מסה גדלה עם המהירות, והוא לא מסביר למה כבידה משפיעה על הזמן.

    הוא גם לא מסביר למה המיואונים מצליחים לחדור את האטמוספירה. מה זאת אומרת הזמן מואט? למה שהחלב לא יתקלקל במהירות גבוהה??

  60. ״כדור הארץ בתנועה יחסית לקרינת הרקע״

    זניח. מיואונים נעים כמעט בc, כדה״א נע במאות קילומטרים/שנ ספורים יחסית לקרינה.

    ״מה זה קרינת הרקע? כמה פוטונים לכל סמ”ק? מה עם האטמוספירה עצמה?״

    לא רלוונטי. ביקשת הסבר אלטרנטיבי- קיבלת. אתה רוצה עכשיו עוד שבועיים של הסברים לא רלוונטיים לדיון על הסתירה כביכול בין היחסות לקוונטים?

    ״הבעיה הגדולה יותר היא שזה לא מסביר כלום. מה זה קרינת הרקע? כמה פוטונים לכל סמ”ק? מה עם האטמוספירה עצמה?
    ולמה זמן יתארך בגלל stress?״

    לא הבנתי את הרלוונטיות. אנחנו מנסים לראות מהי הסתירה כביכול בין היחסות לקוונטים. אתה שואל על תצפיות, ואני מנסה להראות לך שיכול להיות הסבר אלטרנטיבי ומציין שהוא לא בהכרח נכון, אז למה שאכנס לדקויות? ומה בדיוק אני מנסה למכור? תירס?

    אולברס מצביע על פרדוקס אולברס כדי להראות שאפלת שמי הלילה סותרת את ההנחה של יקום סטטי אינסופי ונצחי. הוא אינו מצליח להביא הסבר (ההסבר הנכון: התפשטות היקום). אז האם זה שהוא לא הביא את ההסבר אומר שהפרדוקס אינו קיים?

    אני מנסה להראות שהיחסות אינה מסתדרת עם הקוואנטים (ועוד כמה דברים, מחכים לאבי שישחרר את הממתינה), האם זה אומר שיש לי הסבר אלטרנטיבי לכל תופעה?

    אז אולי אתה, המסביר הלאומי, תסביר לי איך זה מתפרסם בסיינטיפיק מאמר שאומר בדיוק מה שאני אומר? צירוף מקרים? מקרה ממוזל? קסם?

  61. ישראל
    בעיני כן. מכמה סיבות.
    כדור הארץ בתנועה יחסית לקרינת הרקע, אז היינו צריכים לראות תלות בכיוון. כדור הארץ נע בכמה מאות ק”מ לשנייה, שזה לא זניח. לכן, הייתי מצפה לתלות של קליטת המיואונים והשעה.

    הבעיה הגדולה יותר היא שזה לא מסביר כלום. מה זה קרינת הרקע? כמה פוטונים לכל סמ”ק? מה עם האטמוספירה עצמה?
    ולמה זמן יתארך בגלל stress? כי זה מתאים למה שאתה מוכר? 🙂

  62. מה גרוע?

    הנה הבעיה עם המיואונים כפי שאני מבין אותה: אורך החיים הקצר שלהם לא מאפשר להם לעבור את המרחק ממקום היווצרותם עד לארץ לפני שיתפרקו, למרות זאת הם מצליחים להגיע לארץ. כיצד זה קורה ללא התארכות הזמנים/התקצרות האורך. הבנתי נכון?

    והנה ההסבר שנתתי לך: בגלל התאוצה והתנועה כנגד קרינת הרקע, השעון הפנימי של המיואון נמצא בעקה ולכן הוא מואט, בדיוק כפי שתהליכי הרקבון מואטים בקירור ולכן אורך החיים של תפוח במקרר גבוה משל תפוח באזרחות. הסבר לא בהכרח נכון, אבל גרוע?

  63. ישראל
    קיבלתי הסבר מאד גרוע. מיואונים ניקלטים גם בלוס אנג’לס וגם בסידני. איך זה מסתדר?

    גם מה שאומרת תורת היחסות יוצאת בניסוי: התארכות הזמן, התקצרות המרחק, גידול המסה, התעקמות האור, גלי כבידה, קליטת מיואונים מקרינה קוסמית, עידוש כבידתי וכן הלאה.

  64. ניסים

    ״תורת היחסות היא פשוטה, גם הפרטית וגם הכללית. תורת הקוונטים היא מסובכת להחריד. למה להעדיף דווקא את המסובכת?״

    כבר היינו שם פעם, זוכר? 1935, EPR?

    בגלל שזה מה שיוצא בניסוי.

  65. ניסים

    עקה – לא מועקה. פיזיקה – לא פסיכולוגיה.

    מיואון לא רואה כלום, ויש מערכת יחוס מועדפת. ביקשת הסבר – קיבלת.

    אלבנצו

    אז מה אתה אומר, על פי מאמר היחסות המקורי ורק על פיו – קיימת הסתירה עם הקוונטים עליה הצבעתי או לא? זכור שאינך יכול לדבר בקירובים של עיקרון אי הוודאות מכיוון שטרנספורמציות לורנץ הנגזרות מהיחסות אינן מקורבות.

    חוץ מזה שאם אתה רוצה לדבר כמורה אל תלמיד, יהיה עליך לענות על כמה שאלות לפני שתרכוש לך סמכות..

  66. נו, מה יהיה?

    https://www.hayadan.org.il/researchers-discover-the-most-distant-supermassive-black-hole-07121704/comment-page-6/#comment-723358

    בפרט פסקה אחרונה.

    בקורס בחדווא אחד הדברים הראשונים שלומדים (בד”כ הראשון) הוא הגדרת הגבול. בעזרת הגבול מגדירים רציפות, ומשם לנגזרות. אבל אם מנסים ללמוד חדווא רק מהכתבים המקוריים של ניוטון אז מגלים שמושג הגבול בכלל לא קיים שם. את כל ההוכחות ניוטון עשה באופן גיאומטרי והפלא ופלא – רובן שגויות. המסקנות נכונות, הדרך לא ולעיתים קרובות מכילה סתירה פנימית. רק 150 שנה אחרי ניוטון מצאו דרכים להוכיח באופן ריגורוזי את כל העקרונות והמשפטים של החדווא. אז מה תגידו – יש בעיה בחדווא או לא? אם אני מגביל את עצמי אך ורק לכתבים המקוריים (חלקם נמצאים בפרינקיפיה, חלקם פורסמו בנפרד) אני מוצא משפטים עם הוכחות שגויות. מה זה אומר על חדווא?

    אי אפשר ללמוד יחסות פרטית רק ממאמר אחד של איינשטיין כפי שאי אפשר ללמוד אבולוציה רק ממוצא המינים. אין שום צורך לדעת מסלול של פוטון כדי לנסח את תורת היחסות הפרטית. זה שאיינשטיין עשה זאת (כי הוא הרחיב מכניקה קלאסית ובמכניקה קלאסית אין סיבה לחשוב שאנחנו לא יודעים את המסלול של פוטון) זה לא אומר שיש בעיה ביחסות פרטית. זה רק אומר שיש בעיה בתלמיד שחושב שהוא יכול ללמוד את כל התורה ממאמר אחד, ושכמסבירים לו שלא – הוא מתעלם.

  67. ישראל
    אם השעון של המיואון היה איטי יותר (מבחינתו) אז הוא היה רואה את כדור הארץ מתקרב אליו במהירות הרבה יותר גבוהה מ-c.

    שינוי הזמן של שעוני GPS הוא תוצאה של מספר גורמים. אחד מהם, העיקרי הוא העדר הכבידה בגובה של הלווין. אין כאן כל עניין של מהירות. המרכיב השני בשינוי הזמנים הוא המהירות היחסית, וכן אנחנו מודדים שההאטה היא סימטרית, כלומר אין מערכת ייחוס מועדפת.
    זה מש לא מסתדר עם המועקה שאתה מתאר.

  68. בינתיים מיואונים.

    מיואונים נמצאים בעקה – stress עלק – שנובעת מהאצה למהירות גבוהה ותנועה נגד קרינת הרקע. העקה גורמת להאטת השעון הפנימי של המיואון.

    זו גם הסיבה להאטת שעוני כל מערכת מואצת, כגון לוויני GPS.

    זה הסבר אפשרי, אך לא הכרחי. הפיתרון האידיאלי הוא שילוב היחסות כמקרה פרטי בתיאוריה הכוללת אי לוקליות, כמו שתאוריית ניוטון היא מקרה פרטי של היחסות במהירויות נמוכות ותאוריית מקסוול היא צמצום היחסות.

  69. ניסים

    למה אתה עושה לעצמך את זה?

    Hi every body and thank you for coming to my presentation “the problem with relativity”.

    Maybe you are waiting for me to complete the sentence “was Einstein wrong?” This is like saying “was Newton wrong” or “was was Maxwell wrong?” At the end of the presentation you will realize I believe, that if Einstein was wrong at all, it was a very reasonable mistake that he made.

    I also must add that chances are that he was not wrong at all, except maybe on the subject of illocality, and I simply don’t understand the issue good enough. One of the reasons I am making this presentation today is that perhaps one of you will clarify the issues I am about to raise today so I will understand them better. I raised All those subjects in many forums and was in contact with numerous experts, inc physics professors, but none of their answers ended my wonderings. I also invest a huge amount of time – and also money – to try to find an answer to those quarries through research and all kind of experiments.

    I would like to devide the presentation to 4 parts:

    1. the way physics looked until the end of the 19th century.

    2. explaining the reason why Einstein came with special relativity including a brief explanation what relativity is and why we have time dilation acording to relativity.

    3. The problems i see with relativity.

    4. open discussion for the group.

    The subject is vast, and the time is short, so naturally we won’t be able to elaborate too much on each part. never the less, I would still like to tell in chronological order the history of the scientists involved in the road to relativity and hope you’ll enjoy the story and also the physics..

    So let start with part one – A brief history of relativity.

    I’ll start with a description of the way physics looked like before Einstein’s miracle year of 1905.

    Newton’s law of mechanics gave an excellent description of the way physical objects react to forces including all the mathematical formulation. At the end of the 18th century and the beginning of the 19th century, the laws of optic, electricity and magnetism were beginning to emerge. Michael faraday showed that electricity effects magnetism and vice verse. Faraday btw, had never have a mathematical education, he hardly even knew algebra, but never the less he was a great experimenter, who showed the connection between electricity and magnetism.

    The year is 1861 – civil war is raging in America – and in Scotland a physicist named James clerk Maxwell publish a paper named: “on physical line of force”.

    Many of you probably heard of Maxwell, let just say that along with Newton and Einstein, Maxwell is considered one of the greatest physicist of all times.

    Maxwell try to mathematically and physically formulate Michael faraday’s equations of electromagnetism. he called it:

    The Theory of Molecular Vortices applied to Magnetic Phenomena.
    Probably most of you are familiar with the phenomena that if you spread iron dust (or fillings) next to a magnet, you can see the dust ordering itself on the lines of the magnetic field. also, if we move a magnet next to a cooper wire, it creates an electric current in the wire, and if you run a current in this wire, it can move the needle of a compass.

    Maxwell, an expert on the subject of hydrodynamics, built an hydrodynamic model describing and explaining how those phenomena are occurring in the so called ether. according to this model, the ether is somewhat like a sea, and in this sea there are currents, undulations, pressures, vortices, and like in any sea, there are also waves.

    In his model, Maxwell showed that those line of forces that we see when we spread the iron dust next to a magnet, are actually currents that flow from the north pole of the magnet to the south pole, and they create another current in the “sea”, which is perpendicular to the magnetic current and can be view as an electric current. This electric current in its turn creates a 90 degrees magnetic current, which again creates an electric current and so on, till the end of times.

    All of this back and force game of an electric field changing to a magnetic field and vice versa propagates in the sea as a transverse wave, a wave that Maxwell called an Electromagnetic wave.

    Maxwell then went to calculate the velocity of this wave.

    He used Newton’s formula for sound waves in the air for the speed of the wave: the densityρ determine the speed of soundc (pressure waves), according to the Newton-Laplace formula:
    c=\sqrt{\frac{K}{\rho}}.
    and showed that if we substitute K in the formula (elastic modulus) and the Greek letter row (density) with the known coefficients constants for electricity and magnetism, we get the speed of propagation of the wave in this “ether’ medium.

    When he checked to see what the value of this speed, he found for his surprise that this is the speed of light as measured by Armand Fizeau in 1849 (see equation 136 in Maxwell’s ether model):

    Maxwell then concluded that light is also an Electromagnetic wave.
    people told me in the b past that Maxwell already knew that he will get the speed of light and only “drew the the target around the arrow hitting point”, but Maxwell himself denied it.
    4 years later, Maxwell used the same ether model to arrive to his famous differential equations which can bee seen on t shirts students are wearing.

    Maxwell died in 1879 young, he was 48.

    Maxwell model of electromagnetism was praised by the scientists of his time, but there were no experiment which could support his theory. In 1879, the year he died, the streets were mostly dark at night (the light bulb was invented by Edison in this year) and most of the transportation was handled by horses. So you expect people to believe some mysterious waves, unseen and untouched, moving at the speed of light? what another idea those crazy physicist will come with, maybe that people would be able to drive their cars in America and use electromagnetic waves to talk live to their friends in India and see them too? real lunatics..

    Hermann von Helmholtz, the head of the physics department In Berlin, offered the BERLIN PRIZE to the person who will demonstrate experimentally Maxwell’s predictions. The years went by and no one seemed to make good on the mission. Helmholtz was about to give up, but then, his brightest student, Heinrich Rudolf Hertz, succeeded in 1886 to transmit and receive what will be known as radio waves, and also to confirm that they are moving at the speed of light, are polarized and the rest of Maxwell’s predictions.

    Of Hertz’s demonstration of electric waves, Helmholtz told the Physical Society of Berlin: “Gentlemen! I have to communicate to you today the most important physical discovery of the 19th century.

    Hertz died in 1894 very young, he was 36.

    So here we are, almost at the end of the 19th century, and many believe that physics is almost complete and the only thing left is to improve the measurements to better accuracy. Thermodynamics, with its 2 powerful laws, the first and the second, was reduced to a branch of kinetics mechanics. English Newton gave us the laws of mechanics and gravity, Scottish Maxwell gave us the laws of Electromagnetism, and all we need to do now is to enjoy ourselves in the comfortable world those two British Gentlemen arranged for us and drink a cup of tea, like we do in Windsor.
    I have to admit I miss this age of innocence, when classical physics ruled, here was here, there was there, present was not past, and we had a single universe. Can you compare it with the mess we have today, with multiverse, electrons jump from one side of the universe to the other in no time and without even being in the middle, with measurements in the present affecting measurements in the past?

    In my grandfather times, a wave was a wave and particle was a particle.

    There was still an unsolved question: if, as Maxwell theorized and Hertz showed experimentally, Electromagnetic waves propagates in the ether “sea” – then what is the rest frame of this sea? Every hydrodynamic medium has a rest frame. The oceans – the same as earth. the air – earth, or if there is a wind, the wind. The air in an airplane, the airplane.
    I just need to add that the belief in the ether at the end of the 19th century was like the belief in atoms today: almost unquestionable. that’s right, no one saw or felt the ether, but no one saw or felt an atom or electron in our times too.

    So what is the rest frame of the ether?

    in 1887 Michelson and Morley performed an experiment to detect the “ether wind”, and from it to deduce the rest frame of the ether relative to earth. they used an instrument called interferometer, they used it many times in different seasons of the year, but to their dismay they found no rest frame for the ether. this was puzzling to them and most of the scientific community since as we said, almost every body believed in the ether.
    But the “Failure” of the Michelson–Morley experiment – they saw it as a failure, maybe some technical problem – and done it over and over again, along with the problem of black body radiation, led to the most creative and amazing developments in physics – Theory of relativity and Quantum mechanics.

    The null result of the Michelson–Morley experiment in 1887 along with other considerations, led Albert Einstein to publish in 1905 his paper: ON THE ELECTRODYNAMICS OF MOVING BODIES.

    in which he introduced special relativity to the world.

    This concludes part 1 of the presentation, 19th century physics. If any one has some questions or comments, this is the time before we move to the second part.

    Part 2: Relativity in a nut shell.

    I promised Julian not to over complicate the presentation too much, but I feel I must elaborate a little beat on the subject.

    Israel’s first president haim weitzman once sailed with Einstein and Einstein explained to him the subject of General relativity. At the end of the voyage the reporters at the port asked Weizman “so now do you understand relativity?” Weizman replied “no, but I got the impression that Einstein does”.
    Special relativity is actually not that complicated, that’s why I would like to stay on the subject for some time, and if one person would feel he or she understands it better, i did my part. But Julian, if you think we are getting too detailed here, fell free to cut me so we can move on.

    In his theory, Einstein used 2 postulates:
    The laws of physics are the same for all observers in uniform motion relative to one another (principle of relativity).
    The speed of light in a vacuum is the same for all observers, regardless of their relative motion or of the motion of the light source.
    Einstein showed that the problem of the rest frame of the ether that Michelson and Morley failed to find, is solved if we ignore the ether assumption and replace the notion of fixed space and time in Newtonian mechanics with relative parameters while the only fixed thing is c, the speed of light.
    So let see what that this means:

    first postulate:
    The laws of physics are the same for all observers in uniform motion relative to one another (principle of relativity).

    This principle is known since Galileo’s time, 300 years earlier. what it says is that if you have 2 bodies which are moving relative to each other and they are not accelerated, there is no way to tell which one is “really” at rest and which one is “really” moving. as a consequence, there is no experiment that can tell an inertial body if he is at rest or moving.

    Second postulate:
    The speed of light in a vacuum is the same for all observers, regardless of their relative motion or of the motion of the light source.
    To understand this statement, let us think of a long train standing on the railroad. if we shoot a bullet from a gun along the train and the bullet is moving at 1000 m/s, then the movement of the bullet is 1000 m/s relative to the gun which shot it, and the sound of the shot is 330m/s relative to the air, the medium carrying the sound. those are also the speeds of the bullet and the sound relative to the train.
    If the train will start moving after the shot in the direction of the shot, the bullet will now move slower relative to the train because the train is moving relative to the original location of the gun, and also the sound moves slower relative to the train because the train moves relative to the air, the medium carrying the sound.
    If we move at the speed of the bullet, we can catch it with our hand. If we move at speed of sound, we’ll make a supersonic boom. We can move faster than both, the bullet and the sound, and leave them behind us.

    we call the speed of the bullet an emission speed, since it relative to the source that emitted it.
    But what about the speed of the light, the flash created by the shot? is it relative to the gun that emitted it?

    Einstein checked the so called “emission theory”, but realized it cannot be real in the case of light. And as we saw, The null result of the Michelson–Morley experiment in 1887 showed that the speed of light is not relative to the ether medium.
    Einstein also imagined what will happen if he moved at the speed of light. will the light be at rest relative to him? would he be able to reach his hand and grab some light like if you can catch a bullet if you fly over it with the same speed like the bullet?
    Einstein realize the answer must be negative. It does not matter how fast you are moving, the light moves away from you at the same speed c, the speed of light. this is the second postulate of relativity.

    The subject of time and synchronization is a little complicated and I would not touch it unless there is a demand from the group. basically, how do we know that 2 clocks are moving at the same rate if they are in different places? why would they? I must tell you that ever since I started to learn the subject I check from time to time the clock in the other room to make sure it shows the same time like in my room.
    Never the less, Einstein 1st chapter in his relativity paper is dedicated to the subject of simultaneity – he suggest a way to make sure that any number of clocks in an inertial frame are synchronized, i.e. show the same time always regardless of location in the system.

    So lets go back to our train, and let’s say that there are synchronized clocks on the train.

    The train is not moving, so it is an inertial system. for simplicity we have the railway at space, let say that the railway is the galactic railway which was was built by the Vegans and cost the lives of many unlucky Andromedians slaves. it has two parallel lanes and is 100 light years long. a railway, you know, like the ones we have in l.a.

    our train is 300,000 km long, the cars do not block light, and each car is 1 km long. and at the end there is a mirror. so if we turn a flash green light at one side of the train, it will take the light exactly 1 second to go to the other side and to hit the mirror and then to turn back, and another second to reach back to the point of origin.

    Notice that if we have a mirror on the first side too we are getting a clock, since we get a “tick” when a photon hit one side and “tack” on the other side, every second there is tick tack tick tack, just like a clock.
    So if the clocks in the train are synchronized as Einstein suggested, then at time 0 the light will leave one side of the train, will reach the other side at time 1 sec and will reach the first side at time 2 seconds. Every one agree?
    If the train now moves on the railway, we will get the same results for any speed of the train. remember, in its own frame, the train is always stationary and the tracks are moving.
    Now let say that on the parallel track standing an identical train, train 2 with a red light. Naturally same will happen to train 2.

    Now what happens if one train is moving relative to the other, and at time 0 both trains ends coincide.
    At time 0 in both clocks the light leaves the end of the trains, let say the green. We have a high resolution camera and it shows: the clock at train 1: 0, The clock at train 2: 0. Since both trains are inertial systems, it will reach the other side at time 1 second like before, and at time 2 seconds the other hand. remember, each train is a system by itself and the movement of the other has no effect on it. They are both standing, and the other train, or the railway itself, is moving relative to them.
    But when the light reaches the first side of train 2, train 2 had already advanced along train 1, lets say 1000 km, or 1000 train cars. The time in train 2 clock is 2 seconds, but what is the time in the adjacent clock from train 1in car 1000?
    It cannot be 2 seconds because the light reaches the end of train one at time 2 seconds, and it still has 1000 km to go.. that’s why the time in the adjacent clock is less than 2 seconds. If we take a picture – a snap shot – with a high resolution camera of the two clocks together at this point, from either train, the picture will show: the clock from train 2, 2 seconds, the clock from train 1 less than 2 second.
    As we remember, They started both at time 0 in each train, and we even have pictures to prove it.

    This is why we have time dilation in a moving objects in synchronized system. we can continue and show why we have length contraction, but let’s skip it for now.

    Note that time dilation happens also if we do not use light at all – we used it only to demonstrate the point.
    Einstein then went to prove that nothing can move faster than light. I will spare you the proof, and say only this: any speed which is fulfill the first postulate, ie is the same for all observers, regardless of their relative motion or of the motion of its source, is the upper limit for all speeds, does not matter if its the speed of light, the speed of sound or the speed of a hound.

    Later in this year in another paper, Einstein showed that mass and energy are equivalent and related by the now famous formula: E=mc^2. 10 years later in 1915 Einstein expanded his theory to include gravity, in what’s known as General relativity.

    Here we end the first two parts of the presentation, The Physics before Einstein and theory of special relativity in a nut shell. If anyone has comments or questions, this is the time before we move to part 3, CHALLENGES TO RELATIVITY.

    part 3: CHALLENGES TO RELATIVITY.

    1. Even though there are nomathematical inconsistencies within the theory, physically relativity is so strange that many physicists found it very difficult to accept. Theoretical physicists such as Abraham, Lorentz, Poincaré, and Langevin still believed in the existence of an ether.
    For example, Lorenz length contraction is especially difficult to comprehend and has no direct experimental support; http://math.ucr.edu/home/baez/physics/Relativity/SR/experiments.html#Length_Contraction
    Tests of length contraction
    At this time there are no direct tests of length contraction. Imagine that you move toward the the star Alpha centauri (4 light years away) fast enough so that gamma factor:

    is equal to 1000,000, and you take a picture of the star from earth. Since length contraction is only in the direction of your movement and not in the perpendicular coordinates, what will your picture show? — a huge star closer than the moon?

    2. Inconsistencies with Quantum theory. Look at Sky Jason Shields’ presentation, even though I missed June’s lecture.
    From his presentation: Basically you can think of the division between the relativity and quantum systems as “smooth” versus “chunky” or continuously interconnected versus discretely segmented.

    3. If Maxwell’s Ether model is wrong – Maxwell’s model is very complex and delicate – then how was he able to derive the speed of light from this wrong hydrodynamic model (equation 136 above) and his famous equations? Coincidence? Lucky guess? Magic?

    We have in Israel a popular radio fox hero by the name MAR NAHSON, which means Mr guester. He was interviewed in the radio and was asked how did he succeeded to be the only one to guess all the results of the soccer games 4 weeks in a row.

    “Well”, he explained, “this is very simple. What I do is counting the marbles on the floor, multiply by the length of the walls, and divide by the numbers of the neighbors. By default, it must come the correct answer.
    And he has another system for the summer: he takes the all family to the beach when there are lot of waves at the sea and let them in, and then watch: if one comes out, he writes 1. if 2, 2. and if no one comes out – x.
    So maybe Maxwell’s model is like Mr Guester, and he was simply lucky?

    4. First postulate of relativity:
    The laws of physics are the same for all observers in uniform motion relative to one another (principle of relativity). This is inconsistent with big bang theory and cosmic microwave background radiation.
    Look at that in the twin paradox when the young twin meets his older brother, the universe is colder at the time they meet according to Friedman’s equation:
    http://hyperphysics.phy-astr.gsu.edu/hbase/Astro/expand.html#c3
    Thus, during his travel which takes shorter time than his brother, he observe faster drop in the temperature than his brother, even though they are both in an inertial frames.
    Note that acceleration is not a solution in this case: If the older twin was moving relative to the cmbr, he was the one to observe the rapid temperature drop, and the young one would stay warm and cozy.

    5. In the EPR paradox (1935):
    https://en.wikipedia.org/wiki/EPR_paradox
    Einstein, Podolsky and Rosen describe a thought experiment to show why Quantum Mechanics theory is incomplete. According to the article in wikipedia:
    Here is the paradox summed up:
    It is one thing to say that physical measurement of the first particle’s momentum affects uncertainty in its own position, but to say that measuring the first particle’s momentum affects the uncertainty in the position of the other is another thing altogether. Einstein, Podolsky and Rosen asked how can the second particle “know” to have precisely defined momentum but uncertain position? Since this implies that one particle is communicating with the other instantaneously across space, i.e., faster than light, this is the “paradox”.

    A variation of the experiment was performed by Alain Aspect in the 1980’s, and was found to be consistent with Quantum Mechanics and John Stewart Bell’s theorem, and against Einstein claim.
    Later attempts to save relativity from Aspect’s experiment were based on the claim that what relativity really forbids is the transfer of information, And it’s impossible to send information with a Bell experiment (ASPECT is a bell experiment) According to no-communication theorem:
    https://en.wikipedia.org/wiki/No-communication_theorem
    Well, the fact that it’s impossible to send information by entanglement was known already in 1935. So why do we have the notion in EPR’s wikipedia paper:
    “Since this implies that one particle is communicating with the other instantaneously across space”?
    Would Einstein embarrass himself in face of the all world in his most important paper since General relativity? Or maybe the maestro saw that transfer of information faster than light and thus to the past, does occur in contradiction to relativity in the case of entanglement?

    And if Einstein say that there is contradiction between ilocallity in quantum entanglement and relativity – can we tell Einstein he is wrong? Do we understand relativity better than Einstein?

    6. But what about all the unquestionable successes of relativity? GPS? Muons? and above all – E=mc^2?

    I can answer that in a question: What about all the unquestionable successes of Newton’s theory? almost every technology we know is based on it and yet, Newton’s theory is just a special case of relativity in low speeds.
    and E=mc^2? this is pure Einstein isn’t it?

    From Wikipedia:

    Olinto De Pretto (26 April 1857 – 16 March 1921) was an Italian industrialist and geologist from Schio, Vicenza. There are claims[1] that De Pretto may have been the first person to derive the energy–mass-equivalence E=mc^{2}, generally attributed to Albert Einstein. He suggested that radioactive decay of uranium and thorium was an example of mass transforming into energy.

    From 1899 to 1903 De Pretto began to study the emerging field of nuclear physics and its relationship to astronomy. He focused on the theory of aether, a hypothetical substance that at that time was believed to fill all space.

    De Pretto used the expression mv^2 for the “vis viva” and the energy store within matter, where he identified v with the speed of light. His formula precedes by two years, and is in agreement with Albert Einstein’s later formula E=mc^{2} for mass–energy equivalence, which was derived by Einstein as a consequence of special relativity.

    I raised those objections to relativity for many years, and not too many agree with me. But less than a year ago I found an article in SCIENTIFIC AMERICAN’s 2009 paper: A QUANTUM THREAT TO SPECIAL RELATIVITY, which says exactly what i was saying for many years:

    “The status of special relativity, just more than a century after it was presented to the world, is suddenly a radically open and rapidly developing question. This situation has come about because physicists and philosophers have finally followed through on the loose ends of Einstein’s long neglected argument with quantum mechanics”.

  70. ישראל
    זרוק לרגע את תורת הקוונטים. התעלם לחלוטין ממנה.
    האם עכשו יש סיבה לא לקבל את הנחות היסוד של תורת היחסות, ואת המסקנות שלה?

    בתור תרגיל, נסה להסביר את הסיפור של מיואונים בלי קיווץ אורך.

    תורת היחסות היא פשוטה, גם הפרטית וגם הכללית. תורת הקוונטים היא מסובכת להחריד. למה להעדיף דווקא את המסובכת?

  71. בוא נראה מה אני מנסה לעשות פה.

    איני מנסה להסביר שזירה או העברת אינפורמציה – למרות שיש לי איזה רעיון – כבר סיכמנו שכל עוד מקבלים את יכולת ההשפעה מרחוק באפס זמן אין צורך להשתמש במילה אינפורמציה.

    מה שאני מנסה לעשות במסגרת התגובות בכתבה זו הוא להראות מה הבסיס לסתירה כביכול בין קוואנטים ליחסות.

    הבאתי את הכתבה של סיינטיפיק אמריקן. אפשר להסכים לנאמר בה או לא, אך התיזה שם היא שאי לוקליות בשזירה מאיימת על תורת היחסות הפרטית, בגלל השפעה מרחוק מהר מהאור.

    הטענה שלי אינה כוללת שזירה או השפעה מהירה מהאור, אלא נוגעת לבעיה יסודית בהנחת היסוד של היחסות: שהפוטון מצוי בנקודה מסויימת בלבד ברגע נתון. מסתירה זו לידוע לנו ממכניקת הקוואנטים נגרמות לדעתי כל הבעיות האחרות במידה והן קיימות, כפי שכתוב במאמר.

    הבאתי גם תיאור של פוטון שיכול לפתור את מוזרותה של היחסות וגם לשפוך אור על דברים מוזרים מאוד בקוונטים כגון השפעה על העבר.

    התיאור הזה אינו שרירותי סתם – הוא הסקה הגיונית הנובעת ממודל האתר של מקסוול במערכת פתוחה ואינסופית, וממה שנראה לי כטעות יסודית בנקודת המוצא של ניסוי מייקלסון – מורלי.

  72. ישראל
    איך זה מסביר שזירה? איך זוג פוטונים מעבירים ביניהם מידע? הם שולחים ביניהם עוד פוטונים? אם כך אז גלאי ביניהם היה מגלה את הפוטונים האלה.

    כל המהירויות כולל את כל הכיוונים? אם לא, איך אתה מסביר עקיפה?

  73. ללא ספק. אתה יכול לפלוט פוטון מנקודה A וכעבור זמן מתאים לקלוט אותו ב-B.

    אבל אם הוא מרוח במרחב לפני כן, ועוד בהסתברות שווה, אז יש שתי אפשרוריות:

    1. מציאות הפוטון מעבר לB אינה משהו פיזי – זו רק פונקציית הגל או הפילוסופיה או הזברבירולוגיה, אך לא הפוטון עצמו.

    (קצת מוזר אם נחשוב על כך שריבוע פונקציית הגל היא ההסתברות למצוא את החלקיק בנקודה מסויימת).

    2. החלקיק עצמו נמצא גם מעבר לB.

    במקרה 1, אז אין כל ממשות לפוטון שנע מהר מc, איינשטיין צודק וקוונטים חרטה. אך להבנתי, לא זה המצב.

    במקרה 2, אם הפוטון עצמו נמצא מעבר לB בזמן פחות מct, אז אין מנוס מהמסקנה שהוא נע מהר מc ולמעשה בכל מהירות שהיא.

    האם אני יודע מה הפיתרון? שלילי. אך נראה לי שאף אחד כאן לא יודע. דוקטור לפיזיקה הסכים איתי שהחלקיק נמצא גם מעבר לB לפני המדידה.

    אני הצעתי את מה שנראה לי הגיוני ומתאים לתנאי הבעיה: הפוטון נע בכל המהירויות כמו גומיה שנמתחת, אך מכשירי המדידה שלנו יכולים לקלוט רק את החלק שנע יחסית למכשיר המדידה בc. הסברתי גם למה – דוגמת הפגזים שעוברים דרך מנהרה בכדה״א ורק אילו שנעים במהירות מסויימת יכולים להקלט. בדוגמה הזו החלק שנע מהר מc פשוט לא יכול להיקלט במכשיר המדידה, כמו שפגז שנע מעבר למהירות המילוט לא ילכד במנהרה.

    לילה טוב ושנה טובה.

  74. ישראל
    לא אמרתי את זה. אני אומר שכל עוד לא מצאנו אותו במרחק מיליארד שנות אור, זה לא מעניין אותנו.

    הוא יוצא מ-A ומגיע ל-B. אם תחפש אותו בקו הישר בין A ל- B תמצא אותו. אם תחפש אותו באנדרומדה, לא תמצא אותו.
    זו ההסתכלות כחלקיק. אם אתה רוצה להסתכל עליו כגל מרוח במרחב, זה משהו אחר.

    האם אתה מקבל שאני יכול לפלוט פוטון מנקודה A וכעבור זמן מתאים לקלוט אותו ב-B?

  75. פילוסופיה עלק..

    אז אתה טוען שרק פילוסופית הפוטון יכול להיות במרחק מיליארד שנות אור אחרי 9 שניות ולא פיזית? שאין לו כל קיום פיזי במרחק מיליארד שנות אור?

    אז למה התיאור הזה מופיע בספר פיזיקה ולא פילוסופיה?

  76. ישראל
    האם עץ שנופל ביער בלי אף אחד בסביבה, משמיע רעש?
    כשאף אחד לא מסתכל, הפוטון יכול להיות בכל מקום? כן – בתנאי שאף אחד לא הסתכל עליו. זה לא המקרה של הפוטון של איינשטיין. אנחנו נגלה אותו במרחק ct, תמיד.
    תחשוב על ניסוי שני החריצים. אנחנו יודעים מתי חלקיק יוצא ומתי הוא מגיע. בדרך, יש לחלקיק תכונות של גל – הגל עובר תחילה דרך חריצים צרים ולכן נוצרת עקיפה. בקצוות – הוא חלקיק לכל דבר. בניסוי של איינשטיין, התופעה תתבטא בפיזור מסויים של הפוטונים בקולט. גם בלייזר הכי טוב יש פיזור של הקרן.

  77. ישראל
    האם עץ שנופל ביער בלי אף אחד בסביבה, משמיע רעש?
    כשאף אחד לא מסתכל, הפוטון יכול להיות בכל מקום?

  78. אז אתה מאמין כמו איינשטיין שאם פוטון יצא ברגע 0 מנקודה א׳ אז אחרי זמן t הוא יהיה מרוחק ממנה ct?

    אז מה כל הסיפור הזה ששניה קודם הוא היה רחוק ממנה מיליארד שנות אור?

    ומה הקשר למדידה? זהו ניסוי מחשבתי שבו לא מוזכרת מדידה אלא חישוב מיקומו של הפוטון באמצעות המחשבה.

    או שאולי המחשבה גורמת למרכוז הפוטון בנקודה מסויימת?

  79. ישראל
    אני כן מסכים עם זה. אני לא מבין מה מפריע לך בזה.

    זה לא סותר את תורת הקוונטים. יותר מזה – יש ניסויי מעבדה עם פוטונים בודדים, בהם יודעים איפה פוטונים בודדים נמצאים, ומתי.

    אנחנו יודעים לראות הרבה חלקיקים אלמנטרים, בעזרת תאי בועות למשל. מונה גייגר גם מגלה חלקיקים בודדים.

    נכון – עקרון אי הוודאות קובע שלא ניתן לדעת בדיוק לא מוגבל זוגות מסויימות של נתונים, כמו תנע ומיקום. אבל, הדיוק הוא בהחלט מספיק גבוה בשביל לאשש את תורת היחסות הפרטית.

    הםםם … אולי בגלל עקרון אי הוודאות, כשאנחנו רואים פוטון בודד אז אנחנו לא יודעים מה הצבע שלו? (זה קשקוש כמובן… יש סיבה אחרת לכך).

  80. אני לא חושב שהרעיון שלי טוב משלהם או אפילו טוב. רק ניסוי יקבע.

    ותיאורית ניוטון לא מספקת דיוק. נסיוני מדהים? ותאוריית תלמי?

    לא הבנתי למה אינך מסכים שהיחסות בנויה על הרעיון שפוטון נמצא ברגע מסויים במיקום מסויים, מה שמתנגש עם הקוונטים.

  81. ישראל
    יש מלא סתירות ביניהם (חורים שחורים, המפץ הגדול, דטרמיניזם מול אקראיות, א-לוקליות וכן הלאה). אבל – כל אחד בתחומו מספק דיוק ניסיוני מדהים.

    יש רעינות לשלב ביניהם, רעיונות של אנשים שמבינים בשניהם. מה גורם לך לחשוב שהרעיון שלך טוב משלהם?

  82. נו, שוין.

    אבל זה פחות או יותר מה שנאמר לי על הסתירה כביכול שבין הקוואנטים ליחסות, ואז פתאום יש את המאמר בסיינטיפיק אמריקן שאומר בדיוק מה שאני..

    זה לא אומר שהם ואני צודקים, אבל זה בטוח אומר שיש מי שמבין אותי..

    אז אולי בעוד כמה שנים יהיה מאמר על מה שאני אומר עכשיו.

  83. ניס

    גם בעין. אז מה? אתה לא רואה שמדובר בניסוי מחשבתי?

    טוב, אם לא הצלחתי להעביר לך את הפואנטה עד עכשיו, נמתין לתשובה מהבלוגים בהם הצגתי את השאלה.

  84. ישראל
    טכנית – אפשר לשדר פוטון בודד ולקלוט אותו בגלאי. זה נעשה בניסויים במעבדה.

    אני ממש לא מבין מה מפריע לך.

  85. ניסים

    אתה יכול לקרוא ישירות במאמר היחסות של איינשטיין מ1905, ללא פרשנות.

    Let a ray of light depart from A at the time ta, let it be reflected at B at the time tb, and reach A again at the time t’a

    http://www.fourmilab.ch/etexts/einstein/specrel/www/

    הניסוי הוא ניסוי מחשבתי, ומה שהוא אומר זה: תן לפוטון לצאת מנקודה א׳, להגיע לב׳ ולהיות מוחזר לא׳.

    אח״כ הוא ממשיך את פיתוח היחסות בדרך המוכרת, אך בשום מקום לא מדובר על מדידה, רק על ניסוי מחשבתי והסקת מסקנות לוגיות ממנו.

    אבל הלוגיקה הזו מבוססת על מה שהיה ברור מאיליו ב1905, דהיינו שלפוטון מיקום מובהק בזמן נתון ברגע נתון והוא מרוחק ct מהמקור.

    ואם הנחת היסוד הזו אינה נכונה, אז מה זה אומר על היחסות?

    כפי שהנחת היסוד של אי העברת האינפורמציה בשזירה היא ש״כל הנתונים כבר נמצאים במערכת״.

    אך אם הנחת יסוד זו מופרת, כמו בניסוי אספקט שבו נוסף גורם נוסף למערכת, דהיינו מצב המקטבים שאינו כלול בנתונים הראשוניים, אז מה זה אומר על העברת האינפורמציה?

    קצת טריוויה: הניסוי נקרא על שם אספקט למרות שניסויים כמעט זהים נערכו עשור קודם. הגדולה של אספקט היא שהוא הצליח לשנות את מצב המקטבים אחרי שהפוטונים עזבו כבר את המקור, וכך למנוע את החשש שהמקטבים ״ירקחו קונספירציה״ (כך במקור!) ובכך הנתונים יהיו כלולים כבר במערכת.

  86. “שורש המחלוקת”, ישראל שפירא, הוא – בפרשנות. זאת הפוליטיקה במדע שיוצרת מחלוקת. לא הניסויים.

  87. ישראל
    לא, אני לא מסכים. כל הרעיון בהחלט מבוסס על מדידה. התארכות הזמן, התקצרות האורך ושהגידול במסה הם אך ורק תוצאות מדידה.

    אם אתה תשב בחללית שתאיץ ב 1g במשך שנים, שום דבר לא ישתנה בשבילך. אתה לא מסכים לזה? אם מישהו במערכת אחרת ימדוד, הוא זה שיקבל תוצאות יחסותיות.

    אני לא מבין את הבעיה כאן. הניסוי של איינשטיין מבוצע יום יום במדידת מיואונים, למשל.

    טכנית, פוטון הוא חלקיק קל … אנחנו רואים פוטון בודד בעין 🙂

  88. ניסים

    ו?

    כל הרעיון של התארכות הזמנים (לפחות על פי המאמר של איינשטיין), בנוי על זה שהפוטון נמצא במקום מסויים ברגע מסויים, לא קשור למדידה. מסכים?

    אז האם אתה טוען שזה נכון? פוטון יכול להיות ברגע נתון במיקום מסויים? אם כך מה הוא עשה במרחק של מיליארד שנות אור שניה קודם?

    ומה קשור מה שידע איינשטיין ב1905? למה אם הוא לא ידע אז זה לא התקיים?

    ולא צריך להזכיר שאיינשטיין עצמו נלחם בכל כוחו לתיאור הזה של הפוטון, והפסיד.

  89. ישראל
    יצרת פוטון בפנס שלך. דאגת שינוע בכיוון מסויים. אתה לא יכול לקבוע במדוייק את תנאי היווצרות הפוטון, לכן יהיה לו פיזור מסויים.
    תחשוב על קרן לייזר – ככל שהאלומה צרה יותר, הפיזור גדול יותר (לכן כשמודדים את המרחק לירח שולחים את הפולס דרך טלסקופ, שמרחיב את האונה).
    דוגמה אחרת – מכ”מ. כדי לשפר יכולת אבחנה זוויתית מגדילים את האנטנה. ההגדלה מרחיבה את האונה, אבל מקטינה את הפיזור.

    אלבנצו
    במקרה הזה (שליחת פוטון מ-A לכיוון B), קשה לי להבין שאין מידע על המיקום בדרך. אם אני שם דיסקה עם חור (שניתן לחסום) באמצע הדרך, אז רק אם החור פתוח בזמן הנכון נראה את הפוטון ב-B. אם זה יהיה חלקיק טעון, גם אוכל לוודא שהחלקיק עבר דרך החור.

  90. נסה עוד הפעם.

    האם לפוטון מיקום מובהק לפני המדידה?

    האם היחסות לא בנויה על. כך שיש לו מיקום מובהק?

    קראת את המאמר, לא? בניגוד למה שנאמר פה, שם המאמר הוא איום קוונטי על תורת היחסות, והאמירות שם ברורות וחד משמעיות. אני רק מנסה להראות איפה שורש המחלוקת, וזאת בלי שזירה ומהירות האור.

  91. אלבנצו

    פוטונים, אלקטרונים, מהירויות וכוחות. פיזיקה.

    ולא אתה, לא למדת, לא הבנת ושיקרת. פסיכולוגיה.

    מה גם שכאחד שכותב אין שום הבדל מלבד, ואחרי זה מתברר שהמלבד הזה הוא העיקר, הייתי חושב פעמיים לפני שהייתי מרצה למישהו על הבנת הנקרא.

    ניסים, המסביר הלאומי.

    אז איך זה מסתדר?

  92. ניסים,

    בהצלחה. אני מנחש שאתה הצלחת לעקוב והבנת את דברי. אם תרצה לנסות מזלך, אני מחזק ידך.

    ישראל,

    התשובה לשאלתך מצויה בדברים שכתבתי. אם אתה אכן רוצה להבין, כל מה שאתה צריך זה להשקיע כמה דקות בלחזור אחורה – אני מבטיח לך שהתשובה נמצאת שם. אם אתה לא מתכוון לטרוח לקרוא את הדברים שהשקעתי די הרבה זמן בכתיבתם ודי הרבה אנרגיה בנסיון לעזור לך – אז אני לא רואה איך אתה יכול לטעון בכנות שאתה רוצה להבין.

  93. ישראל
    קבעת את מיקומו וכיוונו.

    לא קבעת את קיטובו – לכן תגלה שב-B יש הסתברות של 50% שהפוטון מקוטב אנכית או אופקית, תלוי איך תמקם את המקטב.
    אם תשים מקטב אנכי ב-A, עכשיו תגלה שאם הפוטון מגיע ל-B אז קיטובו תמיד אנכי.
    והנה קסם – אם תשים מקטב ב-45 מעלות בדרך, אזי תגלה ששוב יש הסתברות שהוא מקוטב אנכית, או אופקית (שוב, תלוי איך תמקם את המקטב ב-B).

  94. אלבנציון

    תודה על סדרת החינוך, אך אם כפי שאתה כותב אינך רוצה לריב, השתדל לדבר רק על פיזיקה ואל תעבור לפסים אישיים.

    ואם איינשטיין לא ידע על מכניקת הקוואנטים זה אומר שהיא לא הייתה קיימת? הרי ההנחה הבסיסית של היחסות היא פוטון הנמצא במקום מוגדר בזמן מוגדר, ושנע במהירות אחת בלבד, c.

    אז אם הנחה זו אינה נכונה, מה זה אומר על היחסות?

  95. ישראל
    קיבלת תשובות:
    1) כן, זה נכון גם לגבי פוטונים.
    2) ברגע שאתה מתאר פוטון, בעצם ביצעת עליו מדידה, ולכן אינו פרוס במרחב כמו ב 1).

    אלבנצו
    מה שאני לא מבין, זה איך זה מסתדר עם ניסוי שני החריצים – הרי שם אנחנו יורים את החלקיקים כמו שישראל מתאר, ובכל זאת אנחנו עדים לכך שהחלקיקים מרוחים במרחב. אני מניח שהההסבר הוא הידע החלקי – אני יודע מתי החלקיק יצא, אבל אני לא יודע לאיזה כיוון. לדייק – לא עצם הידיעה שלי חשובה, אלא עצם המדידה.

    שמוליק
    מעניין הרעיון של פנרוז. זה מסביר למה העולם הרגיל נראה לנו … רגיל.

  96. ישראל,

    אתה פשוט לא קורא מה שאני כותב. למה שאטרח יותר?

    ב-1905, לא הייתה מכניקת קוונטים, ואיינשטיין כתב את תורת היחסות הפרטית כהרחבה למכניקה קלאסית. מאז החכמנו והתורה נוסחה מחדש באופן שאפשר ליישמה גם על מכניקה קוונטית. להתעקש להסתכל על מאמר מ-1905 כדי להבין יחסות פרטית זה כמו לנסות ללמוד אבולוציה על ידי התמקדות אך ורק ב”מוצא המינים”, או מדעי המחשב רק מטיורינג ועבודתו בשנות ה-40.

    כנראה אתה פשוט לא רוצה להבין. בצלחה בהמשך.

  97. לא אמרתי שאני לא יודע את זה, רק רציתי לוודא..

    אז אני חוזר לשאלה המקורית שפתחה את כל הווג׳רס:

    ישראל שפירא
    26 בדצמבר 2017 בשעה 12:43
    מישהו מתנדב לעזור?

    עיקרון אי הודאות קובע שלחלקיק קוואנטי אין תכונות מובהקות לפני המדידה. האם זה נכון גם לגבי פוטונים? במידה וזה כך, אז כיצד זה מסתדר עם תורת היחסות שבמאמר המקורי שלה אומר איינשטיין:

    Let a ray of light depart from A at the time ta, let it be reflected at B at the time tb, and reach A again at the time t’a

    http://www.fourmilab.ch/etexts/einstein/specrel/www/

    הרי כאן מדובר על כך שקרן אור (יכול להיות גם פוטון אחד) נמצא במקום מוגדר – נקודה A – בזמן מוגדר – t’a – והתנע שלו מוגדר – חלוקת קבוע פלאנק באורך גל הפוטון.

    אז איך זה מסתדר?

  98. לרוג׳ר פנרוז יש רעיון מעניין הקושר בין מכניקת הקוונטים לבין היחסות הכללית. הוא אומר שחלקיק יכול להישאר בסופרפוזיציה כל עוד עיקום הזמן מרחב לא עובר סף מסוים. אם מתבצעת חציית סף (הוא מניח שהסף הוא בערך מסת פלאנק אחת) תתרחש קריסה. להבנתי הרעיון שלו בא להסביר את המעבר ממערכת קוונטית למערכת קלאסית ולא את הקריסה המתרחשת בניסוי שני החריצים. הוא אף מציע ניסוי שיבדוק את הרעיון שלו.
    https://en.m.wikipedia.org/wiki/Penrose_interpretation

    אלבנטזו, אתה מכיר ניסויים נוספים שמטרתם לנסות לבדוק את גבולות מכניקת הקוונטים?

  99. אלבנצו
    המדידות לא אומרות שאדם יודע מה המיקום, אלא כל מדידה “קלאסית”, נכון?

    דוגמה: אם אני מבצע את ניסוי שני החריצים עם אלקטרוניים במעבדה סגורה, ומקליט את מיקום האלקטרונים באמצעות גלאי, אז לא תיווצר תמונת התאבכות. המצב הזה מתקיים, גם אם הגלאים לא מקליטים.

    מה שלא ברור לי: סרט הצילום הוא גם סוג גלאי… האם זה דומה למקטבים? יש ניסוי פשוט עם 3 מקטבים, שרואים שמדידה בציר אחד “מוחק” את המידע על ציר מאונך. זה ניסויי די מדהים 🙂

  100. כמובן שכשאני כותב “פוטון יכול לנוע מהר מהאור” הכוונה שלי היא “פוטון יכול לנוע מהר מהקבוע c”. הפוטון הוא האור ולכן לפי הגדרתו הוא תמיד נע במהירות האור :). כמובן שהכוונה כאן היא לתנועה מהירה יותר ממהירות האור בפיזיקה קלאסית, הקבוע המסומן ב-c. כפי שאמרתי, גם במכניקת הקוונטים האור נע במהירות זו אבל רק בין שתי נקודות מדידה. כאשר הוא לא נמדד, אינו נע במהירות זו בהכרח.

  101. כמו שכתבתי בתגובות הקודמות (אתה יכול לעבור עליהן שוב ולראות בעצמך), פוטון קינו מחוייב לנוע במהירות האור. ההתחייבות היחידה היא בנוגע למדידות, אבל כל עוד הוא לא נמדד הוא יכול לנוע מהר או לאט מהאור. זה נכון גם לגבי חלקיקים אחרים – אלקטרון יכול לנוע מהר מהאור כאשר לא מודדים אותו. החלקיקים יכולים לנוע גם במסלולים עקומים למרות שלא פועל עליהם שום כוח. מכניקת הקוונטים מבדילה בין תכונות החלקיק לבין התמונה שאנו רואים. ההבדל הזה הוא אדיר בחשיבותו ואחראי על מרבית התופעות הקוונטיות. רוב הדברים שאנחנו רגילים אליהם ממכניקה קלאסית מתקיימים רק בתמונה שמתקבלת ממדידה, ולא באופן עקרוני.

    אם למדת לפני הרבה שנים ואתה לא זוכר אפילו את העקרונות או המונחים הבסיסיים, איך אתה מצפה להבין? ממליץ לך לעשות ריענון יסודי מאוד. הדברים שאני מסביר לך פה הם יסודות בלבד ובלעדיהם לא פלא שאין לך קצה חוט לתפוס בו כאשר אתה שואל שאלות מתקדמות יותר בנוגע לשזירה, למשל.

  102. למדתי, אבל זה היה לפני המון שנים..

    ״אבל בשניה התשיעית (בה לא מדדת אותו) הוא יכול היה להיות במרחק של מיליארד שנות אור״.

    אז אם הפוטון הצליח לעבור מיליארד שנות אור בתשע שניות – זה לא אומר שהוא עבר במקצת את מהירות האור?

  103. ישראל,

    אם אתה רוצה לנסות להבין את מכניקת הקוונטים ולדבר עליה, אתה חייב להתחיל ליישם את הרעיונות הבסיסיים שלה. בפיזיקה קלאסית אין הבדל בין “החלקיק נמצא פה” לבין “כאשר ביצעתי מדידה מצאתי את החלקיק פה”. בקוונטים יש הבדל תהומי.

    כפי שכבר כתבתי כמה וכמה פעמים – אם אתה מודד בנקודה מסוימת את מיקום הפוטון (או מייצר פוטון אשר המיקום שלו פחות או יותר מוגדר היטב, עד אי-וודאות קטנה) ומודד את מיקומו אחרי 10 שניות, ההפרש בין המיקומים יהיה 10 שניות אור. כתבתי את זה אופן מפורש כבר המון פעמים. אבל כל עוד אתה לא מודד את הפוטון לא מובטח לך איפה הוא. כלומר, אחרי 10 שניות מדדת אותו וראית שהוא במרחק 10 שניות אור, אבל בשניה התשיעית (בה לא מדדת אותו) הוא יכול היה להיות במרחק של מיליארד שנות אור. בפיזיקה קלאסית זה לא אפשרי, בקוונטים כן. וההבדל הזה הוא ענק, כי הפיזיקה נקבעת לא רק לפי מה שאנחנו מודדים אלא גם מה שקורה מאחורי הקלעים כאשר איננו מודדים (דוגמאות לכך הם ניסוי שני הסדקים, אינטרפרומטר מאך-זנדר וכו’).

    לפוטון אין מסלול במכניקה קוונטית. אתה יודע איפה הוא ברגע המדידה הראשונה, ואם תמדוד אותו שוב תדע איפה הוא ברגע המדידה השניה. ההפרש ביניהם יהיה ct, אבל זה רק חלק קטנטן מהפיזיקה של הבעיה. אם אתה רוצה להבין את הנושאים האלה אתה חייב חייב חייב ללמוד מבוא לקוונטים.

  104. בפיזיקה קלסית, אם רובה יורה כדור במהירות 300 מטר בשניה, אז אחרי 10 שניות הוא ימצא במרחק 3000 מטר מהרובה. אין אפשרות למצוא את הכדור במרחק 2900 או 3100 מטר.

    אז אני רוצה לוודא: אם מקור לייזר פולט פוטון בודד ברגע 0, ואנו מחלקים את הישר בכיוון תנועת הפוטון לאינטרוולים של 300,000 ק״מ, אז אחרי 10 שניות הוא ימצא במרחק 10 שניות אור מהלייזר? אין לו הסתברות להיות בין אינטרוול 7 ו8 או בין 13 ל14? ובמידה וההסתברות קיימת, האם היא זניחה?

  105. ניסים,

    עכשיו אני חושב שאולי בכלל שאלת משהו אחר. אולי התכוונת שאתה מודד תנע של חלקיק (שאינך יודע מה פונ’ הגל שלו – אתה רק נסיונאי שמוצא אלקטרון במעבדה ומודד את התנע שלו). כמובן שלתוצאת המדידה שלך תהיה סטיית תקן, שמייצגת את אי-הוודאות במיקום האלקטרון (נניח כרגע שמכשירי המדידה שלך מדויקים עד אינסוף ולא מכניסים עוד שגיאה למערכת). האם אתה שואל אם במקרה כזה סטיית תקן גדולה בתנע (או במילים אחרות, חוסר אינפורמציה בתנע) תעיד על יתרת אינפורמציה במיקום?

    כן ולא. כן, אם יש לך מלא עותקים של אותו חלקיק ועל חצי אתה מבצע מדידות תנע, ועל החצי השני מדידות מיקום, אכן הרבה מידע בצד אחד יתרגם למעט מידע בצד שני. כלומר, ברוב המקרים אם אתה יכול לתאר את התנע בעזרת מעט מאוד ביטים של חצי מהחלקיקים תצטרך המון ביטים לתאר את המיקום של החצי השני (יש כמובן מצבים שיש להם אי-וודאות גדולה גם בתנע וגם במיקום, אבל בוא נשכח מהם כרגע).

    לא, כי זו כמובן רק אמירה סטטיסטית על אנסמבל של חלקיקים. ברגע שמדדת את התנע של חלקיק בודד, שינית את המצב שלו ובפרט הזזת אותו (שינית את המיקום שלו). פשוט אי אפשר למדוד בו זמנית תנע ומיקום, ולכן השאלה לא מוגדרת היטב (היא תלויה לגמרי בסדר המדידות שלך – קודם תנע ואז מיקום, או קודם מיקום ואז תנע).

  106. ישראל,

    לפוטונים יש מיקום. מכיוון שפוטונים הם תמיד יחסותיים, צריך לדבר עליהם במסגרת מכניקה קוונטית יחסותית – או במילים אחרים, תורת שדות. יש דרכים לטפל בהם גם במסגרת מכניקה קוונטית “רגילה”, אבל זה סתם סיבוך של הדברים. במסגרת תורת השדות מיקום הוא לא אופרטור אלא תווית של השדה – כל הפורמליזם המתמטי נהיה הרבה יותר מסובך. אז השאלה “האם לפוטון יש אופרטור מיקום” היא מטעה מאוד, בגלל שהתורה במסגרתה אנו מתארים פוטונים לא כוללת בכלל אופרטורי מיקום. אבל כן, אם אתה מתעקש להצמד לפורמליזם הקלאסי של מכניקת הקוונטי, אפשר להגדיר אופרטור מיקום עבור פוטון (אך הוא צריך לערבב בין מיקום וזמן בגלל הקו-ווריאנטיות היחסותית, כלומר בגלל שזמן ומרחב מעורבבים יחסותית). לא ברור לי למה אתה חותר בשאלה הזו, אולי אתה רוצה הבהרה כי קראת משהו על כך? בכל מקרה זו שאלה טכנית קצת מסובכת שאיני רואה את הרלוונטיות שלה. לפוטונים יש מיקום שאפשר למדוד (אך אי אפשר למקם אותם לחלוטין כפי שהוסבר מקודם). הלייזר שלך לא מייצר פוטונים בעלי תנע מוגדר אלא פוטונים בחבילות גלים צרות אשר ממורכזות בצורה חזקה מסביב לתדירות מסוימת, אך הן בכל זאת חבילות גלים – כלומר סופרפוזיציה של תנעים, וזה מספיק כדי לאפשר לנו “למקם” אותם ולתאר אותם בתור אלומה. בגלל שאלומת הגלים צרה, אז כאשר אתה מודד את התדירות שלה האור אתה מקבל בהסתברות מאוד גבוהה את אותה תדירות לכל הפוטונים ולכן מדידה מראה לך שהאור הוא רק בתדר אחד, כלומר מאוד קוהרנטי. אבל כבר דיברנו על ההבדל התהומי בין מה שמראה מדידה לבין מה שקורה במציאות – במכניקת הקוונטים תוצאת המדידה מראה לנו רק קצה אחד של הפיזיקה, והרבה מאוד דברים מתרחשים דווקא מאחורי הקלעים. כמו שניסים הזכיר, זה כל הסוד מאחורי התאבכות של חלקיקים או ניסוי שני הסדקים אשר נהרס לגמרי ברגע שמודדים אותו.

    ניסים,

    צריך להזהר כאן מאוד. בכל מקרה אם יש לנו את פונ’ הגל של חלקיק (או את המצב הקוונטי שלו, זה הרי בדיוק אותו דבר) אנו יודעים כל מה שיש לדעת עליו. המצב (או הפונ’) מתארים את החלקיק באופן מלא. כשאנו אומרים שיש אי-וודאות גדולה במיקום שלו, זה לא אומר שחסר לנו מידע. זה אומר שהחלקיק עצמו מצוי בהתפלגות מיקומים כזו שקשה מאוד לנחש איפה ימדד. אתה בכל מקרה יודע בדיוק מה היא ההתפלגות.

    באמת יש כאן קשר לתורת האינפורמציה – התפלגויות אשר יש להן אי-וודאות גדולה הן התפלגויות עם אנטרופיה גבוהה במובן הקלאסי. אבל לא הייתי נכנס לזה, אני חושב שזה יותר יבלבל אותך מאשר ילמד אותך משהו חדש. תשובתי הקצרה והתמציתית היא – פונ’ הגל אומרת לנו כל מה שיש לדעת על החלקיק בכל מקרה, בין אם היא אומרת לנו שהוא בטוח מצוי בנקודה מסוימת ובין אם היא אומרת לנו שהוא יכול להמצא בכל נקודה ביקום בהסתברות שווה (לצורך העניין, נניח שהיקום סופי). הישומים המעניינים של תורת האינפורמציה במכניקת הקוונטים היא כאשר מדברים על חלקיקים שלא ניתן לתאר אותם בעזרת מצב במרחב הילברט כי הם לא חלק ממערכת סגורה. או במילים אחרות, מצבים תרמודינמיים.

  107. לא פוטון אידיאלי – פוטון אידילי.

    כזה ששרוע לו על מזרון פוטון באידיליה, במצב לא מוגדר, מרוח על כל המרחב, מבסוט חלס..

    ניסים

    להבנתי בקוואנטים זה לא שאם אתה יודע את התנע בדיוק אז אינך יודע את המיקום – פשוט אין תנע או מיקום עד המדידה.

    אלבנצו

    הרבה אנשים יחשבו שאם אתה כותב ״אין שום הבדל מלבד..״ אז אתה מתכוון לכך שאין שום הבדל מלבד פרטים שוליים, ולא במהות.

    אבל כפי שכתבתי, אם הסכמנו שצד אחד בשזירה משפיע מיידית על הצד השני, דיינו.

    איני פונה רק אליך – הפניה הייתה ״לכל מי שמתנדב לעזור״. אתה התנדבת. להתעלם?

    אז האם הפוטונים שנוצרים אצלי בלייזר אינם מרוחים במרחב בהסתברות שווה? האם הם ממוקדים רק במרחק ct? היש לפוטון אופרטור מיקום בכלל?

  108. אלבנצו
    יש לי שאלה קצת מוזרה. להבנתי, ככל שאני יודע יותר טוב את התנע אז יש לי בעצם יותר אינפורמציה (מספר הביטים המשמעותיים המייצגים את הערך של התנע). האם זה קשור ל”חוסר אינפורמציה” שיש לי על המיקום? כלומר – האם בכלל יש כאן קשר לאינפורמציה, או שההסבר הוא פיסקלי יותר.

  109. ניסים,

    נכון. מצבים שיש להם תנע מוגדר היטב (או מיקום מוגדר היטב) קיימים מתמטית אך הם לא פיזיקליים, אלא כאשר המרחב הוא סופי.

    ישראל,

    1. אני מבין. אז שורש הבעיה היה שאני אמרתי באופן מפורש וברור שיש הבדל והוא שאחד הוא קוונטי והשני קלאסי, ואתה החלטת על דעת עצמך שהבדל זה הוא זניח ומכך הסקת שאני בעצם אומר שאין הבדל. קשה לי לחשוב על הבדל פחות זניח, אבל שיהיה.

    2. אני מודה ומתוודה – אפילו לא קראתי מה כתבת על ניסוי אספה. ברגע שהבנתי שאתה לוקח משהו ספציפי שאמרתי על מקרה מסוים (החידה בה המידע החלוק בין שני משתמשי קצה נקבע מראש כאשר הם תיאמו את זוגות הספינים שלהם) ומיישם אותו על משהו שבכלל לא דיברתי עליו, הבנתי שבמקרה הטוב אצטרך לנבור שוב בדיוני עבר ולעשות לך עוד שיעור פרטי (תגיד, האם שמת לב לאורך הדיון כמה דברים אני מסביר לך? כמה מידע אני חולק איתך? עכשיו עבור על הדיון ותראה כמה דברים אתה סיפרת לי. מצטער, אבל אי אפשר בכלל לקרוא למה שקורה כאן דיון מרוב שהוא חד צדדי, פשוט אין מילה אחרת חוץ משיעור פרטי). במקרה הרע, זה סתם עוד ריב. אשאיר אותך לחשוב על ניסוי אספה בעצמך ולהגיע לאיזו מסקנה שבא לך.

    3. שמע, אני לא יודע איך להסביר את זה יותר ברור ממה שהסברתי. כל חלקיק במכניקת הקוונטים מתואר על ידי משהו שנקרא “מצב”. המצב הזה יכול להיות תדירות מסוימת (מה שאתה קורא לו פוטון אידיאלי), אבל הוא לא חייב להיות – יש גם פוטונים שאינם בעלי תדר מסוים אלא בעלי סופרפוזיציה של תדירויות. פוטונים “אידיאליים” כלשונך (אין בהם שום דבר יותר אידיאלי מאחרים, אבל אאמץ את השפה שלך אם זה יעזור לך להבין) מרוחים בכל המרחב, לא משנה מה התדר שלהם. פוטון אידיאלי בתדר 1 הרץ מרוח על כל המרחב, פוטון אידיאלי בתדר מיליארד טרה הרץ גם מרוח על כל המרחב. פוטונים שאינם אידיאלים לא מרוחים על כל המרחב, וגם כמובן לא נבדלים זה מזה בתדר (הרי אין להם תדר מוגדר היטב, אז די ברור שזו לא התכונה המבדילה ביניהם). לכל היותר ניתן לומר שהם נבדלים בפונ’ הגל שלהם במרחב התדר (שכמובן, אינה מכילה מידע רק על התדר שלהם אלא על כל התכונות הפיזיקליות שלהם כמו מיקום).

    4. פוטון אידיאלי כלשונך (כלומר בעל תדר מסוים) לא יכול להיווצר בפנס. הוא מרוח באופן שווה על פני כל המרחב, גם ברגע שנוצר וגם 13 מיליארד שנה אח”כ. הדבר היחיד שיכול לשנות זאת היא אם הוא משנה את המצב הקוונטי שלו, כתוצאה מאינטראקציה עם מערכת אחרת או כתוצאה ממדידה. אז הוא כבר לא יהיה מרוח על כל המרחב, אבל גם כבר לא יהיה אידיאלי.

  110. ישראל
    להבנתי,אלבנצו אומר שברגע שיש לך מידע על הפוטון, ביצעת עליו מדידה, ולכן הוא כבר אינו “מפוזר בכל המרחב”. זה דומה לניסויי החריצים – ברגע שאתה מסתכל דרך איזה חריץ עבר החלקיק אז הוא כבר לא עובר דרך שניהם (אין התאבכות).
    לכן – אותו פוטון מהפנס שלך ינוע בצורה קלסית.

  111. אלבנצו
    אני חושב שאני מבין מה אתה אומר:
    1) אם יש לי ידע מלא על התנע של חלקיק אז אין לי שום ידע על מיקומו, ולכן יש הסתברות שווה למצוא אותו בכל מקום במרחב.
    2) בפועל – לא ייתכן שנדע בצורה מוחלטת את התנע (או את המיקום).

  112. נתחיל מהסוף.

    ״אולי אתה חושב שאני רק רוצה להשפיל אותך ושאני נהנה לספר לך שאתה לא מבין ולא יודע, אבל אני מבטיח לך שזה לא המקרה״.

    מה פתאום? אני שמח שאתה אומר לי דברים שאותם לא ידעתי ומנסה ללמוד מהם.

    אבל זה קצת שונה כשאתה מתחיל עם השקרן, אדיוט, נוכל ושות.. האם לפחות במקרה זה אתה רואה את ההבדל היסודי?

    אבל האם זה אומר שלכל אני מסכים? שלילי. אם אתה מביא טיעון נכון שאני מבין ומקבל אני מסכים ומודה לך, כמו שעשיתי שהסברת על 4 מצבי בל.

    במידה ולא, אני ממשיך לשאול שאלות ומציב את ההבנה שלי מול שלך. זו הדרך לקיים דיון וללמוד, עד שאחד הצדדים משתכנע (או לא).

    אבל מה לעשות שכשאיני מסכים אתך אתה מיד מתרתח ומתחיל לקלל אותי? טוב נקווה שזה ישתפר.

    הנה דוגמה:

    כתבת: ״אין שום הבדל בין פתרון זה לבין כל פתרון אחר של קוד קלאסי מתואם מראש (כפי שניסו חלק מהמגיבים פה להציע), מלבד זה שהקוד הזה פועל לפי חוקים קוונטים, ובפרט הוא מצוי בסופרפוזיציה״.

    בעיני זה כמו להגיד: עליך לבחור בין שתי התאומות חיה ומוארטה. אין שום הבדל ביניהם מלבד כמה פרטים שוליים – לחיה כתם לידה על הגב ולמוארטה צלקת קטנה ביד וכו.

    אבל כשמנסים להבין מהו ההבדל, מתברר שאכן הן זהות גנטית, נראות אותו הדבר לבושות אותם הבגדים הכל זהה חוץ מהעובדה הזניחה והשולית שחיה – חיה, ומוארטה – מתה (מוארטה בספניולית). זניח.

    אז מה פירוש ״אין שום הבדל בין פתרון זה לבין כל פתרון אחר של קוד קלאסי מתואם מראש״? ההבדל הוא בעיקר, לא?

    ומה זה משנה אם נפגשת קודם ותיאמת את הספינים? מהרגע שהכנסת נתונים חדשים – מצב המקטבים בניסוי אספקט, שהוא משתנה אחרי שהפוטונים עזבו את המקור ולכן אינו כלול בתיאום הקודם – התיאום הקודם לא רלוונטי יותר.

    ניתן לראות זאת בדוגמת המספרים שנכתבים על פתקים ומופרדים. נכון שבתיאום קלסי אני מסתכל על המספר שלי ויודע מה המספר שלך ללא העברת אינפורמציה.

    אבל מהרגע שהוכנס גורם נוסף – לדוגמה כל צד צריך להכפיל את המספר שלו במספר אחר שהצד השני אינו יודע עליו – אז מה משנה יותר כל התיאום המוקדם?

    לא משנה. אם הסכמנו שבשזירה מדידה של צד אחד *משפיעה* על המדידה בצד השני (דבר שלא קיים במקרה של מספרים או כפפות שהופרדו) אז אין לנו חילוקי דעות יותר, וכבר כתבתי מזמן שלא משנה לי אם יקראו לזה אינפורמציה, אפשר לקרוא לזה גם משה.

    רק עיצה קטנה לך כמורה: תשתדל כשאתה כותב ״קורלציות לא לוקליות״ ו״חוקים קוואנטיים״ גם להזכיר שזה כולל את יכולת ההשפעה מרחוק.

    פוטונים. עדיין לא ברור לי: האם יש הבדל בין פוטונים מלבד אורך הגל? האם יכול להיות שני פוטונים בעלי אורך גל זהה ושעדיין קיים איזה שהוא הבדל ביניהם? חבילות גלים שונות אולי?

    ומידה והתשובה היא שלילית. האם כשאתה אומר ״נניח שאני מסתכל על פוטון ספציפי שלו אכן יש תנע מוגדר היטב, כלומר אי-הוודאות שלו בתנע היא 0. כן, במקרה זה תהיה לו אי-וודאות אינסופית במיקום, כלומר הוא יהיה מרוח בכל המרחב״, זה לא מחייב שבעצם כל הפוטונים כאלו? הרי ההבדל בין אותו פוטון אידילי לאחרים הוא רק התדר שלהם, לא? (לא נכנס עכשיו לפורי טוריה ומהו התדר, מספיק שקיים פוטון אחד שמרוח שווה במרחב).

    ושאלה שניה. אותו פוטון אידילי יכול להיווצר אצלי בפנס?

  113. ישראל,

    נתחיל מהסוף. אני מצטער, הבנת הנקרא שלך לוקה. כשאומרים “אין הבדל בין משה לדני מלבד זה שמשה בלונדיני ודני ג’ינג’י”, האם זה אומר שמשה ודני הם זהים? האם כשתפגשו אותם תדע להבדיל ביניהם או לא? כתבתי שאין הבדל *מלבד* זה שאחד מהם מצייט לחוקים קלאסיים והשני לקוונטים. זה הבדל ע-צ-ו-ם. איך הגעת מזה למסקנה שאני אומר שאין הבדל בין מערכות לוקאלית למערכות לא-לוקאליות? כתבתי במפורש, שחור על גבי לבן, שיש הבדל וההבדל הזה הוא ההבדל בין החוקים הקוונטים לקלאסיים. הקוונטים מאפשרים אי-לוקאליות, הקלאסיים לא. אלוהים אדירים, זה כאילו אתה אפילו לא מנסה לקרוא מה כותבים לך. כרגיל, הבעיה נובעת מזה שאתה מתביית על משפט אחד (או פסקה אחת) שאתה חושב שיעזרו לך לקדם את הדיעה שלך ולהוכיח שאתה צודק, ומתעלם מכל השאר. חזור לדיון. בדיון המקורי ניסיתי להסביר לך משהו פשוט:

    1. קלאסית, אי אפשר לפתור את החידה מבלי להעביר מידע. כן ניתן לפתור על ידי תיאום מראש של קוד בין שני הנסיינים.
    2. אפשר לפתור אותה בעזרת ספינים שזורים.
    3. אתה הסקת מכך שהספינים השזורים כוללים העברת מידע.
    4. זה שגוי. הספינים השזורים שקולים לא אנאלוגיים לפתרון העברת המידע, אלא אנאלוגיים לפתרון התיאום מראש. לראיה, לא ניתן ליישם שיטה זו מבלי ששני הנסיינים יפגשו לפני ויתאמו את המערכות שלהם. אפשר גם לכמת את כמות המידע שהם צריכים לתאם ביניהם מראש ולראות שהיא לא פחות מכמות המידע שהם היו צריכים להעביר זה לזה כדי לפתור את החידה.

    זה כל מה שאני הולך להגיד בנושא. אמרתי לך שפה זה לא שעת סיפור שאתה יכול להעלות בה כל זכרון מהעבר שבא לך ולצפות שאני אבדר אותך. הסיבה היחידה שעניתי על השאלה שלך היא כי הייתי בהלם מרמת הבנת הנקרא שלך (“אתה אומר שההבדל היחיד בין משקולת של קילו למשקולת של טון זה המשקל? אז אתה בעצם אומר שאין שום הבדל ביניהן?!”)….

    בנוגע לפוטון, אני מצטער לבשר לך – אתה עדיין לא מבין. קודם כל, זה שיש לך מכשיר מדידה שמראה אורך גל מסוים לא אומר שלפוטון יש אורך גל מסוים. כרגיל אסביר שאני לא מתכוון להעליב אותך, אבל אני חייב לחזור על המשפט הקבוע – אתה פשוט לא מבין את היסודות הבסיסיים של מכניקת הקוונטים. גם פוטון שמצוי בסופרפוזיציה של אינסוף תדירויות שונות יראה לך תדירות אחת ספיציפית כאשר אתה מודד אותו. אתה פשוט לא קולט שמצב קוונטי של חלקיק ותוצאות המדידות שמתקבלות עליו הם דברים שונים. זה למעשה כל מכניקת הקוונטים על רגל אחת – במכניקה קלאסית חלקיק מתואר באופן יחיד ומלא על ידי אוסף תוצאות המדידה עליו, בעוד שקוונטית זה מפורש לא נכון. יתכנו שני חלקיקים שונים עם פיזיקה שונה לחלוטין שיתנו בדיוק את אותן תוצאות מדידה.

    באמת שזה הכי פשוט בעולם, וזה בדיוק כמו שאמרתי כבר: הוא נוצר בכל המרחב. השאלה שלך “היכן הוא נוצר” היא כמו לשאול מה צבע השיער של אדם קירח או כמה מטרים נכנסים בקילוגרם. היא שאלת נונסנס שנובעת מחשיבה קלאסית שהיא שגויה באופן מפורש במכניקת הקוונטים. החל מרגע פליטת הפוטון הוא מצוי בכל המרחב. הוא לא נוצר ליד הפנס (כמובן שאני מדבר על הפוטון בעל תדירות מוגדרת היטב, ולא על הפנסים שיש לך במעבדה שיוצרים פוטונים שהם חבילות גלים). ברגע שאתה מודד את מיקומו, אתה מקריס את פונ’ הגל שלו. הקרסה זו מבטיחה שאם תמדוד את מיקומו שוב, כלומר בפעם שניה, אחרי שניה אחת, תקבל תוצאה שנמצאת בדיוק שניית אור אחת מהמקום הראשון בו מצאת אותו. אם אתה באמת רוצה להבין את הדברים האלה אתה חייב ללמוד מבוא לקוונטים, ספציפית ללמוד מהי מדידה, מהו מצב קוונטי ומה ההבדל ביניהם.

    בנוגע להסתברות, שניכם צודקים במידת מה. לחלקיק בעל תנע מוגדר היטב אכן הסתברות שווה להמצא בכל נקודה במרחב. צודק ניסים שאומר שהתפלגות אחידה כזו לא אפשרית במרחב אינסופי, או יותר נכון – לא ניתן לנרמל אותה. זה נכון לגמרי ואכן במכניקת הקוונטים על מרחב אינסופי לא יתכנו חלקיקים בעלי תנע מוגדר היטב (כלומר, המצבים העצמיים של אופרטור התנע הרציף לא נמצאים במרחב הילברט, אלא במרחב עזר אשר כולל בתוכו את מרחב הילברט והם מה שנקרא overcomplete basis of states). לכן הקפדתי לומר שניתן ליצור אותם רק בקירוב – הם אובייקטים פיזיקליים אמיתיים רק במרחב סופי. אנחנו עדיין מדברים עליהם במכניקת הקוונטים מכמה סיבות: ראשית, הם לא באמת פיזיקליים אבל קרובים כרצוננו למצבים פיזיקליים אמיתיים. שנית, כל מצב פיזיקלי ניתן לבנות מהם ולכן הבנה של הפיזיקה שלהם מאפשרת להבין את הפיזיקה של כל מצב פיזיקלי אמיתי. שלישית, הם נותנים לנו אינטואיציה טובה כי המתמטיקה שלהם פשוטה יותר מאשר של חבילות גלים.

    ישראל, אתה ממשיך לחזור שוב ושוב על אותן טעויות. מקודם אמרת שלכל פוטון יש תנע מסוים. הסברתי לך שזה לא נכון קוונטית. אז עכשיו אתה אומר שהדבר היחיד שמבדיל בין פוטונים זה התנע. אוקי, שינית כמה מילים אבל אתה עדיין מתעקש להתעלם לגמרי ממכניקת הקוונטים ולעסוק רק בפיזיקה קלאסית, שם לכל פוטון יש מסלול שמגדיר באופן יחיד מיקום ותנע. למקרה שזה עדיין לא ברור – לא, ההבדל בין פוטונים הוא לא רק בתדר שלהם. לרוב הפוטונים אין תדר, כפי שהוסבר לך כבר מספר פעמים. ההבדל בין פוטונים הוא במצב הקוונטי שלהם (שכולל את התפלגויות התדרים האפשריות שלהם). מצטער להשמע כמו תקליט שבור, אבל אם אתה רוצה באמת להבין משהו מהדברים האלה, אתה חייב ללמוד מבוא לקוונטים. אתה אולי חושב שאני רק רוצה להשפיל אותך ושאני נהנה לספר לך שאתה לא מבין ולא יודע, אבל אני מבטיח לך שזה לא המקרה. תזכור שאני מלמד כחלק מהמקצוע שלי ושיש לי אינסטיקנטים של מורה. אני באמת אומר לך את הדברים האלה כדי לעזור לך להבין – יש לך פער ענק ברעיונות הבסיסיים ביותר של מכניקת הקוונטים (מה מגדיר חלקיק במכניקת הקוונטים? מה ההבדל בין תיאור החלקיק הפיזיקלי לבין התיאור שאנו רואים כשאנו מודדים אותו? מה עושה מדידה לחלקיק? מה הקשר בין תנע למיקום? וכו’ וכו’ וכו’). בלי השלמה של פערים אלה לא תצליח להבין את הפתרונות לשאלות שלך, ורק תשקע עמוק יותר ועמוק יותר לתוך אי-ההבנה שלך.

  114. ELBENTZO

    בבקשה, ענה על השאלה שהצגתי לך מקודם.

    כתבת ״אין שום הבדל בין פתרון זה לבין כל פתרון אחר של קוד קלאסי מתואם מראש (כפי שניסו חלק מהמגיבים פה להציע), מלבד זה שהקוד הזה פועל לפי חוקים קוונטים, ובפרט הוא מצוי בסופרפוזיציה״

    האם החוקים הקוונטים והסופרפוזיציה כוללים את האפשרות להשפיע באפס זמן מקצה אחד של הניסוי על הקצה השני, דבר שלא קיים בקוד הקלסי? זה הרי שורש הבעיה, לא?

    הנושא חשוב כי יש שתי אפשרויות:

    1. התשובה חיובית, דהיינו החוקים הקוונטים והסופרפוזיציה כוללים את האפשרות להשפיע באפס זמן מקצה אחד של הניסוי על הקצה השני, דבר שלא קיים בקוד הקלסי, ואז למעשה קיים הבדל גדול ויסודי בין פתרון זה לבין כל פתרון אחר של קוד קלאסי מתואם מראש (כפי שניסו חלק מהמגיבים פה להציע).

    2.התשובה שלילית, ואז אין באמת הבדל בין שתי האפשרויות.

    אם האפשרות היא 1 – אז אתה אומר בעצם שאין הבדל בין מערכת שמאפשרת אי לוקליות למערכת שלא מאפשרת את זה?

    אם התשובה היא 2 המצב חמור עוד יותר. אתה לא רואה את ההבדל הברור בין שני המקרים?

    ובכל מקרה, אם אתה אומר שאין הבדל – אז למה אתה מתרגז וקורא לי שקרן וסלפן כאשר אני אומר לך שתנסה ליצור קוד קלאסי כזה? אנחנו הרי יודעים שקוואנטית זה עובד, אז אם אין הבדל כדבריך אז למה שלא תנסה להציע מערכת כזו?

    נראה לי שכמו מקרים רבים בעבר, פשוט לא הבנת את שאני אומר ומיד התנפלת עלי.

  115. מה רע בהסתברות שווה?

    מיואונים לא קשורים לנושא הפוטונים. פוטונים הם חיה שונה בג׳ונגל החלקיקים, כי הם היחידים (למיטב ידיעתי) שמצייתים לפוסטולט 2.

    אורך חיי המיואונים מתארך או שהמרחק שהם עוברים בדרך לקרקע מתקצר בגלל התארכות הזמנים או התקצרות האורך על פי היחסות. זה קורה לכל עצם, במיוחד לעצמות של תאומים.

    אך זה לא קשור לנושא שלפנינו, אלא אם כן אתה רואה בכך הוכחה סלאם דאנק ליחסות.

    הנושא שלפנינו הוא זה: אם לפוטון ספציפי שלו אכן יש תנע מוגדר היטב, כלומר אי-הוודאות שלו בתנע היא 0 יש אי-וודאות אינסופית במיקום, כלומר הוא יהיה מרוח בכל המרחב, האין זה מחייב שכל הפוטונים הם כאלו? הרי הדבר היחיד שמבדיל בין פוטונים הוא אורך הגל שלהם, אז מדוע נבדל חלקו של פוטונינו משאר אחיו? מה מיוחד בו?

    ואם כל פוטון בעצם מרוח בהסתברות שווה בכל המרחב, אז למה אנחנו מודדים אותם רק במרחק ct מהמקור?

    אני מאמין שייתכן שזה בגלל שאמצעי המדידה שלנו אינם יכולים למדוד את החלקים המהירים או האיטיים של הפוטון מc אלא רק את החלק שנע יחסית אליהם בדיוק בc, אך יכולה להיות סיבה אחרת.

  116. ישראל
    אורך החיים של מיואון הוא קצר מידי מכדי שיעבור את האטמוספירה. מבחינת צופה על הקרקע, אנחנו רואים שהזמן של המיואון מתארך, כלומר אורך חייו גדל.

    אבל מבחינתו, המסלול דרך האטמוספירה חייב להתקצר.

  117. ישראל
    לא כל כך נראה לי שאתה מבין מה זו התפלגות אחידה (אני מניח שלזה הכוונה שלך).
    התפלגות כזו יכולה להתקיים אך ורק מעל תחום סופי.
    למדנו את זה בתיכון…

  118. ככתוב:

    ״אי-הוודאות שלו בתנע היא 0. כן, במקרה זה תהיה לו אי-וודאות אינסופית במיקום, כלומר הוא יהיה מרוח בכל המרחב״.

    ומה שלא כתוב אך אני מאמין שאלבטזו יאשר בקרוב:

    בהסתברות שווה.

  119. ישראל
    כתבתי מקודם – יש התסברות גבוהה מאד למצוא אותו איפה שאתה מצפה, הסתברות נמוכה יותר בסביבה המאד קרובה לנקודה זו, והסתברות מאד מאד מאד נמוכה למצוא אותו במרחק גדול יותר (אני חושב שההסתברות מעריכית).

    איך הגעת להסתברות שווה?

  120. בוא נראה מה כתב אלבנטזו:

    נניח שאני מסתכל על פוטון ספציפי שלו אכן יש תנע מוגדר היטב, כלומר אי-הוודאות שלו בתנע היא 0. כן, במקרה זה תהיה לו אי-וודאות אינסופית במיקום, כלומר הוא יהיה מרוח בכל המרחב.

    אותו, אותו אני אוהב. רק לא ברור לי: היכן הוא נוצר? אני יכול ליצור אותו בסנטה מוניקה? כי במידה וכן, ואיני רואה למה לא, אז למרות אי הודאות האינסופית במרחב וההסתברות השווה להימצא בכל מקום, היה סמוך ובטוח שרק במרחק שניית אור תקלוט אותו שניה אחרי שהוא נוצר.

    אז להיכן נעלמו שאריותיו הפזורות בהסתברות שווה במרחב?

    מה הבעיה עם מיואונים? התארכות הזמנים?

  121. ישראל
    זה לא נכון שתקלוט אותו בכל מקום – יש הסתברות מסויימת שתקלוט אותו בכל מקום, וזה משהו אחר לגמרי. ההסתברות שתקלוט אותו במרחק c*t אחרי זמן t הוא מאד גבוה, ושי הסתברות מאד קטנה שתקלוט אותו בסביבה הקרובה לנקודה זו.

    דרך אגב – איך אתה פותר את בעיית המיואונים? איך הם מגיעים לקרקע? הם לא פוטונים ולא נעים במהירות c.

  122. הלייזר שלי שנמצא בסנטה מוניקה פולט פוטון ירוק מונוכרומטי באורך גל של 507.65789654346 ננו מטר. זה מה שמראה מכשיר המדידה.

    אם הוא נוצר בכל היקום אז בכל נקודה ביקום אחרי שניה נקלוט אותו בכל נקודה אחרת. מכיוון שהמקום היחיד שבו הוא נקלט הוא במרחק שניית אור מסנטה מוניקה, אז כנראה ששם הוא נוצר.

    ולא הבנת את דוגמת הגומי. חשוב רק על 100 אבנים שנעות במהירויות מ1 עד 100 מטר בשניה. אם אני חייל מג״ב בשטחים שהאבנים נעות בכיווני כמטח, התוכל להגיד לי מה מהירות המטח?

    ניסים זו אינה בעיה, זה הפיתרון.

  123. ישראל
    גם אז, לא הייתי שם כסף שאלה אותם פוטונים.
    אבל בוא נניח שכן – מה הבעיה כאן? באלקטרוניקה יומיומית משתמשים ברכיבים שמבוססים על תופעות כאלה (אתה יכול לקנות אלה באמזון).

  124. אם אתה נמצא בלוס אנג’לס ומדליק פנס שיוצר פוטון חדש, והפוטון הזה הוא *פוטון בעל תנע מוגדר היטב* (כלומר בעל אורך גל מסוים ולא חבילת גלים) אז הוא לא נוצר בלוס אנג’לס. הוא נוצר באופן שווה בכל נקודה ביקום, כלומר בסופרפוזיציה. ברוך הבא למכניקת הקוונטים, ממליץ לך לחזור על קורס המבוא משנה ב’. כמו שאמרתי לך כבר כמה וכמה פעמים, האינטואיציה שלך שהפוטון יוצא מהפנס ויש לו מסלול (אלומה) שאפשר לעקוב אחריה נובע באופן ישיר מכך שהפנסים שאתה מכיר לא מייצרים פוטונים בעלי תדירות מוגדרת, אלא מייצרים אלומות גל שמכילות רצף של תדירויות.

    ולא, גם בעולם קלאסי לאובייקט הגומי אפשר להגדיר מהירות רק אם הוא גוף קשיח, כלומר, על החלקים המרכיבים אותו נעים ביחד. אתה יכול לחזור על זה כמה שתרצה, זה לא הופך את זה לנכון. כל חלק בגומי נע במהירות אחת בודדת, ומפני שהמהירות לא זהות זו לזו נוצרת מתיחות בגומי.

  125. כי לא שלחתי פוטון אחד, שלחתי את כל הסרט חלף עם הרוח וקיבלתי אותו באנדרומדה.

    יתכן אמנם שזה לא אותם פוטונים, אך אותו הדבר ניתן להגיד גם על פוטון הפנס המצוי.

    האם אני מסכים? אנא ערף, מאיפה אני יודע אני? אני אומר שזו אפשרות סבירה לפחות כמו האלטרנטיבה: עולמות מרובים והשפעה על העבר. צריך לעשות ניסוי ולבדוק.

  126. ישראל
    כן – אבל האם אתה מסכים עם זה?

    דרך אגב – הפוטון שלך לא נמצא באנדרומדה אחרי 10 שניות. בוא נניח שעשינו את הניסוי. שמת שני גלאים, אחד באנדרומדה ואחד במרקח 3 מיליון ק”מ. אחרי 10 שניות לא קלטת פוטון בגלאי הקרוב, וכן קלטת פוטון באנדרומדה.
    מניין לך שזה אותו פוטון?

  127. ובקשר לno comm.

    מה שכתבתי נכון. בהתקן אותו תיארתי עם מכשירי הרדיו והמטבעות, n.c מתייחס רק לחלק השני, דהיינו שליחת אינפורמציה באמצעות ההתקן שהיא בלתי אפשרית. הוא אינו עוסק בחלק הראשון, דהיינו איך עוברת התקשורת בין מכשירי הרדיו עצמם.

  128. מתוך

    https://www.hayadan.org.il/is-the-universe-ramdom-0405168/comment-page-6/#comment-707429

    ״לבסוף, בנוגע לההערה שלי על כך שכל המידע כבר היה במערכת. אז התחלנו עם 300 זוגות ספינים שזורים, נכון? אבל רגע, איך קיבלנו אותם? והאם מספיק לדעת שהם שזורים? כל זוג צריך לייצר על ידי מדידה לוקאלית. כלומר, לפני תחילת הניסוי, שני הנסיינים (זה של כדה”א וזה של מאדים) היו צריכים להפגש איפהשהו ולייצר את הזוגות השזורים. אבל זה לא מספיק. הם חייבים לדעת בדיוק איזה מצב שזור יש להם (כי גם במערכת הקטנה ביותר יש לפחות 4 מצבים שזורים), אחרת לא ניתן להגדיר את הקוד. אז לא רק שהם צריכים להפגש לפני ולבצע מדידה, הם גם חייבים לדעת בדיוק איזה מדידה ביצעו ומה היה תהליך הקריסה של המערכת. רק אז כל אחד מהם יכול ללכת לנקודת הקצה שלו ולבצע את הניסוי. כלומר, הם באים עם כמות *עצומה* של מידע שתואם מראש, וכל מה שהם עושים במהלך הניסוי זה להקריס אותו בעזרת מדידה לוקאלית, כל אחד על הספין שלו. אין שום הבדל בין פתרון זה לבין כל פתרון אחר של קוד קלאסי מתואם מראש (כפי שניסו חלק מהמגיבים פה להציע), מלבד זה שהקוד הזה פועל לפי חוקים קוונטים, ובפרט הוא מצוי בסופרפוזיציה (כידוע סוד הקסם של שזירה היא שהיא מצב סופרפוזיציה, ולא סתם אלא סופרפוזיציה מיוחדת שהיא לא ספרבילית לתתי מערכות טהורות)״.

    כשאתה אומר:

    ״אין שום הבדל בין פתרון זה לבין כל פתרון אחר של קוד קלאסי מתואם מראש (כפי שניסו חלק מהמגיבים פה להציע), מלבד זה שהקוד הזה פועל לפי חוקים קוונטים, ובפרט הוא מצוי בסופרפוזיציה״ האם החוקים הקוונטים והסופרפוזיציה כוללים את האפשרות להשפיע באפס זמן מקצה אחד של הניסוי על הקצה השני, דבר שלא קיים בקוד הקלסי? זה הרי שורש הבעיה, לא?

    ״במאמר לא מוזכרת בעיה במודל המתמטי. זה בדיוק מה שאני אומר, למה אתה לא מבין? במאמר מדברים על מתיחות מסוימת, אבל מתיחות זו היא בפילוסופיה של הפיזיקה ולא במתמטיקה שלה״.

    לא ראיתי במאמר איזכור למודל המתמטי. הנה מה שכן כתוב במאמר על המודל הפיזי:

    ״מכניקת הקוונטים ניפצה אינטואיציות רבות, אבל זוהי העמוקה שבכולן. והניפוץ המסוים הזה נושא בכנפיו איום שטרם הוסר צִלו, איום על תורת היחסות הפרטית, מאבני היסוד של פיזיקת המאה ה-21 שלנו.

    הדבר שהכי מדאיג באי-לוקאליות, לבד מן המוזרות המטלטלת הטבועה בה, הוא שהתכונה הזאת טומנת בחובה איום כמוס על תורת היחסות הפרטית כפי שאנו מכירים אותה כיום. בשנים האחרונות נעשתה הדאגה הזו, שסוף סוף זכתה להיכנס להיכל המחשבות הרציניות בתחום הפיזיקה, למוקד של דיונים שעשויים, בסופו של דבר, לקעקע, לעוות, לדמיין מחדש, לגבש או לפורר את עצם יסודותיה של הפיזיקה.

    החדשות הרעות אפוא היו לא מנת חלקה של מכניקת הקוונטים אלא של עקרון הלוקאליות, וממילא, כך נראה, של תורת היחסות הפרטית, מכיוון שלפחות לכאורה, היא בנויה על הנחת לוקאליות.

    דומה שסוג האי-לוקאליות שנתקלים בה במכניקת הקוונטים דורש סימולטניות מוחלטת, שמציבה איום ממשי וקטלני על תורת היחסות הפרטית.

    מצבה של תורת היחסות הפרטית, רק קצת יותר ממאה שנה לאחר שעלתה על בימת העולם, נהיה פתאום לשאלה פתוחה לרווחה ומתפתחת במהירות״.

    אז אתה טוען שביטויים כגון ״לפורר את עצם יסודותיה של הפיזיקה״ ו״איום ממשי וקטלני על תורת היחסות הפרטית״ הם בסך הכל ״מתיחות מסוימת, אבל מתיחות זו היא בפילוסופיה של הפיזיקה ולא במתמטיקה שלה״?

    ״הנה דוגמא מעולה לעיוות מטורף של דברי ואפילו המצאה: “אתה טוען שבסבטקסט הם התכוונו בעצם להיפך ושאין איום ושהן מסתדרות מצויין ושכותבי המאמר פברקו משהו מדומיין כדי להגדיל את התפוצה?”

    האם לא כתבת לפני כמה ימים:

    ״קודם כל, צריך לזכור שכל מאמר במגזין – גם מדעי, ובטח שבסיינטיפיק אמריקן שפונה לקהל הרחב ויש לו אינטרס ענק למכור עותקים – יכול ליפול לנטייה טבעית לכותרות סנסציוניות או בומבסטיות״.

    וגם:

    ״לסיכום, קשה לי מאוד מאוד מאוד לראות איך בתוכן הכתבה יש איזהשהו סיכון לשילוב של יחסות פרטית ומכניקת הקוונטים״.

    אכן אין סיכון. ״לפורר את עצם יסודותיה של הפיזיקה״ ״איום ממשי וקטלני על תורת היחסות הפרטית״ . סתם עיוותתי פברקתי והמצאתי, וכמו תמיד שקרתי שקרתי ושוב שקרתי.

    עניינית:

    ״5. כפי שכתבתי מקודם, לפוטון בעל תדר מסוים אכן תהיה אי-וודאות אינסופית במיקום. אתה שואל אם הוא יכול להמצא במרחק של עשר שנות אור מהמקום בו היה לפני 10 שניות. לשאלה זו יש תשובה בעלת שני חלקים: ראשית, לפני עשר שניות גם לא ידעת איפה הוא. זה הרי פוטון בעל תנע מסוים, המיקום שלו לא ידוע. אתה מניח שלפני עשר שניות ידעת איפה הוא, אבל זה פשוט לא אפשרי. בדיוק כשם שעכשיו אתה לא יודע איפה הוא, גם לפני עשר שניות לא ידעת״.

    אני נמצא בלוס אנג׳לס ומדליק פנס ויוצר פוטון חדש שלא היה קיים קודם. אחרי 10 שניות הפוטון נמצא בין היתר גם באנדרומדה, ששעוניה מסונכרנים עם הארץ.

    אז מה פירוש ״לפני עשר שניות גם לא ידעת איפה הוא״? הוא היה בארץ, שם הוא נולד. ואם אחרי 10 שניות הוא באנדרומדה, אז זהו פוטון זריז ונמהר מאוד, לא?

    ״הגומי לא נע באף מהירות, זה מה שאני מסביר לך. אין אובייקט פיזיקלי כזה “הגומי”. זאת אומרת, ברור שאתה יכול להגדיר אותו (החיבור של כל החלקיקים שמרכיבים אותו) אבל אי אפשר לשייך לו מהירות״.

    אנחנו מדברים על עולם קלסי, ובו יש אובייקט פיזיקלי כזה, גומי. אם אני יורה 10 אבנים וכל אחת נעה במהירות שונה מחברתה, אז הן נעות ב10 מהירויות. ואם הן מחוברות זו לזו בגומי, אז האובייקט אבנים + גומי נע ב10 מהירויות + כל המהירויות באמצע. כך גם עם 1000 וטריליון אבנים, עד שבסוף תשאר רק עם הגומי שנע בכל המהירויות מ0 עד לגבוהה ביותר.

    ומה שחשוב לעניינינו, תיאור מוזר כזה לפוטון פותר תיאורים מוזרים לא פחות כגון השפעה על העבר וגם מסביר את פוסטולט 2.

    אינך חייב להגיב אם אינך מעוניין, לילה טוב.

    ניסים

    זה מה שקוונטים טוענת, לא?

  129. 1. אני מעדיף לא לחזור לריבים ישנים ובהחלט לא רוצה להתחיל לנבור בשיחות עבר. כל מה שקרה פה זה זה שאתה שמת לינק לכתבה מסוימת ואני נתתי דעתי עליה. אח”כ כתבת משהו על משפטי no-comm ואני ראיתי שמה שאתה כותב לא נכון ועלול גם להטעות אחרים, ולכן כתבתי הבהרה. בשום שלב לא התכוונתי ועדיין איני מתכוון לפתוח איתך כל דיון או ריב שאי פעם היה לנו. אבל בחיים אין אבסולוטים ויתכן שתרצה לפתוח שיחה ישנה כי היא דווקא נושא שיחה מעניין שרלוונטי למשהו שכתבתי פה. במקרה זה בהחלט יתכן שאחרוג ואשמח להבהיר למה התכוונתי, כפי שעשיתי כאשר שאלת אותי על אי-הוודאות של פוטון (למרות שכבר הסברתי לך זאת בעבר). לסיכום – וזה הכי ברור שאני יכול להיות – אני לא מעוניין לפתוח דיונים ישנים, אבל איני מתחייב במ-100% שאם תבקש ממני הבהרה לטענה קודמת אסרב. אם הטענה מעניינת ונראה לי רלוונטי או שמגיעה לך הבהרה, אבהיר. במקרה זה אתה חייב לתת לי קישור מלא למה שאמרתי ולקונסטקס בו זה נאמר כי רק כך אוכל להסביר למה התכוונתי.

    2. במאמר לא מוזכרת בעיה במודל המתמטי. זה בדיוק מה שאני אומר, למה אתה לא מבין? במאמר מדברים על מתיחות מסוימת, אבל מתיחות זו היא בפילוסופיה של הפיזיקה ולא במתמטיקה שלה. זה כל מה שטענתי בתגובה המקורית שכתבתי על המאמר, וזה מה שאני טוען עכשיו. בעיני יש הבדל *עצום* בין בעיה במודל התיאורטי לבין בעיה באינטרפרטציה הפילוסופית. אם אתה מעדיף לא לעשות את ההבדל, תפדל. זכותך. לי היה נראה חשוב מאוד להעיר על כך.

    הנה דוגמא מעולה לעיוות מטורף של דברי ואפילו המצאה: “אתה טוען שבסבטקסט הם התכוונו בעצם להיפך ושאין איום ושהן מסתדרות מצויין ושכותבי המאמר פברקו משהו מדומיין כדי להגדיל את התפוצה?”. אתה יודע היטב שמעולם לא אמרתי שום דבר כזה. לא כתבתי שהם פברקו כלום. לא כתבתי שהם התכוונו ההיפך. אני רק חוזר ואומר – לצערי אתה לא מצליח להבין, אבל זה כבר בינך לבין עצמך – שהבעיה שהם מציגים היא בעיה בפילוסופיה של הפיזיקה, שגם הבעיה עצמה וגם הפתרון שלה מצוי באינטרפרטציה של איך אנו מבינים את המודל ואת המציאות, ולא בעיה במודל המתמטי עצמו.

    3. מה זאת אומרת “למה אין לו אורך גל מוחלט במערכת יחוס נתונה”? למה 2+2 לא שווה 9? תלמד מכניקת הקוונטים. לפוטון יש מצב (כלומר, מתואר על ידי וקטור במרחב הילברט). מצב זה יכול להיות מצב עצמי של אופרטור התנע (במקרה זה כל מדידה של תנע הפוטון במערכת יחוס מסוימת תתן תמיד תוצאה כלשהי, ולתוצאה זו אנו קוראים התנע של הפוטון) אבל מצב הפוטון יכול להיות גם מצב שאינו מצב עצמי של אופרטור התנע. מכיוון שאופרטור התנע הרמיטי והמצבים העצמיים שלו הם סט שלם שפורס את מרחב הילברט אז תמיד ניתן יהיה להציג מצב זה כסופרפוזיציה של מצבים עצמיים של תנע – כלומר, לפוטון לא יהיה תנע מסוים אלא כל מדידה תתן בהסתברות כזו תנע אחד, בהסתברות אחרת תנע שני וכך הלאה. אתה פשוט מתעקש להתעלם ממכניקת הקוונטים ולהסתכל על תמונה קלאסית שבה לכל חלקיק יש תנע מוגדר היטב.

    בנוגע לפוטון הירוק – נתחיל בזה שכנראה שמעולם לא ראית פוטון ירוק. כשאנחנו רואים פוטון “ירוק” במציאות אנחנו למעשה רואים פוטון במגוון אורכי גל, אך הירוק הוא בעל האמפליטודה הגבוהה ביותר מביניהם. כלומר, מה שאתה קורא לו פוטון ירוק וחושב באופן אינטואיטיבי שהוא בעל אורך גל מדויק שמתאים לצבע ירוק הוא בד”כ פשוט חבילת גלים צרה יחסית שממורכזת מסביב לתדר הירוק. פוטון בעל תדירות ירוקה בדיוק זה, כמו שאמרתי מקודם, משהו שאפשר ליצור בקירוב במעבדה ולא באמת נמצא בטבע. אבל שוב – כמו שאמרתי מקודם – אני לא טוען שאין דבר כזה פוטון ירוק, שזה לא יכול להתקיים. אני רק טוען שזה לא נכון באופן כללי. להגיד “לפוטון יש אורך גל מסוים” זה כמו להגיד “לפי מכניקה ניוטונית, גוף מאסיבי מפעיל סביבו כוח כבידה של 3 ניוטון”. באופן כללי זה לא נכון (הכוח תלוי במסה של הגוף ובמרחק ממנו) אבל ברור שיתכנו מצבים בהם כוח הכבידה הוא באמת 3 ניוטון… יש פוטונים ירוקים. אבל להגיד באופן כללי שלפוטון יש אורך גל מסוים זה פשוט לא להבין את היסוד של הבסיס של ההתחלה של ההקדמה של מכניקת הקוונטים. מצטער.

    5. כפי שכתבתי מקודם, לפוטון בעל תדר מסוים אכן תהיה אי-וודאות אינסופית במיקום. אתה שואל אם הוא יכול להמצא במרחק של עשר שנות אור מהמקום בו היה לפני 10 שניות. לשאלה זו יש תשובה בעלת שני חלקים: ראשית, לפני עשר שניות גם לא ידעת איפה הוא. זה הרי פוטון בעל תנע מסוים, המיקום שלו לא ידוע. אתה מניח שלפני עשר שניות ידעת איפה הוא, אבל זה פשוט לא אפשרי. בדיוק כשם שעכשיו אתה לא יודע איפה הוא, גם לפני עשר שניות לא ידעת. אתה מניח שאתה יכול לדעת שהוא נפלט מהפנס ולכן היה שם, בפנס. אבל קוונטית זה לא נכון. אם יש לך פנס שפולט פוטונים בעלי תנע מסוים, הוא יכול לפלוט אותם בכל נקודה במרחב – גם הרחק מאוד מעצמו. שוב, זה כמובן קשור לכך שהפנסים שאתה מכיר מחיי היומיום פשוט לא עובדים ככה ולא פולטים פוטונים בעלי אורך גל ספציפי (ולכן כן אפשר פחות או יותר לעקוב אחרי מיקום אלומת האור שלהם).

    החלק השני של התשובה קשור לזה שמכניקת הקוונטים היא תורה סטטיסטית. מצב קוונטי (או פונ’ גל) אינם כפופים לכל החוקים שאתה רגיל אליהם, ובפרא לכך שפוטון נע במהירות האור. מה שכפוף לחוקים האלה זה *מדידות*. יש כמובן הוכחה מתמטית לכך שאם נמדוד מיקום של פוטון ברגע מסוים, ונמדוד את מיקומו שוב אחרי שניה, ההפרש בין המיקום יהיה שניית אור אחת. אבל זה קשור לתהליך המדידה בו פונ’ הגל קורסת. זה לא נכון לגבי המצב הקוונטי של הפוטון, שיכול להיות בסופרפוזיציה של כל מיני מצבים. כאשר הוא לא נמדד החלקיק הוא לצורך העניין וירטואלי ואינו מחוייב לקיים את משוואות התנועה (גם את זה כבר הסברתי לך בעבר). במכניקת הקוונטים התנאים הפיזיקלים מתקיימים על הדברים שאנו מודדים.

    6. הגומי לא נע באף מהירות, זה מה שאני מסביר לך. אין אובייקט פיזיקלי כזה “הגומי”. זאת אומרת, ברור שאתה יכול להגדיר אותו (החיבור של כל החלקיקים שמרכיבים אותו) אבל אי אפשר לשייך לו מהירות. בדיוק כמו שאפשר להגדיר אובייקט “משה ודני” אבל אי אפשר לשייך לאובייקט זה מהירות. אם דני עומד במקום ומשה בחללית שנעה קרוב למהירות האור, זה לא אומר שהאובייקט “משה ודני” נע במהירות האור וגם עומד במקום בו זמנית. אפשר לשייך לאובייקט מהירות רק כאשר כל החלקיקים המרכיבים אותו נעים ביחד, וזה מה שנקרא גוף קשיח. אנו יודעים שבמציאות אין דבר כזה גוף קשיח, אבל בפיזיקה קלאסית הוא קיים ומהווה קירוב טוב (לדוגמא, אפשר לשאול באיזו מהירות אני נע למרות שגם אני מורכב מהמון חלקיקים שונים. ההתייחסות אל אדם כגוף קשיח אינה מדויקת אך מהווה קירוב טוב מאוד בסקאלות היומיומיות שאינן יחסותיות או קוונטיות).

    7. אם תחתוך את הגומי כלום לא יקרוב. פשוט בנקודה מסוימת לא יהיו יותר כוחות פנימיים בגומי, כלומר מתיחות, וכתוצאה מכך חצי מהגומי יפסיק להאיץ (שוב, יש פה תגובת שרשרת כי עדיין יש מתיחות גם בחלק החתוך). אין לי מושג למה אתה חותר.

    ישראל, אתה עדיין מתעלם ממה ששאלתי אותך, ואתה שוב ושוב מתייחס אלי כאל מורה פרטי שאתה יכול פשוט להציף אותו בשאלות ולצפות שילמד אותך מכניקת קוונטים או יחסות. עם כל הרצון הטוב, אין לי זמן לזה. אם אני כותב משהו (בתגובה אליך או למישהו אחר) ואתה רוצה הבהרה או רוצה לטעון שאני טועה או משהו, זכותך. אבל אני חושב שהשעת סיפור הזו בה אתה פשוט יושב ושופך עלי שאלות על פוטונים ואי-וודאות ותדירויות וסופרפוזיציות וגופים קשיחים וגומיות ומה ההבדל בין פיזיקה לבין הפילוסופיה שמאחורי הפיזיקה, הגיעה לקיצה.

    לילה טוב.

  130. לא מבין.

    אתה רוצה שאביא קישורים למה שאמרת בעבר כפי שמשתמע מ״בבקשה תן לי קישור מדויק למה שאמרתי ולקונטקסט״ או אינך רוצה שאביא קישורי עבר כפי שמשתמע מ״לא יודע למה אתה חושב שיש לי זמן ללכת איתך לחפש ריבים מהעבר ולהתפלש בהם שוב״.

    ידע אותי בצורה ברורה, קריאת מחשבות וסבטקסט לא כל כך עובדות בזמן האחרון.

    איפה מוזכרת במאמר ״בעיה במודל המתמטי שמנסה לשלב בין מכניקת הקוונטים ליחסות פרטית, איזו מבין שלוש האפשרויות מתקיימות – האם תורת השדות אינה יחסותית כפי שאני טוען, האם היא אינה קוונטית כפי שאני טוען, או האם היא פשוט תורה אינה קונסיסטנטית? אני טוען שברמת המודל אין שום סתירה ואף נתתי דוגמה למודל שמכיל את שתי התיאוריות. עכשיו עליך לעזור לי להבין שאני טועה והמודל לא באמת משלב בין השניים״.

    הבאתי 4-5 ציטוטים מהמאמר כולל הכותרת: איום קוונטי על תורת היחסות. אתה טוען שבסבטקסט הם התכוונו בעצם להיפך ושאין איום ושהן מסתדרות מצויין ושכותבי המאמר פברקו משהו מדומיין כדי להגדיל את התפוצה? תפדל, שיהיה לבריאות.

    אבל אין ספק שלפחות על פניו, בפשט, הם אומרים בדיוק מה שאני אומר, לא? אם כך הסבר את כל הציטוטים שהבאתי.

    ״לפוטון אין בהכרח תנע מוחלט״ למה אין לו אורך גל מוחלט במערכת יחוס נתונה? אם כן מהו אורך הגל של פוטון ירוק מסויים? והאין התנע של הפוטון שווה לחלוקת קבוע פלאנק המוחלט באורך גל הפוטון?

    ״אבל בוא נתקדם. נניח שאני מסתכל על פוטון ספציפי שלו אכן יש תנע מוגדר היטב, כלומר אי-הוודאות שלו בתנע היא 0. כן, במקרה זה תהיה לו אי-וודאות אינסופית במיקום, כלומר הוא יהיה מרוח בכל המרחב. אז מה? לא ברור לי למה אתה חותר, או למה בכלל בחרת פוטון כי זה נכון לגבי כל דבר במסגרת מכניקה קוונטית״.

    יופי. אז בוא נעבוד עם הפוטון הספציפי הזה. יש לו אי וודאות מוחלטת במקום? הוא יכול להיות גם במרחק שעת אור מהמקור שפלט אותו לפני 10 שניות? אז אם מהירות מוגדרת כחלוקת מרחק בזמן, האם לא קיבלנו מהירות גבוהה בהרבה ממהירות האור?

    ״באופן כללי, אם תסתכל על הפוסטים האחרונים תראה שיש פה מגמה חזקה שבה אתה פשוט מנסה לחפש ריבים״ איך הגעת לזה? אפילו לא פנית אליך למעלה משנה אלא אתה הוא זה שפנית ופונה אלי.

    אבל בוא נמשיך באופן הנוכחי. דווקא מתחיל להיות פרודוקטיבי.

    ״זה שכל חלק בגומי נע במהירות אחרת לא אומר “שהגומי נע בכל המהירויות”.

    אז באיזה מהירות נע הגומי? מה אם נסמן כל מאיון ממנו ונמדוד את מהירות אותו מאיון, האם לא נקבל בקירוב 100 מהירויות שונות שעולות בהדרגה מ0 למהירות החץ?

    ומכייוון שזו רק דוגמה לצורה בה אני רואה את הפוטון, אם נחתוך את הגומי בנקודה כלשהי במהלך מעופו של החץ (״מדידה״) האם לא ״יקרוס״ הגומי באחת כאשר מהירות הקריסה היא בסדר גודל של מהירת הגומי? מזכיר משהו?

    לילה טוב, 3 בבוקר.

  131. זה שכל חלק בגומי נע במהירות אחרת לא אומר “שהגומי נע בכל המהירויות”. זאת אומרת, אתה יכול לומר זאת אם בא לך, אבל אין לזה משמעות בפיזיקה כי הגומי אינו גוף קשיח ולכן לא ניתן לשייך לו מהירות. ניתן לשייך לו מהירות ממוצעת, כמובן, או לומר שכל חלק אחר שלו נע במהירות קצת אחרת (כלומר, מתיחות). אם אני רץ במהירות מסוימת, ניסים רץ קצת יותר מהר, משה רץ עוד יותר וכן הלאה, האם זה אומר שהאדם נע בכל המהירויות? פיזיקלית זו הצהרה חסרת תוכן, למרות שאפשר להבין אותה מילולית.

    הבהרה: אם הגומי עשוי מסדרה של חלקיקים, לכל אחד מהם יש מהירות אחת מוגדרת (כמובן שכרגע אנחנו מדברים באופן קלאסי). אין אף חלקיק שלא נע במהירות אחת מסוימת. הרעיון של “לנוע בכל המהירויות” הוא רק תוצאה של זה שאתה לוקח הרבה חלקיקים שונים ומחליט להתייחס אליהם כאובייקט אחד. אבל הם לא, זה בדיוק מה שנקרא גוף קשיח ואנו יודעים שזה לא אפשרי פיזיקלית – לא בגומי ולא באף גוף מורכב אחר.

  132. 1. כרגיל, אתה מדבר בכותרות. בכתבה הכותבים לא רק אומרים את המילים “יש בעיה” כמו שאתה כל הזמן מצטט, הם גם מפרטים מה לדעתם הבעיה. עצור, קח קצת לקרוא על שני ההיבטים הבעייתים שהם מציגים, ותבדוק מה בעייתי בהם. תפסיק לצטט באופן עיוור את המילים “יש בעיה” רק כי זה מתאים לאג’נדה שלך, תלמד מה היא בדיוק הבעיה. אני השקעתי בנושא קצת זמן (לא רק בקריאת הכתבה עצמה אלא גם מחקר על הנושאים שמוצגים בו בירחונים מקצועיים ובאינטרנט) ואני טוען שהבעיה היא פילוסופית לחלוטין ואינה משתקפת במודל המתמטי. גם הכותבים מספרים על פתרון אפשרי על ידי אימוץ אינטרפרטציה (של מרחב הילברט כמרחב בו אנו חיים במקום מרחב-זמן). במקום כל הזמן לדרוש ממני לענות לך ולהסביר לך, אולי אתה פעם אחת תהיה זה שמסביר: תסביר לי אם יש בעיה במודל המתמטי שמנסה לשלב בין מכניקת הקוונטים ליחסות פרטית, איזו מבין שלוש האפשרויות מתקיימות – האם תורת השדות אינה יחסותית כפי שאני טוען, האם היא אינה קוונטית כפי שאני טוען, או האם היא פשוט תורה אינה קונסיסטנטית? אני טוען שברמת המודל אין שום סתירה ואף נתתי דוגמה למודל שמכיל את שתי התיאוריות. עכשיו עליך לעזור לי להבין שאני טועה והמודל לא באמת משלב בין השניים.

    2. אין לי מושג על מה אתה מדבר. אני לא חושב שאי פעם התפלספתי פה, אני מדבר רק על פיזיקה. שוב, סעיף שלם בלי תוכן, אתה רק מחפש ריב.

    3. אני לא מצליח להבין מה לא ברור לך. עכשיו הוספת את המילים “תמיד אמרת”. אוקי. אני אגיד את זה שוב, אולי קצת יותר לאט – אם אתה רוצה שאני אסביר לך טענה מסוימת שכביכול טענתי, בבקשה תן לי קישור מדויק למה שאמרתי ולקונטקסט. זה בכלל לא קשור לפיזיקה, זה פשוט הגינות בסיסית. כשמבקשים מבנאדם להסביר טענה, חייבים לתת לו את האפשרות להבין בדיוק מה היא הטענה, מתי נאמרה ובאיזה הקשר. אם תתן לי קישור מדויק לדיון ישן, אנסה לפנות זמן לקרוא אותו להסביר שוב. למרות שכמו שאני מקווה שאתה מבין, לריב איתך זה די בתחתית סדר העדיפויות שלי. לא יודע למה אתה חושב שיש לי זמן ללכת איתך לחפש ריבים מהעבר ולהתפלש בהם שוב.

    4. לפוטון אין בהכרח תנע מוחלט. כל חלקיק (גם פוטון, אבל גם אלקטרון וכו’) במכניקת הקוונטים מצוי במצב קוונטי מסוים. המצב הזה יכול לייצג חלקיק שיש לו תנע מוגדר היטב (ברמה המתמטית – המצב הוא מצב עצמי של אופרטור התנע), אבל לא חייב להיות. חלקיק יכול להיות בסופרפוזיציה של שני תנעים, או בכל חבילת גלים אינסופית המכילה ספקטרום של תנעים שונים. זה נכון גם לפוטונים. למעשה, פוטונים עם תנע מדויק זה משהו שנמצא רק בקירוב במעבדה, וכל הפוטונים שאתה נתקל בהם בחיי היומיום הם למעשה חבילות גלים שאין להם תנע מוגדר.

    אבל בוא נתקדם. נניח שאני מסתכל על פוטון ספציפי שלו אכן יש תנע מוגדר היטב, כלומר אי-הוודאות שלו בתנע היא 0. כן, במקרה זה תהיה לו אי-וודאות אינסופית במיקום, כלומר הוא יהיה מרוח בכל המרחב. אז מה? לא ברור לי למה אתה חותר, או למה בכלל בחרת פוטון כי זה נכון לגבי כל דבר במסגרת מכניקה קוונטית.

    באופן כללי, אם תסתכל על הפוסטים האחרונים תראה שיש פה מגמה חזקה שבה אתה פשוט מנסה לחפש ריבים, שואל אותי שאלות מסתוריות בלי להסביר מה אתה רוצה (טקטיקה ידועה של לנסות להפיל מישהו בפח) ולא ברור מאיפה צצו פתאום, ונובר בריבי עבר. במקביל, אתה מתעלם ממה שאני כותב, בפרט על הקיום של מודל מתמטי שלם וקונסיסטנטי שהוא גם קוונטי וגם יחסותי. זה כבר ממש מתחיל לשעמם.

  133. ניסים

    פוטונים יחודיים בכך שהם נעים בc יחסית לכל דבר. כרגע בוא נדבר על פוטונים כי היחסות עוסקת בהם.

    באיזו מהירות נע החץ? והחלק שקשור לעץ? והמרכז שביניהם חוצץ?

    כל חלק בגומי נע במהירות שונה (לא?) ולכן הגומי נע בכל המהירויות.

  134. ישראל
    שזירה היא לא רק בין פוטונים. החלקיקים יכולים להיות אלקטרונים, מולקולות גדוחות ואפילו יהלומים קטנים. הרעיון שלך מכסה גם את אלה?

    בקשר לגומי: אין אף חלקיק שם שיש לו יותר ממהירות אחת.
    אם לחלקיק יש כל מהירות, אז יש לו תנע אינסופי – איך זה מסתדר? וגם אנרגיה אינסופית…

  135. ניסים

    חשוב על גומי ארוך שקשור לעץ בצד אחד ומחובר לחץ בצד השני.

    אתה יורה את החץ. באיזו מהירות נע הגומי? לא מ0 עד מהירות החץ ליניארית?

    איני אומר שזה מה שקורה, אבל הסבר כזה לפוטון יכול להסביר הרבה תופעות מוזרות, כולל אי לוקליות, פוסטולט 2 ופרדוקסי ההשפעה על העבר.

  136. 1. ״במאמר מוצגת מתיחות מסוימת ברמה הפילוסופית בין יחסות פרטית למכניקת הקוונטים. מובהר שמתיחות זו אינה מובילה בהכרח לסתירה, ומוסבר שניתן לפתור אותה ברמת האינטרפרטציה – כלומר, בפילוסופיה. אין כל איזכור לסתירה פיזיקלית (כלומר, סתירה במודלים)״.

    מן המאמר:

    ״הדבר שהכי מדאיג באי-לוקאליות, לבד מן המוזרות המטלטלת הטבועה בה, הוא שהתכונה הזאת טומנת בחובה איום כמוס על תורת היחסות הפרטית כפי שאנו מכירים אותה כיום. בשנים האחרונות נעשתה הדאגה הזו, שסוף סוף זכתה להיכנס להיכל המחשבות הרציניות בתחום הפיזיקה, למוקד של דיונים שעשויים, בסופו של דבר, לקעקע, לעוות, לדמיין מחדש, לגבש או לפורר את עצם יסודותיה של הפיזיקה״.

    שום פילוסופיה, רק פיזיקה.

    ״מצבה של תורת היחסות הפרטית, רק קצת יותר ממאה שנה לאחר שעלתה על בימת העולם, נהיה פתאום לשאלה פתוחה לרווחה ומתפתחת במהירות. המצב הזה נוצר מכיוון שסוף-סוף הלכו הפיזיקאים והפילוסופים בעקבות הקצוות הפרומים של אותו ויכוח נשכח של איינשטיין עם מכניקת הקוונטים״.

    נכון, פילוסופים (אז מה?), אבל גם פיזיקאים.

    אז מה ההבדל בין מה שאומר המאמר למה שאני אומר?

    2. ״ברור שמדידה בצד אחד משפיעה גם על הצד השני״.

    אז מה כל הפלסף על הכפפות ועל המספרים שנכתבו ונשלחו לקצוות שונים? האם יש איזו שהיא דרך שמדידה או איזה פעולה אחרת שנעשו על כפפה או על מספר שנכתב על פתק ישפיעו איך שהוא על הצד השני?

    ומה זה הקשקוש על זה ששיקרתי בכך שעיוותתי את דבריך? לא אמרת חזור ואמור שאין הבדל בין קוד קלסי לקוד קוואנטי חוץ מזה שבקוד קלסי ניתן לעשות דברים שונים? האם הדברים השונים האלו כוללים השפעה מרחוק באפס זמן? האם לא זהו שורש המחלוקת?

    אמרתי פעמים רבות שאם מסכימים על כך שבשזירה צד אחד משפיע על הצד השני באפס זמן אין צורך לקרוא לזה אינפורמציה, אפשר לקרוא לזה משה.

    3. תמיד טענת שלא עוברת אינפורמציה בגלל שכל הנתונים כבר נמצאים במערכת.

    אז כדי שלא תטען שאני מנסה להכשיל אותך, בניסוי אספקט לא כל הנתונים במערכת, מצב המקטבים נקבע אחרי השזירה.

    4. חייב לרוץ (פוקר!) והנושא חשוב ונרחיב עליו יותר מאוחר, אבל הנה העיקר:

    לפוטון הרי יש תנע ברור ומוחלט. האם אין זה מחייב חוסר מיקום מוחלט?

  137. 1. במאמר מוצגת מתיחות מסוימת ברמה הפילוסופית בין יחסות פרטית למכניקת הקוונטים. מובהר שמתיחות זו אינה מובילה בהכרח לסתירה, ומוסבר שניתן לפתור אותה ברמת האינטרפרטציה – כלומר, בפילוסופיה. אין כל איזכור לסתירה פיזיקלית (כלומר, סתירה במודלים). להבנתי אתה מייצר מצג שווא על ידי ציטוט סלקטיבי של דברים כמו הכותרת והתעלמות מגוף המאמר. אבל האמת היא שאני לא כל כך מבין מה אתה רוצה ממני כי אני לא חושב שהאשמתי אותך במשהו (מלבד המסקנה שהמאמר מצביע על סתירה בין יחסות פרטית למכניקת הקוונטים). הטענות שטענתי היו על המאמר ולא על דבריך. אם יש משהו ספציפי שאמרתי שמתייחס לדבריך שאתה רוצה שאסביר או שלטענתך שגוי, פרט מה הוא ואתייחס. כרגע נראה לי שאתה סתם מחפש בכוח עוד ריב כל ידי בקשה כללית שאמצא בכוח משהו שאנחנו לא מסכימים לגביו.

    2. מה פתאום. מדידה של מצב שזור גורמת לקריסה של המצב על כל חלקי המערכת. ברור שמדידה בצד אחד משפיעה גם על הצד השני. אני רק מנסה להסביר לך שההשפעה הזאת היא לא ביטוי של מעבר אינפורמציה. היא מבטאת קורלציה, אבל אלה שני דברים שונים. אין לי מושג מאיפה בכלל עלה לך הרעיון שאני מתכחש לאחת התופעות המוכרות והנחקרות ביותר של מכניקה קוונטית של המאה העשרים. באותה מידה יכלת לשאול אותי אם אני מתכחש לכך שחלקיק יכול להמצא בו זמנית בשני מקומות שונים.

    3. כמו שכבר כתבתי לך, אין לי מושג למה אתה מתכוון. אם אתה רוצה שאתייחס לטענה שטענתי בעבר, מן ההגינות שתביא לי את הטענה כולל כל הקונטקסט שבו היא נטענה, ואשמח לנסות להבהיר אותה. לתת לי ציטוט של ארבע מילים ולהגיד לי שזה “בנושא ניסוי אספה”, ואז לצפות ממני להיות מסוגל להבין על מה מדובר ולהסביר לך זה או בדיחה גרוע או נסיון מכוון להכשיל אותי.

    4. בטח שעקרון אי-הוודאות חל על פוטונים. למעשה, כבר שאלת אותי את זה בעבר וכבר הסברתי לך. אבל אשמח להסביר שוב.

    מכניקת הקוונטים כפי שנכתבה בשנות העשרים והשלושים של המאה הקודמת (שורשיה מוקדמים יותר כמובן אבל אלה שנות הצמיחה העקריות) אינה תורה יחסותית. בתורה זו חיבור מהירויות מתבצע על ידי טרנספורמציית גליליאי, שינוי בפוטנציאל גורם לשינוי מיידי בתאוצות של כל הגופים במרחב וכו’. כלומר, היא סתרה את יחסות פרטית.

    באופן דומה, יחסות פרטית כפי שנכתבה ב-1905 הייתה הרחבה של מכניקה קלאסית. בפרט, המיקומים והתנעים של חלקיקים הם פונ’ וניתן לדעת אותן (לא צריך להסתכל על פוטונים, אפשר להסתכל על סתם חלקיק מאסיבי של חומר). חלקיקים לא יכולים להמצא בו זמנית בשני מקומות, לא יכולים לבצע התאבכות וכו’. תורה זו אינה קוונטית והיא סותרת את מכניקת הקוונטים.

    אבל כבר לפני זמן רב – קרוב למאה שנה – אנשים הבינו שבעיות אלה הן בעיות בניסוחים של התיאוריות. ניתן ליישם את העקרונות של יחסות פרטית על בוריהן בתיאוריה אשר בה מיקום ותנע אינם פונ’ אלא אופרטורים בעלי אי-וודאות. ניתן ליישם את כל עקרונות המכניקה הקוונטית ללא יוצא מן הכלל בתיאוריה בה חיבור מהירויות אינו מבוצע על ידי סכימה ישירה. כבר שנים בודדות אחרי פרסום תורת היחסות אנשים הבינו שלא מדובר בתורה אחת, אלא בסט עקרונות ושכל תורה שמקיימת אותו היא “יחסותית”. כמו כן, מכניקת הקוונטים אינה תורה בודדת אלא סט עקרונות וכל תיאוריה שמקיימת אותם נקראת “קוונטית”. עכשיו ניתן היה לשאול את השאלה האם יש תורות שהן גם קוונטיות וגם יחסותיות? התשובה ניתנה כבר לפני 80 שנה בערך (לדעתי דיראק היה הראשון ששאל אותה וביצע התקדמות משמעותית מאוד במציאת התשובה). כמובן שהתשובה היא כן – אין שום בעיה לכתוב תורה שהיא גם יחסותית וגם קוונטית. ב-1965 קיבלו פיינמן, שווינגר וטומונגה פרס נובל על כך שכתבו תורה מלאה, קונסיסטנטית מתמטית ובעלת דיוק תצפיתי גבוה, אשר הייתה גם קוונטית וגם יחסותית (אלקטרודינמיקה קוונטית).

    בפרט, אם תקח את התורה הזו ותסתכל על הגבול המתמטי בו מהירות האור הולכת לאינסוף (כלומר, כל המהירויות זניחות ביחס למהירות האור ובגבול זה טרנספורמציית לורנץ מזדהה עם טרנספורמציית גלילאי), תקבל בדיוק את התיאור הקוונטי של אלקטרומגנטיות כפי שהכירו אותו בגרסה המקודמת ביותר של מכניקת הקוונטים, לפני שידעו כיצד לשלב אותה עם יחסות. אם תקח את אותה תורה אבל במקום הגבול של מהירות האור הגבוהה תסתכל על הגבול שבו קבוע פלאנק הולך לאפס (כלומר, הגודל האופייני הקוונטי קטן באופן זניח ביחס לגודל של כל מערכת), תקבל בדיוק את התיאור של אלקטרומגנטיות במסגרת יחסות פרטית, כפי שפורסמה ב-1905 ולפני שהבינו שניסוח אחר שלה יושב בקנה אחד עם קוונטים. הגבולות האלה הם לא סתם המצאה שרירותית: הגבול של מהירות האור הולכת לאינסוף היא בדיוק מה שמקשר בין תורת היחסות הפרטית לבין מכניקה ניוטונית והגבול של קבוע פלאנק הולך לאפס הוא בדיוק מה שמקשר בין מכניקת הקוונטים לבין מכניקה ניוטונית.

    לסיכום – אם תסתכל על האופן שבו איינשטיין כתב את יחסות פרטית ב-1905, תראה תמונה שלא מסתדרת עם מכניקת הקוונטים. אבל מה לעשות, ב-1905 עדיין אף אחד לא ממש ידע כלום על קוונטים, אז זה לא פלא שאיינטיין לא כתב את התיאוריה באופן קומפטבילי. ברגע שמכניקת הקוונטים התבססה, אנשים מצאו דרכים שקולות לחלוטין לנסח את יחסות פרטית מבלי להצטרך לדעת בו זמנית מיקום של חלקיק וגם את התנע שלו וכו’, ולא הייתה בעיה לכתוב תורה יחסותית וקוונטית.

  138. נו שוין, אי אפשר להגיד שלא ניסינו..

    “אתה שיקרת פעמים רבות, כי לרוב הטיעונים שלך ריקים מתוכן, בגלל הנטייה החזקה שלך לדבר על דברים שאין לך מושג בהם”

    אז אתה טוען ששיקרתי במודע.. יפה. תמיד חשבתי שהפיזיקה עוסקת בעובדות והגיון, אך הנה פיזיקאי בעל כוחות שעוסק בתורת הנסתר יודע עלי דברים שאפילו אני לא יודע..

    כמה שאלות אליך:

    1. התוכל להסביר לי ולפורום מה ההבדל בין מה שנאמר במאמר למה שאני אומר?

    2. האם אתה טוען שמדידה של חלקיק שזור אחד לא משפיעה על מצב החלקיק השני?

    3. האם בניסוי אספקט “כל הנתונים כבר נמצאים במערכת” כפי שטענת מספר פעמים כדי להסביר מדוע לא עוברת אינפורמציה בין החלקיקים?

    4. והשאלה שהצגתי (אני באמת רוצה לדעת): האם עקרון אי הוודאות חל על פוטונים? נראה לי שזה מתחייב, אחרת ניתן היה להשתמש באפקט קומפטון כדי לדעת את מיקומו והתנע המדוייק של אלקטרון (לא בטוח אבל זה נראה הגיוני).

    ואם כן – אז איך זה מסתדר עם היחסות שבבסיסה מיקום ותנע מוגדרים לפוטון?

  139. ישראל,

    :). אתה יודע בדיוק מה אני חושב, אז למה אתה מבזבז את זמנך? שוב טכניקלטי ושום נעליים, אתה שיקרת פעמים רבות, כי לרוב הטיעונים שלך ריקים מתוכן, בגלל הנטייה החזקה שלך לדבר על דברים שאין לך מושג בהם. בדיוק כמו שקורה עכשיו, כאשר אתה כותב עוד ועוד דברים על אינפורמציה כאשר ברור לכולם – כולל לך – שאתה פשוט לא יודע כלום על תורת האינפורמציה, כולל ההגדרות הבסיסיות ביותר של מה היא אינפורמציה או מה הוא מעבר אינפורמציה.

    שוב, אני אמשיך לכתובת תגובות על פיזיקה. זו זכותי. אתה יכול לענות לי. זו זכותך. אתה יכול להתעלם. גם זו זכותך. אתה יכול לנסות לגרור אותי לריבים. עשה מה שבא לך.

  140. מממ…

    אז אתה טוען ששיקרתי, ולכן בטח אכשל בבדיקה של גלאי שקר, לא? הרי זה מה שהוא עושה, מגלה שקרים..

    לא עדיף שפשוט תתנצל ותסגור עניין? אתה הרי יודע עכשיו שאתה פשוט מנסה לצאת זכאי על טקניקליטי ואין לך שום אפשרות לדעת אם שיקרתי או שאני מאמין במה שאני אומר (אפילו אם אני אולי טועה).

  141. 1. זה לא הסבטקסט שחסר לך, זה הטקסט. אתה מסתכל על הכותרת (דרך אגב, לא יודע אם אתה יודע או לא, אבל את הכותרת וכותרת המשנה כותב העורך ולא מחבר המאמר) וכל שורת הסיכום. תסתכל על כל מה שבאמצע. כן, כאשר מחברי המאמר כותבים שפון נוימן נתן ערובה לפיזיקאים שאין סתירה, והחל מרגע זה כל חלוקה על טענה זו באה מטיעונים פילוסופים (תחילה מספר מ-94 ואח”כ מפי אחד המחברים), והאיזכור היחיד להתייחסות מדעית עדכנית יותר היא של מתמטיקאי אשר בחר בהסתכלות שונה משל פון נוימן והגיע למסקנה זהה – אני רואה זאת כאישור של מחברי המאמר שאין חלוקה על דו-הקיום בקרב פיזיקאים. לא שאני צריך כזה אישור. כל מה שצריך זה לפתוח כל מאמר מודרני על תורת שדות.

    2. כן, אתה בהחלט שיקרת בדיונים הרבים שהיו לנו. מצחיק מצידך לצפות שאדע עכשיו לצטט לך בדיוק איפה, כמה למה ואיך בהתחשב בזה שמדובר בדיונים שחלקם היו לפני 3 או 4 שנים, אבל יש לי כמה דוגמאות בראש. אחת למשל היא שבמשך תקופה ארוכה היית חוזר וכותב שוב ושוב ציטוט שלי, אשר באופן פלאי תמיד היה נחתך לך בול באמצע המשפט. הסברתי לך פעמים רבות שאתה מביא חצי ציטוט ובכך משנה לגמרי את מה שאמרתי, אבל לך זה לא הפריע. חלקו הראשון של המשפט נגע לכך שיש אנשים שלא מבינים את מכניקת הקוונטים ולכן טוענים שהיא שגויה – ושלך יש אותה בעיה. החלק שהיית משמיט הבהיר במפורש שאני לא מאשים אותך בהכחשת מכניקת הקוונטים אלא באותו סוג של טעות – אתה לא מבין משהו ולכן מסיק שהוא כנראה שגוי. דוגמא נוספת היא שבמהלך דיון ארוך על פתרון חידת מטבעות כזו או אחרת בעזרת שזירה, טענת שאני אומר שיש לה פתרון קלאסי למרות שכתבתי במפורש פעמים רבות שאין לה פתרון קלאסי. רק ניסיתי להסביר לך שהפתרון הקוונטי לא כולל העברת מידע כי הוא דורש סנכרון מראש של מספר עצום של זוגות שזורים – שזו המקבילה הקוונטית לתיאום מוקדם של קוד, רמאות לפי חוקי המשחק. ואתה בכל זאת האשמת אותי שוב ושוב בכך שאני טוען שיש פתרון קלאסי.

    אתה כמובן תדחה את הדוגמאות האלה, למרות שיש עוד המון. אבל זה באמת לא מעניין אותי. אם עדיין לא הבנת, אין לי שום כוונה להתחיל לריב איתך. לא מתכוון גם להתנצל, ולא מתכוון לחתום איתך על שום חוזה. שנינו יודעים בדיוק למה אתה מנסה בכוח לדחוף לשם את הדיון – כי אין לך מה לומר. רוצה לדבר פיזיקה? אפשר. לא רוצה? זכותך, מבטיח לא לזקוף את הסירוב שלך כנגדך כפי שלעיתים אנשים עושים. רוצה לכתוב עוד מאה פעמים שאתה קורבן ושאני רודף אותך? כנראה שכן.

    3. אוקי, כל אחד יכול לטעות לפעמים. לא נורא. אבל אני מקווה שאתה מבין עכשיו שאפילו בסאקוראי אין שום שמץ לחיזוק לטענות שלך, שלא לדבר על הספרות המקצועית הרלוונטית. וזאת בניגוד למה שכתבת בתגובה הקודמת, אשר כביכול ניתן להבין מסאקוראי שיש טעם בדבריך.

    4. “אבל איך נוכל לדעת אם עבר מידע, שימושי או לא, אם סקוראי אינו מגדיר מהו מידע ואת זאת לומדים רק אחרי סקוראי”? קל מאוד. נלך לספר רלוונטי על תורת האינפורמציה במקום לספר מבוא על מכניקת הקוונטים, נלמד אותה, את ההגדרות שלה ואת ההוכחות למשפטים שאנשים גילו. אז נראה שלפי תורת האינפורמציה לא עבר שום מידע. וכרגיל, אזכיר לך שאם בא לך להמציא תורה חדשה עם הגדרה חדשה למידע שימושי ולא שימושי וכל מיני כאלה, זכותך. כרגע הכי טוב שיש לנו זו תורת האינפורמציה ולפי ההבנה שהיא מספקת לנו (שנראה שהיא הבנה מצוינת לפי ההצלחות הרבות שלה בפיזיקה, מדעי המחשב והנדסה), לא עוברת אינפורמציה.

    “ובכלל, איך מדברים הביולוגים בכזה ביטחון על “מידע גנטי” שעובר בתורשה וזאת ללא הגדרה”? גם זה קל. ביולוגים אינם עוסקים בתורת האינפורמציה והם משתמשים במילה אינפורמציה או מידע באותו אופן שאתה משתמש בו, במובן היומיומי. בדיוק כפי שהסטוריון יכול לומר על האימפריה הרומית שהיא הייתה הכוח החזק בעולם בזמנה, אבל זה לא אומר שהיא קיימה את משוואת ניוטון. את ההקבלה בין מידע כפי שהוא מוגדר בתורת האינפורמציה לבין מידע בקוד מסוים (למשל קוד מחשב, או באופן אנאלוגי קוד גנטי) בהחלט ניתן לעשות אך היא לא טריוויאלית. שוב, הפתרון הוא על ידי לימוד התורה והגדרותיה *לפני* שמחליטים שהיא לא נכונה או שמנסים לפרש את המשפטים המוכלים בה.

    האם זה קשור למה שטענת כמה פעמים ש”כל הנתונים כבר נמצאים במערכת”? אין לי מושג על מה אתה מדבר. אולי אתה מדבר על מה שהזכרתי קודם, על כך שפתרון חידת המטבעות באמצעות זוגות רבים של ספינים שזורים דורש שלפני תחילת החידה ישבו שני המשתתפים ויכינו ביחד את כל זוגות הספינים, ואף יוודאו בדיוק באיזה מצב שזור נמצא כל זוג (כי הרי מצב שזור הוא לא יחיד, תמיד יש הרבה ממנו שההבדלה ביניהם אינה טריוויאלית כלל). לא יודע, אולי בקונטקסט הזה אמרתי לך פעם שהם לא מעבירים ביניהם מידע אלא שכבר תיאמו מראש את המידע בעת קידוד הזוגות השזורים. אבל אם אתה רוצה התייחסות ספיציפית יותר אצטרך הפניה בדיוק למה שאמרתי ולקונטקסט בו אמרתי זאת.

  142. ניסים

    פשוט מאוד: אין אפשרות לשלוח אינפורמציה באמצעות שזירה.

    ELBENTZO

    “קיבלת הסברים די מפורטים למה המאמר בסיינטיפיק לא אומר את מה שאתה טוען שהוא אומר”

    שם המאמר הוא: איום קוונטי על תורת היחסות. התיזה המרכזית: ״דומה שסוג האי-לוקאליות שנתקלים בה במכניקת הקוונטים דורש סימולטניות מוחלטת, שמציבה איום ממשי וקטלני על תורת היחסות הפרטית״.

    וזו הצרה״.

    המשפט הפותח את הסיכום שלו: “מצבה של תורת היחסות הפרטית, רק קצת יותר ממאה שנה לאחר שעלתה על בימת העולם, נהיה פתאום לשאלה פתוחה לרווחה ומתפתחת במהירות״.

    האם יש פה איזה שהוא סבטקסט שלא הצלחתי להבין?

    והאם אתה מתייחס למשפט הבא:

    “במשך עשרות שנים ראתה קהילת הפיזיקאים כולה בהוכחה של פון נוימן כעין תעודת ביטוח לכך שהאי-לוקאליות של מכניקת הקוונטים ותורת היחסות הפרטית יכולות להתקיים בשלום זו לצד זו”.

    כסימוכין לקונצנזוס בקהילה האקדמית על חוסר סתירה בין היחסות לקוואנטים? קראת את ההמשך?

    אני לא קורא להם שקרנים חסרי בושה… טוב, אוקי. דרך אגב, הם לא שיקרו למיטב ידיעתי, אז לא ברור לי למה שאקרא להם כך.

    אז אני מניח שאני כן שיקרתי בחוסר בושה..

    תוכל להצביע על מקום אחד ויחיד שבו שיקרתי – דהיינו שאכשל במבחן של גלאי שקר?

    “גם אתה אמרת לי המון דברים לא יפים”.

    תוכל להצביע על פעם אחת ויחידה שבה תקפתי אותך ראשון? כי אני אוכל להצביע על עשרות פעמים שאתה עשית זאת.

    ואם אתה או מישהו מתכוון לבוא עם טיעון הגננת, דהיינו שלא חשוב מי תקף ראשון ומי מגיב – אז האם אותו חוכמולוג מוכן להיות מותקף בעצמו ולסתום?

    “האם אתה מדבר על מקום אחר בספר? או שהתבלבלת בין “לא עובר מידע שימושי” ל”עובר מידע לא שימושי”?”

    לא הסתכלתי בספר כבר הרבה זמן ואני בעבודה, אבל אני מניח שאתה צודק והכוונה היא ל”לא עובר מידע שימושי” .

    אבל איך נוכל לדעת אם עבר מידע, שימושי או לא, אם סקוראי אינו מגדיר מהו מידע ואת זאת לומדים רק אחרי סקוראי?

    ובכלל, איך מדברים הביולוגים בכזה ביטחון על “מידע גנטי” שעובר בתורשה וזאת ללא הגדרה?

    האם זה קשור למה שטענת כמה פעמים ש”כל הנתונים כבר נמצאים במערכת”?

    אנו – בקרוב נחזור לגלקסיה הרחוקה שבה נוכל לדון בחופשיות.

    גם אתם נתקלים בבעיה עם הקלדת הטקסטים? חשבתי שזה רק במק, אבל אני רואה שזה גם בפי סי.

  143. אלבנצו
    תודה!
    לפי הבנתי – מה שתורת היחסות לא “מרשה” זה סתירה. מריו ליביו מתאר ניסוי מחשבתי שמטוס נוחת מתנגש ברכב שחוצה את המסלול (הצופה בקצה מסלול). אם מהירות האור הייתה מסתכמת עם מהירות המטוס אז מבחינת הצופה הם לא היו מתנגשים, וזה כמובן סתירה.
    אבל – אין כל הגבלה, כל עוד לא נוצרת סתירה. בפרט – שזירה לא יוצרת סתירה.

    ובהקשר של מה שאמרתי, בגלל שמדובר על תהליך אקראי, אין משמעות להגיד ש”מידע עבר מצד אחד לצד שני”. מצד שני – אנחנו (אני לפחות) לא ממש מבינים מה קורה שם.

  144. עוד משהו: רציתי לקרוא בסאקוראי על מה שאמרת (“העברת מידע לא שימושי”) כדי לנסות להבין מה קרה שם – האם הוא משתמש במילה מידע פשוט בצורה יומיומית (הוא הרי לא מגדיר אינפורמציה כמושג מתמטי ואפילו מתקרב לנושא בכל הספר), או שאולי לא הבנת משהו כהלכה. קראתי שוב את כל החלקים הרלוונטים בספר ואף הרצתי חיפוש מילים. לא מצאתי שום דבר כזה. כל מה שמצאתי היה מקום שבו כתוב שלא עברה אינפורמציה שימושית (שזה שונה *מאוד* מאשר “עברה אינפורמציה לא שימושית”), וכמה שורות אח”כ כתוב שלא עברה אינפורמציה בכלל. אני חושב שאת הניסוח הלא-זהיר הזה עשו פשוט כי בספר לא מבחינים בין קורלציה לבין אינפורמציה. מדובר במהדורה שניה, עמוד 245.

    האם אתה מדבר על מקום אחר בספר? או שהתבלבלת בין “לא עובר מידע שימושי” ל”עובר מידע לא שימושי”?

  145. ניסים,

    1. לא נראה לי. באופן עקרוני, תורת האינפורמציה מגדירה אינפורמציה ומדברת על אינפורמציה בקונטקסט של משתנים מקריים (התורה הקלאסית, כמובן. בתורה הקוונטית הכל קצת שונה). כל משתנה מקרי הוא בעצם הגרלה אשר לאחר ביצועה מתקבלת תוצאה אחת בלבד, לכן התופעה שאתה מזכיר משותפת לכל אינפורמציה ולא רק לשזירה. למעשה, האינפורמציה קשורה בדיוק ל”קריסה” הזו – במובן מסוים היא מתארת כמה צפוי היה שמבין כל התוצאות האפשריות, דווקא התוצאה שהתקבלה היא הנכונה.

    2. אני לא חושב שהבנתי בדיוק למה אתה מתכוון. אתה יכול לנסות להבהיר? מה שמיוחד בשזירה היא שכל חלק מהמערכת אינו מוגדר היטב לבדו, ולכן כל מדידה למעשה נעשית במקביל על שני החלקים. מה שגורם בעצם לכך שאין פה סתירה ליחסות כללית הוא שמכניקת הקוונטים היא תורה סטטיסטית וניתן “למצע” על אחד מחלקי המערכת. מיצוע זה מראה שמישהו בקצה אחד של המערכת לא באמת מקבל או מוסר מידע למישהו בקצה השני. אני לא בטוח אם זה פישט או סיבך את העניין…. בכל מקרה, גם אני כמוך טוען שאין כל סתירה בין יחסות פרטית למכניקת הקוונטים, וכך גם כל עולם הפיזיקה. כפי שאתה וודאי יודע, במשך שבעים השנים האחרונות (פחות או יותר) התורה השלטת היא תורת השדות הקוונטים, אשר נמצאת שוב ושוב להיות מדויקת באופן חריג ותיאור פנומנלי של המציאות (אם כי אנו יודעים בוודאות שאינה תיאור מלא כי היא אינה יכולה לתאר כבידה, ולכן היא לכל היותר הולוגרפיה של המציאות). תורה זו היא באופן מפורש קוונטית ויחסותית (פרטית בלבד) ומעולם לא נמצאה בה אף סתירה בקונטקסט הזה. לכן גם במאמר שהביא ישראל, אשר מאתגר את יכולתן של התיאוריות לחיות בשלום זו עם זו, כתוב במפורש שאצל פיזיקאים קיים קונצנזוס על כך שאין בעיה, והטענות שמועלות שם (שבניגוד למה שאמר ישראל, איני צוחק עליהן או מזלזל בהן) הן טענות פילוסופיות, וכמו שהכותבים אומרים במפורש, ניתן לפתור אותן על ידי קבלת פרשנות שונה מהמקובל לגבי המציאות בה אנו חיים.

  146. ודרך אגב, בשום שלב לא “הגחכתי” את המאמר בסיינטיפיק. אמרתי שהוא אכן יותר פילוסופי ולא כל כך מדעי, אבל לא הגחכתי אותו ואפילו כתבתי שהוא מעניין. אם תקרא אותו, תראה שגם הכותבים מסכימים איתי – כל הטיעונים שלהם בנוגע לאפשרות של קיום מתח מסוים בין התיאוריות הם טיעונים פילוסופיים. הם בעצמם לא מתיימרים למצוא אף סתירה מתמטית, ואף מדברים על כך שבשנים האחרונות מתמטיקאים מוצאים דרכים יותר ריגורוזיות להראות שאין סתירה (יותר ריגורוזיות מאשר הדרכים הטריוויאליות שהוצעו בשנות השלושים והארבעים, כמו למשל של פון-נוימן).

  147. ישראל,

    אני לא הולך להתנצל בפניך. בניגוד למה שאתה אומר עכשיו, את הדברים שאמרתי לך לא אמרתי לך כי “העזת להביע דיעה שונה משלי”. אמרתי לך שאתה טיפש כאשר חשבתי שאתה מתנהג כמו טיפש, וכו’. גם אתה אמרת לי המון דברים לא יפים. אני לא מתכוון להתנצל ולא מתכוון להכנס שוב לויכוחים אישיים. חשבתי שגם אתה לא מעוניין. אם טעיתי – תהנה, אתה יכול לכתוב מגילות על כמה שאתה קורבן ושסתם יום אחד באתי והתעללתי בך בגלל שלא אהבתי את השם שלך או איזו שטות אחרת. לא נראה לי שאני אגיב לדברים האלה.

    בנוגע לפיזיקה – 1. קיבלת הסברים די מפורטים למה המאמר בסיינטיפיק לא אומר את מה שאתה טוען שהוא אומר, ומה הוא כן אומר. כל מה שיש לך לומר זה שאני לא קורא להם שקרנים חסרי בושה… טוב, אוקי. דרך אגב, הם לא שיקרו למיטב ידיעתי, אז לא ברור לי למה שאקרא להם כך.

    2. אתה רק חוזר על אותה טעות. מביא ציטוט מויקיפדיה אבל לא מבין בכלל מה הוא אומר. אתה הרי לא יודע מה זה observer או מה זה to communicate (כמובן שאתה יודע מה משמעות המילים, אבל לא יודע מה הן מגדירות בתיאוריה המתמטית של תורת האינפורמציה). אז אתה לוקח משפט ומפרש אותו איך שבא לך לפי ההבנה היומיומת שלך של המילים האלה. אם אני טועה ואתה כן יודע מה אומרים מושגים אלה, אנא הסבר לי הרמה המתמטית מהי הטעות שלי.

    3. כמובן שסאקוראי זה רק איש קש שמטרתו – ואני מאמין שכנראה באופן לא מודע – לא להתמודד עם חוסר הידע שלך בתורת האינפורמציה. כפי שהוסבר לך פעמים רבות מאוד בעבר, סאקוראי לא עוסק כלל באינפורמציה. הוא לא “מיתולוגי” והסיבה היחידה שאתה מכיר את השם הזה בכלל היא בגלל שבאופן שבכלל אינו קשור, ניסיתי לגרום לך להבין שיש לך פערי ידע ע-צ-ו-מ-י-ם במכניקת הקוונטים שלא מאפשרים לך בכלל להבין את נושא השזירה והאינפורמציה, כמו אדם שמנסה להבין אנליזה מרוכבת בלי לדעת חדווא. לכן כל הזמן הפנתי אותך לסאקוראי, וניסיתי לגרום לך להבין שהספר הזה שמשקף את הידע של תלמיד בסוף התואר הראשון, נמצא עוד הרבה לפניך. ואני לא אמרתי את זה אז כדי להעליב וגם לא עכשיו, רק כדי לגרום לך להבין. במילים אחרות, כל התפסות שלך למילה כזו או אחרת (שים לב, פיזיקה תיאורטית היא מערכת מתמטית מוגדרת היטב – למילים אין בה הרבה משמעות) בספר שלא מתיימר לומר דבר או חצי דבר על אינפורמציה קוונטית, היא סתם בריחה מהמציאות. לך לטקסטים שעוסקים בנושא, ראה מה היא אינפורמציה, ראה מה הוא מעבר אינפורמציה, ראה מה הוא צופה. אז תוכל לראות שהדבר שאתה קורא לו “אינפורמציה לא ידועה” הוא בכלל לא בהכרח אינפורמציה לפי תורת האינפורמציה, ולכן “מעבר” שלו הוא לא בהכרח מעבר אינפורמציה לפי תורת האינפורמציה. כל הטיעון שלך מבוסס על זה שאתה יוצא מנקודת הנחה אינטואיטיבית-יומיומית שבה כל קורלציה בין שני מצבים מלמד על מעבר אינפורמציה ביניהם, אבל תורת האינפורמציה מגדירה במפורש מידע וקורלציה, ולפיה תיתכן קורלציה שלא כוללת מעבר.

    כפי שאמרתי לך פעמים רבות בעבר -אם יש לך רעיון לתורת אינפורמציה טובה יותר, עם הגדרות אחרות שלפיהן יש מעבר אינפורמציה לא ידועה (ושמשחזרת את ההצלחות המוכרות של התורה הישנה), סחתיין. אני בכיף אלמד משהו חדש. אבל לקחת את התורה הישנה, עם המשפטים הקיימים בה לפי הגדרות מסוימות, ואז לנסות ליישם אותם תוך שינוי ההגדרות והמצאת מושגים חדשים שכלל אינם מוגדרים היטב – זו בדיחה.

  148. ELBENTZO

    ביקשתי ממך שתתנצל על כל הכינויים המעליבים בהם כינית אותי על שהעזתי להביע דעה שונה משלך. מעניין אגב שסיינטיפיק אמריקן טוענים את שאני טוען כמעט מילה במילה, ולמרות שאתה מנסה להגחיך את עמדתם כ״פילוסופית״ ולא מדעית, לפחות אינך קורא להם שקרנים חסרי בושה, טרולים מטומטמים וסנילים המפיצים אשפה ״מדעית״ כביכול שאינם מסוגלים להתמודד עם כך שהם אינם מבינים את מה שילדים בני עשרים מבינים בקלות.

    ולעצם העניין, מתוך ויקיפדיה:

    In physics, the no-communication theorem is a no-go theorem from quantum information theory which states that, during measurement of an entangled quantum state, it is not possible for one observer, by making a measurement of a subsystem of the total state, to communicate information to another observer

    אם מישהו קרא את דוגמת המטבעות ומכשירי הרדיו שהבאתי, אפשר לראות בבירור את שאני טוען:no-communication theorem מתייחסים רק לחלק השני – דהיינו לשליחת האינפורמציה באמצעות ההתקן שהצעתי עם מכשירי הרדיו המשחזר שזירה, ולא לחלק הראשון, כיצד מתקשרים מכשירי הרדיו זה עם זה no-communication theorem בכלל לא מתייחסים לנושא.

    בעניין אינפורמציה לא ידועה: סקוראי המיתולוגי, למרות שנטען שהוא נלמד רק לפני שלומדים את תורת האינפורמציה, מדבר על העברת אינפורמציה לא שימושית בשזירה (וזאת ללא הגדרת אינפורמציה או אינפורמציה לא שימושית).

    אני מאמין שה״אינפורמציה לא שימושית״ של סקוראי זהה ל״אינפורמציה לא ידועה״ שלי. לא אוכל להביא הוכחה ריגורזית מתמטית, אך גם סקוראי לא עושה זאת ומשתמש במושג ללא הגדרה, אז אולי אני בסדר..

  149. אלבנצו
    יש לי שתי שאלות.
    נניח שאישתי איבדה את המפתחות שלי בבית. כל עוד לא מצאתי אותם – יש הסתברות מסויימת שהם בכל מיני מקומות בבית. אני יודע מה ההסתברויות: מהניסיון, המפתחות בד”כ במגירה, לפעמים בכיס המכנסיים, ולפעמים בין הכריות בספה.
    ברגע שמצאתי אותם – כל ההסתברויות האלה מתאפסות. האם הדוגמה הזו נותנת אינטואיציה למצב של שזירה?

    השאלה השנייה קשורה במה שאמרתי לישראל – האם הנקודה החשובה היא אקראיות המדידות? זה נכון שיש שזירה בין המצבים, אבל כל מצב לעצמו הוא אקראי (נניח, קיטוב פוטונים).
    אני מסתכל על זה כאילו שזה בעצם פוטון יחיד שנמצא בו”ז בשני מקומות – אני לא רואה בזה סתירה לתורת היחסות (קוזליות וכו’).

  150. ישראל
    איך אתה מבין את המשפט הבא (מויקיפדיה)
    “In these experiments, the no-communication theorem shows that failure of local realism does not lead to what could be referred to as “spooky communication at a distance” (in analogy with Einstein’s labeling of quantum entanglement as “spooky action at a distance”)”

  151. ישראל,

    או אולי בעצם עלי להפנות את התגובה לניסים ולשאר האנשים שאולי קוראים את הדיון ורוצים להבין, כי ישראל לא רוצה לדבר איתי וזו כמובן זכותו המלאה. המטרה שלי היא לא לגרור אותו לויכוח אלא לספק מידע למי שמתעניין באינפורמציה קוונטית – ומי שלא מתעניין (או שמתעניין אבל לא רוצה לשמוע ממני באופן אישי) יכול להתעלם.

    משפטי no-comm הם משפטים מתמטיים. הם מוכיחים טענות מסוימות בנוגע לתורת האינפורמציה. כמובן שהטענות האלה, כמו התנאים להתקיימות המשפטים, מנוסחות בשפה של מערכת לוגית על הגדרות מסוימות. כלומר, יש הגדרה מתמטית מדויקת למה היא ה”אינפורמציה” במסגרת המשפט, מה הוא “מעבר אינפורמציה” וכו’. זה פשוט מגוחך לנסות ליישם משפטים אלה, אבל להחליף את ההגדרות בהגדרות אחרות (במיוחד שההגדרות האחרות האלה אינן מוגדרות היטב מבחינה מתמטית). מי שבאמת מעוניין בנושא יכול ללמוד את יסודות תורת האינפורמציה ולהיווכח בעצמו שתחת ההגדרות שלה אין “אינפורמציה ידועה” או “אינפורמציה לא ידועה”. אינפורמציה היא גודל כמותי שמוגדר ללא שום תלות ב”צופה” וכאשר משפט no-comm מוכיח שלא ניתן להעביר אינפורמציה, זה נכון גם לשיחה בין שני חברים שמחליטים מה לומר זה לזה, וגם לאות רדיו שאין לו שום שליטה על מערכת אבל יכול לתקשר למשתמש קצה אחר את מצב המטבע שלו ולהפעיל בהתאם לכך מכונה.

    שוב, לסיכום – משפטי no-comm ממש לא אומרים את מה שישראל טוען. הטענה שלו מבוססת על אי-הבנה של מה היא אינפורמציה או מעבר אינפורמציה תחת ההגדרות של התורה המתמטית, והחלפת המושגים הנכונים במושגים האינטואיטיבים שמתאימים לצורת החשיבה של רוב האנשים בחיי היומיום.

  152. ויקי

    לא ידוע על פרדוקסים בלתי פתורים ביחסות, כולל Ehrenfest paradox. אך היא ללא ספק מוזרה ואם קראת את המאמר, מתנגשת אולי עם הקוואנטים, ואם קראת את הקישור לpresentation בסנטה מוניקה, עם עוד כמה תאוריות.

  153. מסילה עגולה והיא עולה.

    אינטואיציה? הלא בקוואנטים ויחסות עסקינן אז מה קשורה האינטואיציה?

    אני רק מנסה להראות עד כמה מוזרה התקצרות האורך (ראה עד כמה הסתבכנו אנחנו ולבסוף הגענו למסקנה שגויה, ראה עד כמה הסתבכו 4 פרופסורים בפרדוקס פרדוקס התאומים ורק אחד מצא תשובה הגיונית, כל היתר הביאו תשובות סותרות) כדי להראות שפיתרון של פוטון הנע בכל המהירויות פותר הרבה בעיות למרות מוזרותו.

    ״עצם העובדה שאי אפשר להעביר מידע בצורה כזו, מראה שקורה כאן משהו שהוא לא “העברת מידע” קלאסי.
    זה נכון ששתי המטבעות תמיד נופלות על אותו הצד, אבל אין לנו שום דרך להשפיע על איזה צד…״

    הנה הסבר שהבאתי כבר פעם.

    חשוב על מטבעות בחדרים שונים ללא תקשורת ביניהם. היש דרך לגרום למטבעות ליפול תמיד על אותו הצד? שלילי.

    עכשיו חבר את המטבעות ברדיו, וכאשר אחד המטבעות נופל על עץ או פלי, האינפורמציה הזו עוברת לחדר השני ומנגנון מכני מסדר את השני מיד באותו המצב.

    עכשיו כמובן ניתן לגרום למטבעות ליפול תמיד על אותו הצד. האם עברה ביניהם אינפורמציה? חיובי. מכשירי הרדיו הם אילו שהעבירו אותה.

    האם ניתן לשלוח באמצעות ההתקן הזה אינפורמציה? שלילי. כדי להיות מסוגלים לשלוח אינפורמציה עלינו להיות מסוגלים להשפיע על איזה צד יפול המטבע שנפל ראשון וקבע את מצב המערכת. מכייוון שאיננו יכולים לעשות זאת, הצד עליו נפל המטבע הוא מבחינתינו אינפורמציה לא ידועה, ולכן איננו יכולים לשלוח אינפורמציה בדרך זו.

    רק על החלק השני, שליחת אינפורמציה, מדברים no-communication theorems. מה שאני מדבר עליו הוא החלק הראשון, עם הרדיו.

  154. ישראל
    אני מסכים איתך שזה לא אינטואיטיבי.
    אבל – עצם העובדה שאי אפשר להעביר מידע בצורה כזו, מראה שקורה כאן משהו שהוא לא “העברת מידע” קלאסי.
    זה נכון ששתי המטבעות תמיד נופלות על אותו הצד, אבל אין לנו שום דרך להשפיע על איזה צד…

  155. הרכבת נוסעת על מסילה עגולה כמו רכבת צעצוע וממלאת את כל המסילה וכמובן שפועל עליה כוח רדיאלי ותאוצה רדיאלית.

    אם הרדיוס קטן והמהירות גבוהה, היא תעוף מהפסים. אבל הרדיוס כה גדול שמבחינתה היא נוסעת כמעט על קו ישר.

    מה יקרה כשהיא תגיע לפקטור גאמא 2? זה מקרה זהה לשאלת הקרונות עליה דיברנו קודם.

    מקודם חשבנו שבין שני שעונים על המסילה ברגע נתון יהיו עכשיו כפול קרונות בגלל התקצרות האורך, אך זה בלתי אפשרי כי זה אומר שקרונות מסויימים יעברו את מהירות האור.

    אך לא זה מה שיקרה. יהיו בדיוק אותו מספר קרונות (דיברנו על מיליארד) אך יהיו ביניהם רווחים או שהם יקרעו, ראה פרדוקס בל.

  156. נראה לי שהבנתי מה קורה עם הקרונות.

    לא (קורה).

    הפיתרון הוא יחסיות הסימולטניות.

    כדי לראות זאת, נחשוב על מסילה הקפית ענקית שעליה נמצאת רכבת הקפית ענקית לא פחות. לכל יחידת אורך במסילה מתאימים x קרונות.

    ברגע מסויים הרכבת מתחילה להאיץ ומגיעה לפקטור גאמא שווה 2. בגלל הרדיוס העצום התאוצה הרדיאלית זניחה.

    ההגיון אומר שבגלל התקצרות האורך, כאשר תגיע הרכבת למהירות קבועה אז לכל יחידת אורך במסילה יתאימו עכשיו 2x קרונות, אך זה בלתי אפשרי כי נקבל כפול קרונות מקודם וזה בלתי אפשרי.

    מה שכן יקרה זה שבגלל יחסיות הסמולטניות הקרונות יתקצרו. אם הם נפרדים ובעלי הנעה עצמית, הם יפרדו זה מזה וייווצרו בינהם רווחים אך עדיין לכל יחידת אורך במסילה יתאימו x קרונות.

    אם הם מחוברים זה לזה כמו ברכבת רגילה, אז הם פשוט יקרעו ויתפרקו, כמו החבל בפרדוקס ספינות החלל של בל.

  157. נראה לי אולי שההסבר הוא אבירציה, דהיינו שבגלל התקצרות האורך יותר חלקיקים בעלי מטען חיובי משפיעים ויותר חזק. אני מנסה לראות אם זה יפתור גם את בעיית הקרונות, אבל בינתיים aloha

  158. אני ממליץ לך לשתות הרבה מימ.

    ואיני רואה מדוע מספר האלקטרונים צריך לגדול, כמו שבמערכת סגורה של משאבה וצינור כמות המים אינה גדלה.

    מה גם שהאפקט שציינת אינו יחסותי.

    בוידאו אגב אפשר ממש לספור את כמות המטענים (12 מכל סוג כאשר החתול במנוחה, 16 חיוביים ו8 שליליים כשהוא בתנועה).

    ומכיוון ששאלת מה ההסבר שלי (אין לי), אבל אוכל לנחש מה לסאז׳ היה אומר..(איפה יודה באמת?).

  159. ישראל
    תן המלצות – אני שם בעוד חודש.
    אני רוצה קודם להבין את מה שאלבנצו כתב… על פניו – נראה לי שלא כדאי לדון על על-מוליכות כי זו תופעה קוונטית.
    במקרה הקלאסי מספר האלקטרונים צריך לגדול כי אחרת אין זרם (חוק אום). אני לא בטוח שאני צודק, כי בס”כ אפשר ליצור זרם בתיל ע”י שדה מגנטי אחר (נניח – 2 טבעות רחוקות, כשמזיזים מגנט ליד טבעת אחת אז מושרה זרם גם בטבעת השנייה, ופה באמת לא מוסיפים אלקטרונים.

    יש הרבה מה להבין פה 🙂

  160. הוואי בייבי, הוואי.

    ניסים

    אתה יכול להשתמש בכא״מ זניח, אבל בשבילך ספיישל – טבעת בסביבות האפס המוחלט, הזרם לא יפסיק במשך כמה ימים גם ללא כא״מ.

    ולא הבנתי את עניין הצפיפות. הרי כל הפואנטה היא שהצפיפות ליחידת נפח (או אורך) עולה בגלל התכווצות האורך ויש יותר מטענים חיוביים משליליים ביחידת נפח ולכן נוצר כוח. רואים את זה בבירור בוידאו בסביבות הדקה השניה, שם זה מוקצן ומספר המטענים החיוביים ליחידת אורך כפול ממס המטענים השליליים, בזמן שמקודם הם היו מאוזנים.

    אז אם זה כך לאורך כל המעגל הסגור, מאיפה צצו פתאום כל כך הרבה מטענים חיוביים נוספים?

  161. אלבנצו
    תודה על ההסבר, למרות שרק את ההתחלה הבנתי. אני מבין את זה באנלוגיה הבאה: אם אני ליד מסילת רכבת ומרגיש כבידה מהרכבת, אז הכבידה הזו לא תגדל אם הרכבת תנוע במהירות רלטיוויסטית.
    הבנתי נכון?

  162. מספר האלקטרונים הוא לא הצפיפות שלהם. מספר האלקטרונים הוא האינטגרל של הצפיפות על פני הנפח (או השטח, או האורך. תלוי באיזו צפיפות מדובר, כמובן). אפשר לראות באופן כללי – מבלי להסתכל דווקא על טבעת או דיסקית או קוביה או תיל – שטרנספורמציית לורנץ על הצפיפות וטרנספורמציית לורנץ על מרחב מקזזות זו את זו, כך שהאינטגרל אינווריאנטי לטנרספורמציה. ברמה המתמטית זה נובע מכך שלא רק האינטגרנד עובר טרנספורמציה, אלא גם מידת האינטגרציה צריכה לקבל יעקוביאן ואפשר לראות באופן כללי שמטרצית היעקוביאן היא הופכית למטרצית הטרנספורמציה. לכן ממש לא משנה אם טרנספורמציה מסוימת שהופכת מעגל לאליפסה או ריבוע למלבן או וואטאבר נראית מוזר כשמסתכלים על הצפיפות שלה. אם היא קונסיסטנטית עם טרנספורמציית לורנץ, מספר החלקיקים לא ישתנה. כמובן שכל זה נכון רק ביחסות פרטית בה המרחב שטוח והצופים כולם מקושרים זה לזה במהירות קבועה. אם המרחב אינו שטוח או שחלק מהצופים מאיצים, צריך להסתכל על יחסות כללית ושם הסיפור אחר לגמרי (מספר החלקיקים בהחלט אינו קבוע, ואפילו רואים זאת בניסוי).

  163. ישראל
    כדי לגרום לזרם בתיל צריך מתח לאורך התיל. המשמעות היא שיש עודף אלקטרונים בתיל (זה לא שונה מהזרמת אוויר בצינור פנאומטי).

  164. על המטוס, בדרך להוואי, עוד 5 דקות ממריאים.

    אז אין מצב שקרון המרוחק שעת אור יגיע לצילום בתוך 10 שניות.

    ובנוגע לאלקטרונים המצטופפים: אם תחשוב על טבעת שבה זורם זרם חשמלי, אז איך יכולה צפיפות האלקטרונים לעלות בנקודה כלשהי על הטבעת בלי שתרד בנקודה אחרת? הרי מספר האלקטרונים קבוע על הטבעת..

    כפי שאמרתי, מוזר ביותר.

  165. עזוב רגע פוטונים, בוא נגמור את נושא הקרונות.

    אם כשהתחלת היו לך קרונות מול נקודות 1 ו2 שמפרידים בינהם מיליארד קרונות, וכשסיימת קיבלת קרונות מול אותן נקודות שמפרידים בינהם 2 מיליארד קרונות, אז מאיפה שהוא הגיעו קרונות שלפני ההאצה היו רחוקים משתי הנקודות לפחות שעת אור. את זה אתה מקבל? אין לזה שום קשר ליחסות.

    אם לא – הסבר לי איפה הם היו קודם.

  166. ישראל
    בוא נחשוב על מקרה פשוט. יש 3 נקודות על המסילה, א מצד שמאל שלי במרחק שנת אור, ב מולי, וג במרחק שנת מצד ימין. ברגע 0 יוצא פוטון מא וגם מב יוצא פוטון לכיוון ימין. אחרי שנה, הפוטון מא יגיע אלי והפוטון מב יגיע לג. כלומר, המרחק בין הפוטונים נשאר שנת אור.

  167. ישראל
    כתבתי לך קח:
    “שראל
    בוא נתעלם תחילה מההאצה. בשלב ראשון מול נקודה 1 נמצא קרון 0 ומול נקודה 2 קרון מיליארד.
    עכשיו – בוא נניח שרכבת שנייה נעה במהירות קבועה של גאמא=2. במצב זה, אם מול נקודה 1 נמצא קרון x אז מול נקודה 2 נראה את קרון x פלוס מיליארד כפול 2.”

    הבעיה שלא כתבתי “מספר הקרון הוא x פלוס מיליארד כפול 2” ? אתה ילד בגן?

  168. ניסים אתה קרוא לסדר.

    מה כל כך קשה בלענות משהו בסגנון ״מס הקרון הוא x + שני מיליארד?

    לא משנה, אני מניח שזו תשובתך, אבל הנה הבעיה: קרון מס x + שני מיליארד היה מרוחק ממצלמה 2 כשעת אור לפני שהרכבת התחילה להאיץ.

    מכייוון שכל התהליך אורך רק 10 שניות (מזמן 0 לזמן 10) אין לו שום אפשרות להגיע למצלמה 2 בזמן.

    ולא משנה איך תשחק עם הנתונים, אם הגעת למצב שבתוך 10 שניות מתחילת ההאצה יש מול 2 המצלמות באותו הרגע בשניהן קרונות ש2 מיליארד קרונות מפרידים בינהם, אז לאחד הצדדים הגיע תוך 10 שניות קרון שהיה מרוחק ממנו שעת אור.

    אתה רואה אפשרות אחרת?

  169. ישראל
    כבר עניתי לך. אם מהירות הרכבת גאמא=2, והיו מיליארד קרונות במהירות 0, אז יהיו 2 מיליארד קרונות.

  170. תואם, תואם

    אבל לא קשור לשאלה שלי. גם תאוצות לא. השאלה היא: איזה קרון יצולם עיי מצלמה 2 בזמן 10? היא תצלם איזה שהוא קרון, לא?

    הרכבת כבר הגיעה מזמן לשיווי משקל והיא כבר מערכת אינרציאלית הנעה במהירות קבועה של 0.87c, פחות מהמיואונים שלך וגם לקח לה הרבה יותר זמן להאיץ מהם.

    אז מה מספר הקרון? אינך חייב לדייק אלא לתת הערכה הגיונית כי אחרי הכל, מהם חמשים-שישים מיליון קרונות בין חברים?

  171. ישראל
    מה שאני כתבתי נכון או לא? האצות זה עניין מסובך, אז בוא נסגור על מהירות קבועה. האם מה שאמרתי תואם את תורת היחסות הפרטית?

  172. בוא נעשה את זה פשוט:

    ברגע 10 בנקודה 1 המצלמה חדת הרזולוציה שבנקודה 1 מצלמת את קרון x ברכבת שממולה.

    מה מס הקרון שתצלם המצלמה בנקודה 2 ברגע 10?

    וכדי להוסיף חטא על פשע, אל תשכח שעל פי יחסות הסימולטניות הרכבת דווקא מתארכת, לא מתקצרת (זוכר את פרדוקס הספינות של בל? אבל גם התארכות אינה פותרת את הבעיה שלפנינו).

    יהיה אשר יהיה הפיתרון, ואני כרגע לא מוצא כזה, הוא מוזר ביותר.

    וזו הנקודה שהעלתי בנוגע להתקצרות האורך, היא מוזרה מאוד.

    אז נסה לאמר מה יהיה מספר הקרון בערך מול מצלמה 2 ברגע 10, וזה בלי שדיברנו על האם אי לוקליות לא ממוטטת את היחסות כטענת המאמר בסיינטיפיק אמריקן, לא בפיזיקה אלטרנטיבית.

  173. ישראל
    בוא נתעלם תחילה מההאצה. בשלב ראשון מול נקודה 1 נמצא קרון 0 ומול נקודה 2 קרון מיליארד.
    עכשיו – בוא נניח שרכבת שנייה נעה במהירות קבועה של גאמא=2. במצב זה, אם מול נקודה 1 נמצא קרון x אז מול נקודה 2 נראה את קרון x פלוס מיליארד כפול 2.

    לזה אני מסכים.

  174. נו ניסים, באמת.

    1. ברגע 0 בשעוני המסילה, כשהרכבת במנוחה, נמצא קרון מס 1 מול נקודה מס 1 במסילה וקרון מס מיליארד מול נקודה מס 2 במסילה המרוחקת שעת אור מנקודה מס 1.

    2. הרכבת מאיצה בחמש שניות על פי שעוני הרכבת כך שגאמא שווה ל2.

    3. ועכשיו אמור לי אתה: אם ברגע 10 שניות על פי שעון נקודה 1 במסילה נמצא מולו קרון מס x, מה יהיה מס הקרון שניצב ברגע 10 במסילה מול נקודה 2? לא x + 2 מיליארד?

  175. ישראל
    אוקי, אתה אומר שהרכבת התחילה בעמידה, והאיצה תוך שניות לגאמא=2. כלומר, מ-0 ל-260 מיליון מטר לשנייה. מבחינת צופה מהצד, 2 קצוות הרכבות נעות במהירויות שונות, הקרונות האחרונות יותר מהירות, בהרבה, מהקידמיות.

    שוב – אני באמת לא רואה כאן סתירה.

  176. הכוונה היא שאיינשטיין לא טעה בכך שששזירה סותרת את היחסות בגלל שפירושה תקשורת במהירות על אורית, כפי שכתוב בויקיפדיה.

  177. ניסים

    אתה מפספס את הנקודה.

    אם בתוך שניות בודדות בשעוני המסילה שהם גם שניות בודדות בשעוני הרכבת הרכבת מתקצרת בחצי, אז אם תיקח שתי נקודות על המסילה ותיקח זמן מסויים בשעוני המסילה (נאמר 0) אז בין אותן שתי נקודות כלואים בהתחלה מספר מסויים של קרונות (נאמר מיליארד) ואחרי כמה שניות בשעוני המסילה (נאמר 10 שניות) 2 מיליארד.

    תוכל לראות זאת בבירור אם הקרונות ממוספרים: ברגע 0 נמצא מול נקודה 1 במסילה קרון 1 וברגע 0 בנקודה 2 במסילה קרון מיליארד.

    ברגע 10 בנקודה 1 במסילה ימצא מולה קרון x וברגע 10 בנקודה 2 במסילה קרון x + שני מיליארד.

    מאיפה נוספו כל הקרונות בזמן כה קצר? רק את זה.

    Elbentzo.

    אין לי כוונה להתדיין אתך לפני שתתנצל על כל קיתונות החרה ששפכת עלי בשנים האחרונות (זוכר? שקרן, אידיוט, מטומטם, בור, סנילי וכו). וזה רק תנאי ראשון.

    ההסטוריה העגומה מוכיחה שאתה מיד מתרתח כאשר לא מסכימים לדעתך, אז מה הטעם בדיון? להכנס לעוד ריב?

    ולא, איינשטיין לא טעה במאמר EPR, אבל אין לי כוונה להכנס לריב עוד הפעם. מחר נוסעים להוואי, חבל על העצבים.

  178. לקחתי עכשיו כמה דקות לקרוא את המאמר מסיינטיפיק וגם לעשות טיפה מחקר על המקורות אליהם הוא מפנה.

    קודם כל, צריך לזכור שכל מאמר במגזין – גם מדעי, ובטח שבסיינטיפיק אמריקן שפונה לקהל הרחב ויש לו אינטרס ענק למכור עותקים – יכול ליפול לנטייה טבעית לכותרות סנסציוניות או בומבסטיות. זה קורה גם במאמרים מדעיים. ככה שאין טעם להסתכל רק על הכותרת, וצריך לפנות לגוף המאמר.

    הדבר הראשון שקפץ לי לעין, זה ששני הכותבים הם פילוסוף (אמנם פילוסוף של המדע, אך בכל מקרה לא מתמטיקאי או פיזיקאי) וסופרת. ישר מההתחלה ברור שזה לא בהכרח מקור טוב ללמוד ממנו פיזיקה. אקדים את המאוחר ואומר שאני לא טוען שהם לא יודעים על מה הם מדברים, מפני שרוב המאמר נוגע לפילוסופיה ולא לפיזיקה, ולכן אין שום בעיה. אבל אם בפיזיקה עסקינן, המאמר הזה לא צריך להיות יותר מאשר תמריץ לקרוא מאמרים ומחקרים אמיתיים של פיזיקאים.

    עכשיו למאמר עצמו. החלק הראשון של המאמר (בערך 75%, לא ספרתי עמודים) רק עושה סקירה היסטורית של הרקע של שזירה ויחסות פרטית ומסתיים בקונצנזוס המלא של הפיזיקאים לכך שאין סתירה בין יחסות פרטית למכניקת הקוונטים. אחרי חלק זה, מגיע החלק השני ובו שלושה חלקים:

    1. טענה שפילוסוף בשם מודלין כתב ספר ב-94′ שבו הצביע על בעיה (לא סתירה כאם נקודה עדינה) בשילוב של יחסות פרטית ומכניקה קוונטית. חוץ מזה שזהו ספר שאינו מדעי ולא עבר ביקורת עמיתים, הבעיה לא מוסברת בדיוק. יש הסבר קצר של מה לא הבעיה, ואז הבעיה מנוסחת בדרישה “לסימולטיניות אבסולוטית”. לי אישית לא ברור איך זה עולה, אבל לא קראתי את הספר.

    2. התייחסות לעבודה של מתמטיקאי (הציטוט שניסים הביא מקודם) שעבור תורות חופשיות מראה באופן ריגורוזי יותר מפון-נוימן שאין בעיה בשילוב של שזירה עם יחסות פרטית.

    3. התייחסות לעבודה של אחד הכותבים. העבודה לא מוסברת באופן מפורט אך מהמעט שכן מוסבר ברור שהיא הולכת על חבל מאוד דק ומסוכן, שזה התייחסות לגיאומטריה הטריוויאלית של יחסות פרטית. יחסות פרטית היא תורה גיאומטרית שכל סטיה ממנה, ולו הקטנה שבקטנות, מחייבת שימוש ביחסות כללית. יחסות כללית היא תורה שבאופן מפורש אינה קוונטית (בניגוד ליחסות פרטית). לכן שילוב של מכניקת קוונטים ויחסות פרטית יכולות להתבצע רק תחת תנאים של גיאומטריה קבועה שאינה עושה אינטראקציה או כל פלקטואציה. כלומר, יש פה דברים מאוד עדינים שגם הרבה פיזיקאים שאינם חוקרים כבידה יכולים לטעות בהם. אבל גם אם הכל נעשה כשורה, הבעיה שהוא מציג היא נטו בעיה פילוסופית שיכולה להפתר על ידי קבלת מרחב הקונפיגורציות כמרחב פיזיקלי. במילים אחרות, להבין שההסטוריה שלנו מצויה במרחב הילברט ולא במרחב spacetime.

    לסיכום, קשה לי מאוד מאוד מאוד לראות איך בתוכן הכתבה יש איזהשהו סיכון לשילוב של יחסות פרטית ומכניקת הקוונטים. שילוב שנעשה באופן מפורש בתורות שדה קוונטיות, שהן נמצאות קונסיסטנטיות מתמטיות (בקונטקסט הזה) כבר בערך 60 שנה ושרמת הדיוק בניסוי שלהם מגיעה לעיתים למיליוניות האחוז. לא הייתי מתייחס לכותרת יותר מדי ברצינות.

  179. ישראל
    בוא נניח שכל קרונות הרכבת מתחילות להאיץ בשעה t, לפי שעון הרכבת (אין בעיה לסנכרן את השעונים כך). מבחינת צופה על הקרקע – הקרונות לא מתחילות להאיץ באותו זמן. בפרט – צופה על הקרקע יראה את הקרון האחרון מאיץ לפני שהקרון הראשון האיץ.
    לכן – מוזר, אבל אין כאן סתירה.

    אנסה לפשט את המקרה כדי להבהיר. בוא נדמיין שיש רק שתי קרונות מרוחקות מחוברות במוט קשיח, באורך 2 שניות אור, והרכבת נעה במהירות v. במרכז המוט יש מבזק – וכל קרון מאיץ ברגע שהוא רואה את ההבזק. המבזק מופעל בזמן t. מבחינת הקרונות – האור מגיע לקרונות בזמן t+1, הן מתחילות לנוע ביחד והמרחק ביניהם נשאר 2 שניות אור.

    מבחינת צופה על הקרקע. בוא נניח שהצופה רואה את ההבזק גם בזמן t (בה”כ….). מבחינתו האור מגיע לקרון האחורי בזמן (c+v)/1 ולקרון הקידמי בזמן (c-v)/1. לכן, הקרון האחורי האיץ קודם ולכן הרכבת נראית קצרה יותר.

  180. ישראל,

    על השאלה האחרונה שלך כבר קיבלת תשובה ממני בעבר, אבל זה היה מזמן אז אולי אתה לא זוכר.

    למרות שאתה לא רוצה לחשוב על כך, איינשטיין טעה. יש הרבה דברים לא לוקאליים בעולם, והרבה דברים שנעים מהר מהאור. לא כולם סותרים את תורת היחסות הפרטית. למשל, אנרגיה של מערכת היא גודל שאינו לוקאלי. דוגמא שנותנים תמיד בתואר ראשון היא פנס שמאיר על מסך מרוחק ממנו מאוד. אז מסיטים את הפנס שמאלה בעשרים מעלות, והזזה זו לוקחת נגיד חצי שניה. אם המסך מרוחק מספיק אז כתם האור ינוע מרחק של יותר מחצי שניית אור, כלומר יזוז מהר יותר מהאור.

    איינשטיין חשב ששינוי מצב קוונטי על פני מרחק מתווך כנראה על ידי חלקיק ולכן הוא ממשפחת הדברים שכן עומדים בסתירה ליחסות. מה לעשות, גם איינשטיין טועה. ואפילו המון. אתה אולי יודע שאיינשטיין היה מפרסם מאמר כל שבוע או שבועיים, ולחצי מהם היה מפרסם אח”כ התנצלות ותיקון כי הוא גילה שטעה. היום אנו יודעים שמכניקת הקוונטים למעשה אינה מדויקת, והאובייקטים היסודיים בעולם הקוונטים אינם חלקיקים אלא שדות. שדות מאפשרים קורלציה שאינה מתווכת על ידי חלקיקים ויכולה להיות לא-לוקאלית מבלי לסתור שום דבר בתורת היחסות הפרטית.

    לא נראה לי שזה ישכנע אותך, אבל אם אתה באמת רוצה להכנס לעובי הקורה – לא כדאי להתעלם מהקונצנזוס המלא שיש היום (אחרי יותר מ-80 שנה של אנשים חכמים כמו איינשטיין שגילו הרבה מאוד דברים שלא היה לו מושג לגביהם) בנוגע לנושא.

  181. חחח.. אז נטפלת למאמר שולי ודלגת על העיקר?

    איינשטיין אומר שאם נבצע ניסוי מסויים נקבל תוצאות קונסיסטנטיות עם היחסות ולכן תיאוריית הקוואנטים שגויה.

    הניסוי מבוצע. קוואנטים נכונה. אז מה עם היחסות?

    ובוא נרד מזה שהוא לא הבין את משמעות הניסוי. הוא איינשטיין והוא הגה אותו.

  182. חשבתי קצת על ההסבר היחסותי למשיכה בין הכבלים המוליכים. לכאורה זהו סלאם דאנק, אך אם חושבים קצת לעומק, קבועי החשמל והמגנטיות כבר מגולמים בתוך מהירות האור והטיפול המתמטי היחסותי רק חילץ אותם משם.

    וזאת בניגוד למודל מקסוול שלקח 3 קבועים שלכאורה אין קשר ביניהם – קבועי החשמל, המגנטיות ומהירות האור – וקישר אותם בנוסחה אלגנטית. זה סלאם דאנק! (לא סלאם ג׳נק!).

    אך ההסבר היחסותי ממחיש את מה שאמרתי על התקצרות האורך: היא מוזרה מאוד.

    כדי לראות זאת, חשוב על ההסבר: בגלל תנועת האלקטרונים עולה צפיפות המטענים.

    אז בוא נחשוב על רכבת ארוכה מאוד הנמצאת על מסילה ארוכה עוד יותר. ברגע מסויים בשעוני הרכבת כל הקרונות מאיצים בתוך כמה שניות על פי שעוני המסילה כך שפקטור גאמא שווה ל2.

    על פי ההסבר היחסותי, הרכבת מתקצרת וצפיפות הקרונות מוכפלת.

    אך משתמע מכך שאם ניקח שתי נקודות על המסילה במרחק שעת אור זו מזו שביניהן היו מספר מסויים של קרונות, אז בתוך שניות ספורות יהיו כפול קרונות, לא?

    מאיפה נוספו בזמן כל כך קצר כל כך הרבה קרונות??

    כפי שאמרתי, מוזר מאוד.

  183. ישראל
    אם לא הבנת, אני מאלה שמשתדל לא להמציא תיאוריות חילופיות לדברים שאני לא מבין בהם.

    הדברים הטכניים התפלים האלה, הם בדיוק מה שמפיל הרבה תיאוריות חילופיות – והמאמר שלך על ה-GPS זה רק דוגמה. החיכוך בפושינג גרוויטי הם עוד פרט טכני תפל,שפוסל את כל הרעיון.

    אני לא מבין מספיק בפיסיקה, או בפסיכולוגיה, בשביל להסביר למה מישהו נתן כותרת מסויימת למאמר, במיוחד שתוכן המאמר לא תואם ב-100% לכותרת.

    אני לא פוסל דיון בנושאים האלה, ואני לא אומר שאסור לבוא עם רעיון משוגע. אבל זה טוב לתרגיל אקדמי בלבד ולא צריך לערבב את זה עם מדע אמיתי.

    מצד אחד, אתה מביא כל מיני סיבות לפסול תיאוריה, ומצד שני, כל פעם שמנסים להסביר למה הסיבה שגויה – זה רק פרט טכני טפל. המאמר על ה-GPS זה דוגמה טובה לכך – “אל תבלבל אותי עם עובדות”.

  184. ניסים

    סתם עשיתי גוגול.. המאמר שקישרתי איליו קודם הוא של סיינטיפיק אמריקן וכותרתו: איום קוואנטי על תורת היחסות הפרטית.

    אז מה התאוריה שלך ללמה אתה לא מתייחס לכותרת המאמר אלא נטפל לפרטים טכניים תפלים?

  185. ישראל
    המאמר שקישרת אליו הוא מעניין. הוא גם שגוי. יש סיבה שכתוב שם alternate physics…

    זה מדגיש מה הבעיה בכל התיאוריות החלופיות – שלך, של יהודה ואפילו של ידידי רפאל: חוסר ידע. כשמישהו שמבין אומר משהו שאתה לא מבין (אלבנצו לדוגמה) אז אפשר להמציא תיאוריה חלופית, או אפשר לפתוח ספר וללמוד.

    אם זה מעניין – אני יכול להסביר את הטעות במאמר שקישרת אליו….

  186. רפאל,

    1. זה שאנחנו יודעים בוודאות מאוד גבוהה שמשהו לא יכול להיות נכון כי הוא סותר את מכניקת הקוונטים (שהיא למיטב ידיעתנו תיאור מאוד טוב של המציאות), לא אומר שאנחנו יודעים מה כן נכון. אם אתה רוצה תובנות לשאלה מה כן יתכן שקרה ביקום המוקדם מאוד בהתאם לתורות כבידה קוונטיות, אתה יכול לחזור לסרטון שאתה בעצמך הבאת כאן לפני כמה שנים בו פיזיקאים מאוד מוכשרים הסבירו מה הרעיונות שלהם וכיצד ניתן לבדוק אותם. נכון להיום אין לנו תשובה חד משמעית לשאלה מה קרה ביקום המוקדם בהתחשב במכניקת הקוונטים.

    2. נתתי דוגמא לפונ’ סינגולרית כי הסברתי לניסים על סינגולריות כנושא כללי במתמטיקה ולא בקונטקסט של המפץ. לא אמרתי שפונ’ זו מתארת את הסקאלה של המפץ הגדול או שהיא בכלל קשורה למודל. אפילו לא אמרתי שהסקאלה של המפץ הגדול היא פונ’ סינגולרית. אמרתי שההעתקה הקונפורמית היא סינגולרית בנקודה a=0 כי היא מנוונת ומעתיקה את כל נקודות המרחב לנקודה אחת. אני לא רוצה להשמע מתנשא או מזלזל, אבל אם אתה רוצה להבין את הפן הטכני של מודל המפץ הגדול (או כל מודל אחר של פיזיקה מודרנית), אתה חייב ללמוד. יש הרבה דברים שניתן להסביר פחות או יותר בשפה של מדע פופולרי אבל הבנה טכנית מחייבת ידע של המתמטיקה שעומדת מאחורי המודל.

  187. אלבנצו

    סליחה שאני חוזר קצת אחורה אבל עדין לא הבנתי את זה:

    1. כתבת “יקום בגודל 0 הוא לא הכרחי למודל וסביר להניח (למעשה, די בטוח) שהוא לא יתקבל כפתרון של תורה קוונטית מסיבות של לוקאליזציה. ”

    מה זה בעצם אומר? שתמיד היה שם משהו רק שהוא התחיל להתרחב בזמן מסויים?

    2. לגבי זה “דוגמה טריוויאלית לפונ’ שהיא סינגולרית באמת היא אחד חלקי z. יש לה סינגולריות בנקודה z=0.”

    סינגולריות אמורה לתת סקאלה אפס. 1/0 לא נותן אפס. איך יש כאן סינגולריות? (השמטתי את הנושא של אינסוף כי מקודם הוא גרם להסיט את כל תשומת הלב מן העיקר)

  188. ישראל
    לא הבנתי מה אתה אומר על המיואונים. האטת הזמן נבדקה בשני הכיוונים – בלוויני GPS למשל.

    שוב, במאמר שלך כתוב שיש הסבר אפשרי ל-EPR שלא סותר את תורת היחסות.

    אני לא אומר שתורה זו או אחרת נכונה. אני כן אומר שיש ראיות להתקצרות המרחק.

  189. ניסים

    האם קראת אתה את הפסקה האחונה במאמר?

    The status of special relativity, just more than
    a century after it was presented to the world, is
    suddenly a radically open and rapidly developing
    question. This situation has come about because
    physicists and philosophers have finally followed
    through on the loose ends of Einstein’s longneglected
    argument with quantum mechanics

    אני יכול להתחיל לנסות ולהסביר כל שאלה שאתה שואל – לדוגמה המיואונים אותם הזכרת הואצו ונעים כנגד קרינת הרקע, בעוד שעל פי היחסות גם במצב ההפוך, דהיינו אילו הם היו נייחים יחסית לקרינת הרקע גם אז היה שעונם הפנימי מטקטק לאט יותר יחסית למערכת מסונכרנת שנמצאת בתנועה יחסית לקרינה וזה לא הודגם בניסוי מעולם.

    אני יכול גם להצביע על זה שגם תורת ניוטון עובדת ונלמדת והוכחה כנכונה ומדוייקת במשך מאות שנים, ולמרות זאת היא אינה נכונה אלא מקרה פרטי של היחסות במהירויות נמוכות.

    אבל האמת היא שמאיפה אני או מישהו יודע. אנחנו גם לא אמורים לדעת. על דבר אחד שנינו מסכימים אני מאמין: איינשטיין כותב מאמר – EPR – שבו הוא מנסה להראות כי מכניקת הקוואנטים שגויה או לא שלמה כי (על פי ויקי לפחות) היא עומדת בסתירה ליחסות. הוא גם מציע ניסוי מחשבתי להוכיח את טענתו.

    הניסוי נערך במציאות (שמעוני, אספקט), והתוצאות קונסיסטנטיות עם מכניקת הקוואנטים.

    אז מה זה אומר על היחסות?

  190. ישראל
    ציטוט מהמאמר שקישרת אליו:
    The first result appeared in an astonishing
    2006 paper by Roderich Tumulka, a young German
    mathematician now at Rutgers. Tumulka
    showed how all the empirical predictions of
    quantum mechanics for entangled pairs of particles
    could be reproduced by a clever modification
    of the GRW theory (recall that this theory
    proposes a philosophically realist way to get the
    predictions of quantum mechanics under many
    circumstances). The modification is nonlocal,
    and yet it is fully compatible with the spacetime
    geometry of special relativity.

    קרא שוב את המשפט האחרון….

    אבל עזוב – תסביר לי בבקשה איך מיואון עובר 6000 מטר ב-2 מיקרו, בלי התקצרות המרחק.

  191. ישראל
    אז בוא נתחיל
    1) בקשר להתקצרות. בוא נניח שהגודל של הכוכב הוא כזה שיכול להתקיים ליקוי מדוייק של של הכוכב ע”י הירח. כך נקודה על ההיקף של הירח ואת הנקודה המתאימה על הכוכב המרוחק. נקודות אלה הן על קו ישר מאיתנו, נניח בזווית קונית a. בכל מהירות v של הצופה (לכיוון מרכז הירח/כוכב) , המהירות לשתי נקודות אלה היא v כפול קוסינוס a. המשמעות היא שההמרחק לשתי הנקודות יתקצר באותו מקדם, ולכן הן תשארנה על אותו הקו. כדי להמחיש את זה – צייר את המצב על משטח פלסטיק, ותמתח את הפלסטיק לאורך ציר אחד. המתיחה היא טרנספורמציה אפינית, שאחת מהתכונות שלה היא שנקודות על קו ישר תשארנה על קו ישר.

    2) מה חדש? אם יש לנו שתי תורות שסותרות, אחת פשוטה ואחת מסובכת להחריד, אז יותר סביר שהמסובכת היא השגויה. בתקווה, נוכל למצוא תיאוריה שמשלבת את התצפיות משני התחומים.

    3) החישוב של מקסוול למהירות האור מתבסס על שני קבועים (אפסילון-0 ומיו-0) שאין לו כל הסבר להם. לפי מקסוול מהירות האור צרירה להיות תלויה במהירות “המוחלטת” של מקור האור, ולזה אין בסיס תצפיתי.
    גם משוואות ניוטון עובדות בתנאים מסויימים. …

    4) אנחנו יודעים שיש מערכת יחוס מועדפת ליקום, וזה בדיוק הטמפרטורה שאתה מדבר אליו. זה לא סותר את תורת היחסות.

    5) מאד בוגר מצידך לצטט כהוכחה משפט של אדם שאתה לא מסכים עם דעתו 🙂 איינשטיין האמין בלוקליות, והתצפיות מראות שהוא טעה.

    ישראל – יש לך הסבר אחר לכך שתילים מוליכים נמשכים אחד לשני? ובוא אתן לך עוד דוגמה.

    מיואונים נוצרים מהתנגשויות של קרינה קוסמית באטמסופירה העליונה (כ-20,000 רגל), וזמן החיים הממוצע שלהם כ-2 מיקרושניות. המהירות שלהם היא בערך גאמה = 10. מבחינתינו – הם חיים 20 מיקרו, ויכולים לנוע 6000 מטר, שזה אומר שנוכל לגלות חלק מהם על הקרקע.
    אבל – מבחינתם, הם חיים רק 2 מיקרו! איך הם עוברים מרחק של 6000 מטר?? המממממ …. מה יקרה עם נחלק את המרחק ב-10? נקבל 600 מטר 🙂 והנה – כמחציתם אכן יגיעו לאדמת הקודש.

    אם אתה לא מסכים להסבר – מה ההסבר שלך?

  192. ישראל
    ויכול להיות שכל אלקטרון מושיט יד לאלקטרון בתיל השני, וכמובן שהוא יכול לעשות את זה רק כששני האלקטרונים נעים באותו כיוון.

    החיזוי היחסותי להתקצרות האורך בתורת היחסות הפרטית היא תוצאה ישירה לתיאוריה מאד מוצלחת, לא סותרת שום תצפית קיימת, ואפילו קלה להבנה. ויש גם תצפיות שמאששות את החיזוי.

    למה לא לקבל את זה? התקצרות הזמן ועיקום המרחב כן, ודווקא התקצרות באורך לא?

    כולנו יודעים שתורת היחסות ותורת הקוונטים לא משחקים ביניהם יפה, אבל למה דווקא התקצרות האורך מפריעה לך?

  193. QED היא תורה יחסותית וכוללת בתוכה את יחסות פרטית כולה, כולל התקצרויות/התארכויות למיניהן. אולי אני מפספס את הפואנטה.

    וסתם לידע כללי – ניסים, ההוכחה של סכימת הטור הטבעיים למינוס אחד חלקי שתיים עשרה בעזרת מניפולציות של טורים שאינם מתכנסים בהחלט (כמו למשל 1-1+1-1+…) היא היוריסטית ונחמדה אבל נחשבת במתמטיקה קצת להוכחת צעצוע. בסטנדרטים מודרניים היא לא תקרא הוכחה, בגלל שהיא יוצאת מנקודת הנחה שהאלגברה של הטורים (שאינם מתכנסים בהחלט) מוגדרת היטב וזה פשוט לא נכון. יש כמובן הוכחות ריגורוזיות שמביאות לאותה תשובה שאפשר להגיע אליה בפשטות כפי שהצעת. למשל, על ידי המשכה אנאליטית של הטור וביצוע רגולריזציה בעזרת פונ’ זטא של רימן.

  194. אתה אולי מתכוון לזה:

    http://galileo.phys.virginia.edu/classes/252/rel_el_mag.html

    יפה ומרשים. אך השאלה שלנו הייתה: האם זה ההסבר היחיד הבא בחשבון?

    הנה מויקי:

    Quantum electrodynamics[edit]
    See also: Standard Model and quantum electrodynamics
    In modern physics, the electromagnetic field is understood to be not a classical field, but rather a quantum field; it is represented not as a vector of three numbers at each point, but as a vector of three quantum operators at each point. The most accurate modern description of the electromagnetic interaction (and much else) is quantum electrodynamics (QED),[37] which is incorporated into a more complete theory known as the Standard Model of particle physics.
    In QED, the magnitude of the electromagnetic interactions between charged particles (and their antiparticles) is computed using perturbation theory. These rather complex formulas produce a remarkable pictorial representation as Feynman diagrams in which virtual photons are exchanged.

  195. הנוסחאות הן לב העניין.

    אנחנו זזים לקבלת שבת, בינתיים במטותא נסה לחשב את פקטור גאמא כשv קטנה ממטר בשניה (מהירות האלקטרונים במוליך).

    אם חסרים לך דפים בשביל האפסים אחרי הנקודה העשרונית, תוכל למצוא כמה יערות חסרי שימוש בקנדה הסמוכה.

  196. ישראל
    תתיחס לנושא המגנטיות. האם אתה יכול להסביר בצורה אחרת משיכה בין תילים מוליכים מקבילים?
    זה מוסבר יפה בספר של ברקלי – והנה וידיאו קצר שמסביר, בלי הנוסחאות ….

  197. ישראל
    מויקי:
    the behavior of colliding heavy ions can only be explained if their increased density due to Lorentz contraction is considered. Contraction also leads to an increase of the intensity of the Coulomb field perpendicular to the direction of motion, whose effects already have been observed. Consequently, both time dilation and length contraction must be considered when conducting experiments in particle accelerators.

    התקצרות אורך גם מסביר למה שני מוליכים מקבילים נמשכים אחד לשני.

  198. ניסים

    על פי הלינק לאפקט סניאק

    https://en.wikipedia.org/wiki/Sagnac_effect#Relativistic_derivation_of_Sagnac_formula

    ניתן כמו תמיד להסביר אותו גם כהתקצרות האורך וגם כהתארכות הזמנים.

    איפה ההסבר היחיד למסלולים של יונים לאחר התנגשות? לא ראיתי בלינק שהבאת.

    אם תביא מושג מתמטי עם הסבר הגיוני, אקבלו בשמחה. אך לאמר ש 1+2+3+4+5+….. = 1/12-, ואז להביא לינק שבו לא כתוב כלום, זה מגוחך. אולי אפילו מגוכך.

  199. ישראל
    שים לב למילה direct. אין לנו אפשרות טכנית לבדוק את השינוי באורך של גוף מהיר. אבל זה ההסבר היחיד למסלולים של יונים לאחר התנגשות. זה מה שאומר ויקיפדיה. בכלל, אם תחשוב על זה, אורך מאד קשה למדידה ממערכת תנועה אחרת.

    במטוסים (לפחות מטוסי קרב) יש מערכת שנקראת RLG. המערכת הזו מסתמכת על תופעת סאניאק, שהיא תוצאה של שינוי בארוך מסלולו זל לייזר עקב תנועה מעגלית. בנוסחאות שמסבירות את התופעה מכניסים את שינוי האורך עקב התנועה.

    אני לא מבין למה אתה מזלזל במושג מתמטי. גם שורש מינוס 1 הוא מגוכך, אבל משתמשים בו הרבה בפיסיקה, ואפילו בחשמל.

  200. ניסים

    בלינק על ההתכנסות אין שום הסבר או הוכחה, רק טענה והרבה טענות נגד שזה סתם בולשיט.

    גם בלינק על התקצרות האורך לא מצאתי שום דבר על יונים או על שובך ושום עדות להתקצרות האורך. לעומת זאת בלינק הבא:

    http://math.ucr.edu/home/baez/physics/Relativity/SR/experiments.html#Length_Contraction

    כתוב בפירוש:

    At this time there are no direct tests of length contraction

  201. תחליט, 1/12- או 1/12?

    לא זה ולא זה.

    רעיון שמעיון, סזרו מפוזרו, אם תשים כסף על זה שהטור יתכנס לחצי, תפסיד. גם אם תשים על הצב בתחרות ריצה עם אכילס או על רוזאן באר בתחרות יופי עם סנדי באר.

    איפה הלינק מויקי על היונים?

  202. ישראל
    לא צריך להוסיף כלום …. הטור אינסופי וערכו מינוס 1/12… ההוכחה פשוטה, אבל היא מסתמכת על ההתכנסות שהזכרתי(1-1+1-1+1-1+1-1+1 = 0.5). הרעיון של סזרו הוא להסתכל על הגבול של הערך הממוצע.

    וכן – יש ניסויים שמראים את התקצרות האורך. אם מסתכלים על התנגשויות של יונים גדולים אז הדרך היחידה להסביר את זוויות החלקיקים היא ע”י התקצרות היונים. ראה ויקיפדיה….

  203. 1+2+3+4+5+….. = 1/12- רק אם נוסיף בצד שמאל של המשוואה איבר חיובי, או איבר שלילי בצד ימין איפה שהנקודות.

    יחסות – אתה מכיר איזה אישור חד משמעי להתקצרות האורך כמו ניסוי המטוסים מקיפי הארץ שהדגימו את התארכות הזמן?

  204. ישראל
    אלבנצו יכול להסביר לך למה מבחינה פיסית 1+2+3+4+5+….. = 1/12-
    לא אינטואיטיבי אבל זה כתוב בספר פיסיקה (של פולצ’ינסקי) (חשבתי שבעברית המילה המתאימה היא “הגיון”, אבל אתה צודק שלזה התכוונתי).

    אני יכול להסביר לך מבחינה מתמטית למה זה נכון ….

    ובקשר לרכבת – מבחינת הנוסעים, הרכבת לא התקצרה. זה נכון רק לצופה חיצוני.

  205. ניסים

    זה מה שאמר לייבניץ ואולי פילוסופית זה נכון. פיזית, תמיד תקבל אחד או שתיים (כמו בקוואנטים).

    לגבי חלוקה ב0: כשמתקרבים למהירות האור, מסתו של גוף עולה ושואפת לאינסוף על פי טרנספורמציית לורנץ. כשמגיעים למהירות האור וv שווה לc, מקבלים חלוקה ב0, אך במקרה זה המנה שווה לאינסוף ולא למינוס אינסוף ולכן מוגדרת.

    וזאת בניגוד לפונקציה אחד חלקי x ששואפת גם למינוס אינסוף. אפשר להבחין בין מתמטיקה לפיזיקה (כפי שעושה ניסימיינו בפרדוקס זנון), אך אם עניינינו הוא פיזיקה, אז פיזית פרדוקס זנון פתור (אינפי), ומסת גוף במהירות האור מוגדרת (אינסוף).

    ומכיוון שחזרנו לפיזיקה (אינסופסוף!), אז כפי שציינתי היחסות היא תיאוריה יפה ומושלמת כשלעצמה, עם הבעיה השולית שהיא אינה עולה בקנה אחד עם הידוע לנו מתאוריות הקוואנטים, המפץ, המיתרים, מקסוול..

    ולא שהיא לא הגיונית ( ניסים התכוון אני מאמין לאינטואיטיבית), אבל בכל זאת, רכבת באורך קילומטר שנדחסת לסנטימטר ללא ידיעת הנוסעים?

  206. ישראל
    סכום הטור שאמרתי יכול להיות גם 1.5, אם אתה מגדיר סכימה בצורה המתאימה. סכימה זו ניקראת סכימת סזרו (Cesàro), הסכימה הזו מוגדרת היטב ואף שימושית בפיסיקה.

  207. רפאל
    אינסוף מוגדר היטב. וכמו שכתבתי, יש מספר סוגים של אינסוף, ויש גם הגדרות שונות לאותו “סוג” של אינסוף.
    בוא נסתכל רגע על המספרים הטבעיים. דרך אחת להסתכל עליהם ניקראת מספרים סודרים. הכוונה היא שהמספרים הטבעיים מהווים סידרה, ויש ראשון, שני, שלישי וכן הלאה. הסידרה היא של קבוצות – 0 היא הקבוצה הריקה, 1 היא הקבוצה המכילה את הקבוצה הריקה (יש בה איבר 1). 2 היא הקבוצה המכילה את 0 ו-1 (כלומר היא מכילה שני איברים – הקבוצה הריקה וקבוצה שמכילה את הקבוצה הריקה). בשיטה הזו יש הגדרה לאינסוף – והוא נקרא אומגה. אומגה היא מספר לכל דבר. ואפשר לחבר מספרים לאומגה, להכפיל אותו ואף לעלות בחזקה. כל פעולה כזו מוגדרת היטב.

    הדרך השניה היא מספרי מנייה (נקרא גם עוצמה). המספר 3, למשל, מוגדר ע”י כל הקבוצות שיש בהם 3 איברים (או – שעוצמתם 3). ההגדרה הזו נשמעת מעגלית אבל לא כך המצב: לקבוצה ‘א’ ולקבוצה ‘ב’ יש אותו מספר איברים אם ורק אם אפשר למצוא פונקציה חד-חד-ערכית ועל מכל קבוצה לקבוצה השנייה.
    כדי להמחיש הנה דוגמה חמודה – ‘X’ היא קבוצת המספרים הטבעיים, ו-‘Y’ היא קבוצת הזוגיים. העוצמה של ‘X’ היא א-0, אבל מה העוצמה של הזוגיים? לכל x מתוך X ניתן להגדיר את ה-y המתאים – ערכו 2x. לכל y מתוך Y ניתן להגדיר את ה-x המתאים כ-y חלקי 2 (זכור ש-y בהכרח זוגי). אז הוכחנו שעוצמת המספרים הזוגיים הינו גם א-0.

    באותו דרך אפשר להראות שעוצמת המספרים הרציונליים (שברים שבהם המונה והמכנה טבעיים) היא א-0, וגם עוצמת המספרים האלגבראיים (משוואות כמו 7x^4 + 3x – 19 = 0) – גם א-0.

    ואם תרצה יש גם הגדרה למספרי ב, ואנחנו לא יודעים אם מספרי ה-ב שונים ממספרי ה-א.

  208. עשיתי בלאגן בפסקה לפני האחרונה. כתבתי שאני בחרתי לשים במכנה את x בריבוע וישראל את x ברביעית, אבל אחר כך כשבדקתי מה קורה כשמכפילים ב-x בריבוע, הפכתי את היוצרות והתייחסתי לבעיה כאילו אני בחרתי את x ברביעית וישראל את x בריבוע. מקווה שזה לא מבלבל מדי.

  209. אנסה לשפוך קצת אור על נושא החלוקה באפס אם זה מעניין אנשים, למרות שאני מקווה שברור שסטינו לגמרי מהנושא של הסינגולריות של המפץ הגדול.

    החלוקה באפס לא מוגדרת אריתמטית מהסיבה שניסים הזכיר. מבחינה מתמטית, חלוקה מוגדרת פשוט כהכפלה בהופכי – למשל, חלוקה בשתיים זה להכפיל במספר שהופכי לשתיים (המספר שאם כופלים אותו ב-2 מקבלים 1, וזה כמובן חצי). לאפס אין הופכי (אין אף מספר שאם כופלים אותו ב-0 מקבלים 1) ולכן אריתמטית הפעולה לא מוגדרת.

    ניתן להסתכל על גישה פונקציונלית. במקום לחלק באפס, נחלק בפונ’ כלשהי (למשל של המשתנה x) ונקח את הגבול שהפונ’ הולכת לאפס. דוגמא פשוטה היא לקחת את הפונ’ הלינארית x עצמו. ישראל אומר (ובצדק) שהגבול אינו מוגדר היטב כי הוא תלוי באם שואפים אליו מימין או משמאל. אם נקח את הגבול מימין נראה שככל שאנו מתקרבים ל-0 המנה גדלה ללא חסם עליון, כלומר לא קיים גבול סופי והביטוי מתבדר – וזו הסיבה לאינטואיציה ש-1 חלקי 0 נותן אינסוף. אם נשאף לגבול משמאל, נקבל מינוס אינסוף. אבל זה לא נכון שזו הסיבה שהמנה לא מוגדרת היטב. הרי אם אנחנו רוצים, אפשר פשוט להסתכל על פונ’ אחרת – למשל על איקס בריבוע, ולה לא משנה אם אנו לוקחים את הגבול משמאל או מימין. הבעיה הייתה אי הרציפות של הסימן, היה אפשר לפתור אותה בקלות.

    אז מה היא כן הבעיה בגישה הפונקציונלית? הבעיה היא כמובן הרגישות לבחירת הפונ’. ישראל רוצה לחקור את 1 חלקי 0, אז הוא בוחר לשים במכנה את הביטוי x ולבדוק מה קורה כשהוא שואף ל-0. אני בחרתי לשים x בריבוע. אף אחד מאיתנו לא צודק יותר מהשני, ושנינו נשאף לביטוי 1 חלקי 0, אבל נקבל תוצאות שונות. למשל, ישראל יתקל בבעיית הסימן ואני לא. זה רק קצה הקרחון.

    נניח שאני בוחר לשים במכנה x בריבוע וישראל בוחר x ברביעית. אף אחד מאיתנו לא נתקל בבעיית סימן, ושנינו מקבלים התבדרות כאשר x שואף ל-0. על פניו נראה שאנו מסכימים שהמנה היא אינסוף. אבל התבדרות אינה דבר בינארי (יש התבדרות או אין התבדרות), אלא רצף של התנהגויות (איזה סוג של התבדרות, כמה מהירה ההתבדרות). למשל, אפשר לשאול מה קורה אם נכפיל את האינסוף הזה שקיבלנו ב-0. ישראל יכול לומר, “אוקי, במונה במקום לשים 1 אני אשים את הפונ’ x בריבוע (כדי להמנע מבעיות סימן) ואקח את הגבול. התשובה יוצאת 1”. לעומת זאת, אם אני אעשה אותו דבר ואכפיל ב-x בריבוע, אקבל שהתשובה היא עדיין אינסוף, כי ההתבדרות שאני קיבלתי חזקה יותר מזו של ישראל.

    אריתמטית, הפעולה פשוט לא מוגדרת. פונקציונלית הפעולות מוגדרות היטב, אך דורשות הרבה יותר מידע ויכולות לתת תוצאות שונות לגמרי בהתאם לכמה מהר הפונ’ במכנה שואפת ל-0. למעשה, יש תחום שלם ומפותח במתמטיקה שעוסק כמעט רק בשאלה הזו במסגרת אנאליזה מרוכבת.

  210. רפאל

    את ההגדרה של אינסוף תוכל למצוא בויקי.

    מה הבעיה עם אינסוף? מה לא ברור באינסוף?

    וגם מויקי, חלוקה ב0:

    ״חלוקה באפס היא הפעולה המתמטית של חלוקת מספר במספר 0, ותוצאתה לרוב אינה מוגדרת״.

  211. ישראל

    אינסוף אינו מספר. אם היה אפשר להגדיר אותו אז היו מגדירים מה הוא כן ולא מה הוא לא.

    לעניננו אתה אומר שסינגולריות אינה מוגדרת?

  212. 1 או 2 (סדרת השטן, עלק..).

    רפאל

    למה אינסוף לא מוגדר? ואיפה ההתחכמות? אם אתה מחלק ב0 אתה מקבל גם מספר גדול מאוד (אינסוף) וגם את המינוס של אותו מספר, ולא רק את הראשון. בגלל זה החלוקה ב0 אינה מוגדרת.

  213. אלבנצו
    הבנתי מה שאתה אומר. אם אני לוקח, לדוגמה, שדה של טמפרטורות ועושה עליהם טרנספורמציה לינארית (נניח כפל ב-x), אז אין מניעה ש-x לא יהיה 0. לדוגמה x הוא סינוס של הזמן.

  214. ישראל
    אני מתכוון הגיוני במובן – common sense. לאדם הממוצע לא הגיוני שקצב הזמן משתנה, או שאור מתקדם. גם לא “הגיוני” שבו-זמניות אינו דבר מוגדר.

    תרגם “לא הגיוני” ל-it doesn’t make sense.

    רפאל
    אמרת ש 1/0 שווה לאינסוף. זה עניין של הגדרה, אבל זו לא ההגדרה המקובלת במתמטיקה. ההגדרה של חילוק הוא הפכטלה ההופכית לכפל. כלומר a/b = c אם ורק אם a = b*c. וזה לא מתקיים בהגדרה שנתת. אפשר להחליט ש-0 כפול אינסוף שווה ל-1, אבל אז מייד מגיעים בעיות אחרות: תכפיל את שני צידי המשוואה ב-7, אז תקבל 1=7.

    אמרת שאינסוף אינו מוגדר. זה בפירוש לא נכון. כמה שזה נשמע מוזר, מוגדרים מספר סוגים של אינסוף. א-0 (כן, בעברית) הוא העוצמה של המספרים הטבעיים (מספר האיברים בקבוצת המספרים הטבעיים). א-1 (שנקרא בד”כ c, או עוצמת הרצף), הוא מספר המספרים הממשיים בין 0.0 ל-1.0 – וזה יותר גדול מ א-0 (כמובן שצריך להגדיר מה זה “יותר גדול”, ויש לכך הגדרה).

    צריך להזהר במילה אינסוף, ולא להתבלבל אם הרעיון של מאד גדול או מאד קטן. יש לי “חידה”:

    כמה זה 1+1-1+1-1+1-1+1-1+1-1…… עד אינסוף?

  215. ישראל

    זו התחכמות. ידוע שאינסוף אינו מוגדר אבל הוא אינו אפס. ואילו בסינגולריות הוסבר שהסקאלה היא ממש אפס.

  216. רפאל

    ״עד עכשיו ידעתי ש 1 חלקי אפס זה אינסוף״.

    למה לא מינוס אינסוף? זה לא תלוי מאיזה צד אתה שואף לחלוקה באפס?

    ראה לדוגמה…….000000000000000000000000000000-/1

    ככל שהמספר השלילי קטן, המנה שואפת למינוס אינסוף, בעוד שאם המספר חיובי היא שואפת לאינסוף.

    אז כמה היא כשהחלוקה בדיוק ב0? אינסוף או מינוס אינסוף?

    הווה אומר: לא מוגדרת.

  217. אלבנצו

    1. כתבת “יקום בגודל 0 הוא לא הכרחי למודל וסביר להניח (למעשה, די בטוח) שהוא לא יתקבל כפתרון של תורה קוונטית מסיבות של לוקאליזציה. ”

    מה זה בעצם אומר? שתמיד היה שם משהו רק שהוא התחיל להתרחב בזמן מסויים?

    2. לגבי זה “דוגמה טריוויאלית לפונ’ שהיא סינגולרית באמת היא אחד חלקי z. יש לה סינגולריות בנקודה z=0.”

    עד עכשיו ידעתי ש 1 חלקי אפס זה אינסוף. אז אולי בכל זאת יש בפיסיקה או מתמטיקה קשר בין אפס מוחלט לאינסוף?

  218. למעשה, פונ’ אשר ניתן לעשות להן מה שכתבת בתגובה שלך (לשייך להן ערך בנקודה הסינגולרית על ידי לקיחת גבול מימין ומשמאל) לא נחשבות בד”כ לפונ’ סינגולריות באמת, אלא רק פונ’ שלא הוגדרו כהלכה. דוגמה טריוויאלית לפונ’ שהיא סינגולרית באמת היא אחד חלקי z. יש לה סינגולריות בנקודה z=0.

  219. כן. זה לא שיש הבדל בין פיזיקה למתמטיקה, זה שגם במתמטיקה מה שכתבת לא מדויק. קודם כל, אתה מדבר על נקודת אי-רציפות סליקה, שזה מקרה מאוד פרטי ונקודתי של נקודה סינגולרית של פונ’, וממש לא המקרה הכללי. שנית, אתה מדבר בפרט על סינגולריות של פונ’. סינגולריות יכולה להופיע בפונ’ אבל גם באובייקטים מתמטיים אחרים, בפרט באופרטרטורים, או העתקות.

    במקרה של המפץ הגדול, הסינגולריות היא בהעתקה הקונפורמית. דרך נחמדה לחשוב על זה היא שלקחת מרחב שטוח (כלומר, שהסקאלה שלו היא הקבוע 1 ואינה תלויה בזמן), ושהפעלת עליו העתקה מסוימת שמתחה אותו והחזירה לך מרחב אחר. ספציפית עבור a=0 מתקבל ניוון כי המרחב הופך ממרחב d מימדי למרחב 0 מימדי, וההעתקה למעשה מעתיקה את כל הנקודות במרחב לאותה נקודה.

    באופן עקרוני סינגולריות כבידתית יכולה להתפרש גם כנקודה סינגולרית בפונ’ (למשל, אם כותבים את העקמומיות הסקלרית כפונ’ של פרמטר מסוים, הסינגולריות הכבידתית יכולה להיות התבדרות בפונ’ זו). אבל כמו שאמרתי מקודם, גם במקרה זה זו לא תהיה אי-רציפות סליקה (כלומר, שניתן להגדיר אותה מחדש כגבול של הפונ’ בסביבה מסביב לנקודה). חוץ מזה, לעיתים בפונ’ האלה יש נקודות סינגולריות אשר אינן מיוחסות לסינגולריות כבידתית (למשל, כאלה שנובעות פשוט מבחינה גרועה של מערכת קואורדינטות).

  220. אלבנצו
    בד”כ במתמטיקה המשמעות של ערך פונקציה בנקודה סינגולרית מוגדרת כגבול של פונקציה רציפה. למשל sin(x)/x הוא 1 כאשר x=0, במובן שנוכל לקחת x קרוב ל-0 ככל שנרצה. האם המשמעות בפיסיקה, בפרט בתורת היחסות הכללית, היא שונה?

  221. יקום בגודל 0 הוא לא הכרחי למודל וסביר להניח (למעשה, די בטוח) שהוא לא יתקבל כפתרון של תורה קוונטית מסיבות של לוקאליזציה. עצם שינוי הסקאלה במרחב הוא הסבר פשוט ומדויק למיטב ידיעתנו לתצפיות, ולכן נכלל גם במודלים קוונטים. אך האקסטרפולציה עד לנקודה בה a=0 נובעת מפתרון של משוואות קלאסיות שלא יתקיימו בתורה קוונטית.

  222. ממתינה.
    אני מניח שכמות כזו של קללות, נאצות וקישורים לאתרי איקס איקס איקס וביטקוין שהעמסתי בתגובתי הקודמת גורמת לחסימה

  223. אלבנטזו,
    אנסה לחדד. כתבת ש-״לפי מודל המפץ הגדול של תורת היחסות, היקום התחיל מנקודה סינגולרית, כלומר, כל המרחב האינסופי היה בעל סקאלה 0 ״
    מאחר וכנראה זה לא היה המצב, האם ההסבר שנתת שורד? האם תנאי ההתחלה: יקום בגודל אפס הוא פרט קריטי למודל או שלא אך המודל ישרוד את השינוי בתיקונים המתבקשים?
    לאור תשובתך האחרונה אני מבין שכן וכן אני מקווה שכעת אני טיפה יותר מובן

  224. שמוליק,

    אני לא בטוח למה אתה מתכוון ב”הסבר הסקאלות”. עברתי בזריזות על התגובות הקודמות שלי ונראה לי שכל מה שכתבתי משותף גם לקוסמולוגיה קלאסית וגם למה שנראה לנו שמתקיים באופן קוונטי, מלבד כמובן הפיזיקה שקרבת נקודת המפץ. אבל אם אתה רוצה תשובה יותר מדויקת או החלטית, תצטרך להבהיר בדיוק למה אתה מתכוון.

    יוני,

    קודם כל, סקאלר זה משהו אחר לגמרי. למרות הדמיון במילה, לסקאלר אין קשר לסקאלה. סקאלר זה מונח מעולם המתמטיקה שפירושו גודל שנשאר קבוע לאחר טרנספורמציה כזו או אחרת, כאשר השימוש הפופולרי ביותר במונח הוא כלפי טרנספורמציית סיבוב (ולכן וקטור אשר יש לו כיוון ולכן ישתנה תחת טרנספורמציית סיבוב אינו סקלר, אך מספר כלשהו – כמו אנרגיה של חלקיק – לא ישתנה תחת סיבוב ולכן גודל סקלרי).

    א. מאמר שמדבר על מה בדיוק? על האופן שבו יקום אינסופי מתרחב? המאמרים הנוגעים לכך נכתבו והתפרסמו לפני עשרות שנים. היום קל יותר למצוא את הדברים בכל ספר מבוא לתורת היחסות, בפרק על קוסמולוגיה. למשל, הספר של שון קארול הוא ספר ברור למדי שמכסה את הנושא בצורה פשוטה מאוד.

    Spacetime and Geometry / Sean Carrol

    ב. הצורה המפורשת של פונ’ הסקאלה (שים לב, זו פונ’ של הזמן: זו כל הפואנטה, זה לא גודל קבוע אלא משהו שמשתנה בזמן) נקבעת לפי פתרון משוואות איינשטיין, או למעשה משוואות פרידמן, ולכן תלויה במה יש במרחב. כל הרעיון של תורת היחסות הכללית היא שכבידה היא האינטראקציה בין המרחב עצמו לבין מה שהוא מכיל. לכן הפתרון של פונ’ הסקאלה תלוי במודל המדויק ובתכולת היקום שלו. מה שנהוג להסתכל עליו זה יקום שהוא פחות או יותר הומוגני ובעל צפיפות נמוכה (קירוב מצוין ליקום שלנו), ואז ניתן לפתור ולמצוא את פקטור הסקאלה. שים לב שהוא פונ’ לא טריוויאלית של הזמן, ומתנהג שונה בין אם הגורם הדומיננטי ביקום הוא חומר או קרינה, למשל. סלח לי שאני לא כותב פה פונ’ או משוואות, אבל כמו לסעיף הקודם – את הצורה המפורשת אפשר למצוא בכל ספר מבוא ליחסות כללית וקוסמולוגיה, ובפרט בספר שהזכרתי קודם.

    ג. לפי ארבעה מיליארד שנה הכל היה קטן יותר. בנוגע לשמש בפרט, אני לא יודע לענות בשלוף כי אני לא מומחה להתפתחות כוכבים (זה אסטרופיזיקה ואני חוקר את תורת המיתרים). אין לי מושג איך נראתה השמש לפני ארבעה מיליארד שנה, אולי היא הייתה גדולה יותר בגלל אופי תהליכי ההתפתחות של כוכבים. אם אתה רוצה לדעת מה פקטור הסקאלה בזמן מסוים, אתה מוזמן להציב בעצמך מספרים בנוסחאות שמצויות בספרים שהזכרתי.

    ד. אין לי מושג למה אתה מתכוון או על מה אתה מדבר.

  225. הבנתי את מה שכתבת לגבי הסקאלר.
    א. אשמח אם תפנה אותי לאיזה מאמר שמדבר על זה.
    ב. אשמח אם תתן לי את ערך הסקאלר ואת קצב השינוי שלו, שהרי לפי זה מודדים את הזמן מהמפץ הגדול.
    ג. האם לפי דבריך לפני כארבע מליארד שנה השמש היתה קטנה יותר מהיום כיון שהסקאלר היה קטן יותר, ואם כן בכמה? (אם אפשר אנא תן הפניה למקור מדעי).
    ד. לפי הידע המעודכן קצב ההתפשטות גדל, ולמרות זאת הכוכבים הכי רחוקים שניצפים על ידינו אינם מתרחקים במהירות של חצי ממהירות האור, מה שמלמד פעם היה קצב השינוי של הסקאלה נמוך יותר, ואם כן איך למרות שעברו כ13 מליארד שנה הצליחו הכוכבים להתרחק מאיתנו מרחק גדול מזה?

  226. אלבנצו
    אנחנו מתעלמים מהתארכות הזמן בתרגיל של כדורי ביליארד, כי אין להתארכות הזמן כל משמעות בהקשר הזה. לא כך המצב בנקודה הסינגולרית.

    אני חושש שאנחנו מדברים על דברים שונים, וכנראה שממש לא הסברתי את עצמי טוב. אין לי שום סיבה להתווכח איתך על הפיסיקה עצמה 🙂

    אני לא מתווכח על מה אומרת תורת היחסות. אני רק אומר שזה מוזר לי לדבר על מה אומרת תורת היחסות במקום שאנחנו יודעים שהיא לא תקפה.

    אני מבין (עכשיו) שאתה אומר שהטענות של רפאל שגויות, גם בהקשר של תורת היחסות. תודה על ההסבר!

  227. 1. מה זאת אומרת “איך אפשר להתעלם”? כאשר אתה פותר בעיה של כדור ביליארד במכניקה קלאסית, אתה מתחשב בהתארכות הזמן? מתחשב בכוח פאולי בין הכדורים? בפיזיקה אנו בונים מודל ואז בוחנים אילו פתרונות הוא מספק לבעיות. במודל של יחסות כללית אין אי-וודאות ואין אף תופעה קוונטית, ולכן כאשר פותרים בעיה עם מודל זה מקבלים תוצאות שסותרות עקרונות אלו.

    2. לא, רפאל טעה כשאמר זאת וגם אתה טועה עכשיו. אפשר לכתוב מודלים של נקודה (0-מימדית) שמתרחבת למרחב d-מימדי. אין עם זה בעיה גיאומטרית כמו שרפאל חשב (וכמו שנראה לי שאתה חושב). האם יש בכך בעיות שנובעות מלוקאליזציה קוונטית? כן, עקרונית. לכן בתורה שאינה כוללת עקרונות קוונטים דבר כזה יתאפשר, ובתורת כבידה קוונטית הוא כנראה שלא, לפחות לא באותו האופן (למרות שאי אפשר להגיד בבטחון שהוא לא יתאפשר בכלל פשוט מפני שאנחנו עדיין לא יודעים בוודאות).

    3. הדיון שלנו בהחלט נגע גם למה שתורת היחסות אומרת, וגם לשאלה האם היא מתאימה או לא למציאות. מודל המפץ הגדול הוא מודל במסגרת תורת היחסות, לא תורת המיתרים או לופ קוונטום גרביטי. אתה יכול לחזור אחורה ולראות שלאורך כל התגובות שלי אני מבחין בין מה אומר מודל המפץ לבין מה באמת נכון, ומשתדל להבהיר מתי תורת היחסות אמינה ומתי לא. כשאתה כותב שמודל המפץ הגדול לא מתחיל מנקודה אתה פשוט טועה, כי הוא כן. אולי הוא לא מודל טוב כי הוא לא קוונטי, אבל הוא מתחיל מנקודה.

    4. מספר הפעמים שכתבתי שתורת היחסות לא נותנת הסבר מלא לבעיה הקוסמולוגית גדול מכדי שיהיה לי כוח לספור אותו ולצטט אותו כאן.

    5. שוב, קרא את תגובותי ותראה כמה פעמים אני כותב שהסינגולריות היא פרדיקציה של יחסות ושתורות מודרניות סותרות אותה ולכן אין להתייחס לפרדיקציה הזו כאמת. לא ברור לי איך זה לא נהיר מדברי. ציטוט לדוגמא,

    “כפי שהערתי, זה שלפי יחסות כללית זה נכון לא אומר שזה נכון באופן עקרוני כי תקופת הזמן הזו (בה הסקאלה הייתה מאוד מאוד קטנה) היא בדיוק נקודת החולשה של תורת היחסות בה היא לא אמינה. לכן חוקרים מודרנים מסתכלים על תורות אחרות ומגיעים למסקנות אחרות.”.

    6. לרפאל היו תהיות בנוגע למודל התיאורטי, ניסיתי להבהיר אותן. אנחנו יודעים שמכניקה קלאסית לא נכונה, אבל אם מישהו יבוא אלי ויטען שמכניקה קלאסית לא הגיונית או שמשוואות ניוטון מייצרות מצב שבו לא יכולה להתקיים תנועה יחסית, אעמיד אותו על טעותו ואשתדל להסביר לו את הנושא. זה לא יאמר שאני טוען שהיקום מתואר על ידי מכניקה קלאסית. כשם שנחמד להבין מכניקה קלאסית (למרות שהיא שגויה) ושיש טעם להבין אותה כי אפשר לשאוב ממנה ידע ואינטואיציה לתורות מתקדמות ומדויקות יותר, כך יש הרבה יופי וגם המון טעם בלהבין מודלים קלאסים של קוסמולוגיה. לכן כשרפאל שאל שאלות, עניתי עליהן תוך הבהרה חד-משמעית שאני מסביר לו כיצד עובד מודל למרות שהמודל אינו מדויק ואינו באמת מה שפיזיקאים חושבים שקרה ביקום המוקדם.

    כתבתי זאת מספר פעמים במהלך הדיון, אתה יכול להסתכל בעצמך.

  228. אבי,
    מה כבר כתבתי שהתגובה מחכה לאישור?
    בנוסף, אני מצטרף לבקשה של אחרים שתחזיר את עמוד התגובות האחרונות. חלק גדול מהכיף באתר זה היא היכולת להגיע בקלות לתגובות האחרונות.

  229. האם הסבר הסקאלות יהיה נכון גם לא היתה נקודה סינגולרית בגלל אפקטים קוונטיים?
    במילים אחרות, האם ההסבר מה״ל יהיה נכון גם תחת תורת כבידה קוונטית?

  230. אלבנצו
    אני מבין מה אתה אומר.
    אבל, אני לא מבין איך אפשר להתעלם מתורת הקוונטים בנקודה הסינגולרית. רפאל מעלה טיעון שהוא היגיוני – שום התנפחות מגודל 0 לא תגדיל את הנקודה, ובטח לא למשהו אינסופי. הדיון שלנו הוא לא על מה אומר תורת היחסות בנקודה הזו, אלא על מה באמת קורה שם (או אז..).

    כל מה שאני מנסה להגיד זה שאני לא חושב שצריך לטעון שתורת היחסות נותנת הסבר מלא למה שקרה, ולהבנתי, כך משתמע ממה שאתה כותב. אם הסינגולריות היא תוצאה של תורת היחסות, במקום/זמן שאתה בעצמך אומר שהיא אינה תקיפה, אז למה אתה מסביר איך היקום התפשט מנקודה זו? לי נראה שאין בכך צורך.

  231. ניסים,

    מה שאני מסביר שוב ושוב זה שכל תורת היחסות סותרת את תורת הקוונטים. היא תורה קלאסית, לא קוונטית. לא צריך לחפש נקודות סינגולריות במפץ הגדול, תסתכל על תנועה של חלקיק חופשי במרחב ריק בתורת היחסות ותראה שעקרון אי-הוודאות נשבר.

    מה שכתבתי במספר התגובות האחרונות שלי היא שתורת היחסות כן מכילה נקודה סינגולרית שגודלה 0, אבל שהיא תורה קלאסית. לכן אנו יודעים שהיא לא מדויקת. עד לנקודה מסוימת היא מהווה קירוב טוב למציאות, אבל החל מנקודה מסוימת היא כבר לא ואז צריך כבידה קוונטית. בכל מקרה, לא ניתן להשתמש בטיעונים של אי-וודאות במסגרת יחסות כללית כי היא לא מכבדת אותם, בדיוק כפי שלא ניתן לקחת פתרון של בעיה בפיזיקה ניוטונית ולהגיד שהוא טוב או לא טוב כתלות בעקרון אי-הוודאות. כל התורה עומדת בסתירה למכניקת הקוונטים אז הטיעונים האלה לא רלוונטים.

  232. רפאל
    הפתעת אותי. חשבתי שאתה התחלת להאמין כי זה נראה לך הגיוני. שמעתי ממך הרבה טיעונים “הגיוניים” לקיומו של בורא, כמו למשל – קיומם של חוקי טבע.

    כנראה שטעיתי.

  233. רפאל
    מה שאנחנו קוראים “היגיון”, לא מטפל בבעיות של המאד קטן או מאד גדול. ההיגיון אומר שכדור הארץ הוא שטוח והשמיים מעלינו. הגיון זה מה שמנחה ילדים בגן (בקש מילד קטן לצייר – איפה הוא שם את השמיים? אחרי זה משכנעים אותנו שזה לא עובד מעבר לחיי היום-יום שלנו.

    לוגיקה זה משהו אחר לגמרי, ולקח הרבה מאד שנים עד שלמדנו מהם חוקי הלוגיקה (אריסטו, בול, פרגה, ראסל, וויטגנשטיין וכן הלאה).

    רפאל – אני חושב שגם אתה מסכים שהאמונה הדתית באה מההיגיון, לא?

  234. אלבנצו
    זה לא סותר את תורת הקוונטים האמירה שכל החומר סטטי, בנקודה אחת? אני לא יודע אם זה סותר את עקרון אי הוודאות, או את העקרון של פאולי, אבל זה בטח אמור להשפיע, לא?

  235. נסים,

    אני שומר את הפסקה אחרונה שלך “אל תנסה להשתמש בהיגיון כדי להבין את…” כדי לעשות בה שימוש בזמן אחר 🙂

  236. ניסים,

    זה לא נכון. קודם כל, לאי-וודאות אין קשר כי תורת היחסות היא לא קוונטית. אז בפירוש אין בה אי-וודאות. שנית, בהחלט יש נקודה סינגולרית בה פקטור הסקאלה מתאפס. בדיוק כשם שיש נקודה סינגולרית בתוך חור שחור קלאסי, אשר קוונטית כנראה תמרח ולא תהיה סינגולרית.

  237. רפאל/אלבנצו
    אני לא חושב שכל המודלים של המפץ הגדול מתחילים בנקודה אבסולוטית. זה סותר את עקרון אי-הוודאות, ואני גם לא חושב שיש הכרח שההתחלה הייתה נקודה. ה”סונגולריות” היא הנקודה בה תורת היחסות לא מתקיימת, בדיוק ש x/x לא מוגדר ב-0. המתמטיקה מאפשרת לנו להגדיר את הערך ב-0 כגבול, אבל זה לא אומר שהערך ב-0 הוא 1!
    זה מזכיר קצת את התורה של פרמינדס, והפרדוקסים של זנון. מבחינה מתמטית – הפרדוקסים פתורים , אבל מבחינה פילוסופית – עדיין אין פתרון 🙂

    רפאל – בקשר לזמן, יש מושג שנקרא “זמן נכון” (proper time) זה הזמן שצופה באותה נקודה ימדוד, וזמן זה מוגדר היטב.

    אל תנסה להשתמש בהיגיון כדי להבין את המפץ הגדול. תורת היחסות נוגדת את ההגיון (אני מתכוון ל common sense, לא ללוגיקה פילוסופית), תורת הקוונטים נוגדת את ההגיון. אז אל תצפה שאם אתה מביא את שניהם לגבול אז תקבל משהו הגיוני 🙂

  238. רפאל,

    לא יודע מה להגיד לך, זה פשוט לא נכון. אנחנו “תופסים את כל המרחב” כלשונך אם ורק אם מקדם הסקאלה הוא 0. עבור כל מספר אחר, אנחנו לא. תסתכל על הדוגמא שנתתי, תציב כל מספר חוץ מ-a=0, תראה שאני ואתה מתרחקים זה מזה. זו מתמטיקה די פשוטה, אם אתה רוצה אתה יכול לנסות להסתכל עליה – אם אתה מבין מה זה מרחב מטרי אז כל מה שאתה צריך להסתכל עליו זה מטריקת FRW. כל הדיבורים האלה קצת חסרי טעם מכיוון שאפשר לראות שאתה טועה ע”י חישוב של שניה אחת (אם אתה יודע מה זו מטריקה).

  239. אלבנצו

    אם אני ואתה לא גדלים יחסית לסקלה, אז נוצר ביננו מרחק שמאפשר תנועה במרחב אבל אם אני ואתה גדלים יחד עם הסקלה אז כמו שבהתחלה תפסנו את כל המרחב (שהיה אז אפס) כך תמיד נתפוס את כל המרחב ולא משנה כמה תגדל הסקלה, ובמקרה כזה לא יהיה מקום לנוע.

  240. המרחק היה 0 רק ברגע אחד בזמן. תנועה תמיד מוגדרת על פני קטע בזמן. בוא נשאיר את הסקאלה בתור נעלם, נקרא לה a.

    נניח שהמרחק ביני לבינך הוא סרגל אחד, כלומר מרחק של 1*a (כמו בדוגמא הקודמת – אם הסקאלה היא 1 אז זה מרחק של מטר, אם היא חצי אז חצי מטר וכו’). אבל זו תמונה ברגע בודד. תחשוב על התמונה בה ברגע ההתחלתי המרחק בינינו a ושניה אחרי הוא 2a, שתי שניות אחרי הוא 3a וכו’. אתה אומר שאם נציב a=0 אז נקבל שאין תנועה וזה נכון טכנית, אבל כמובן חסר כל משמעות כי כל הפואנטה היא שהסקאלה משתנה עם הזמן. ברגע המפץ היא שווה ל-0 אבל שבריר שניה אחרי היא כבר לא 0. כך שאין שום בעיה עם תנועה יחסית כל עוד לא מסתכלים רק על רגע המפץ עצמו (וכפי שאמרתי, בלי שום קשר ליחסות כאשר מדברים על תנועה לא ניתן להסתכל רק על רגע בודד).

  241. אלבנצו,

    1. תודה.

    2. אם המרחק ביני לבינך היה בהתחלה אפס והכל מתרחב באותו יחס אז המרחק ביננו תמיד ישאר אפס כי גם אני ואתה גדלים באותו יחס ואז לא תתכן שום תנועה…

    3. תודה על שהבהרת ש 13.7 מיליארד שנים קשור למערכת היחוס של המודד התיאורטי (שברגע המפץ התיישב במקום בחלל ריק ומאז לא זז או עבר בשדה כבידה משמעותי). הבעיה היא שלא כולם מודעים לכך וזה יוצר הרבה חיכוכים וקושיות שלמעשה אין להם מקום.

  242. 1. לפי מודל המפץ הגדול של תורת היחסות, היקום התחיל מנקודה סינגולרית, כלומר, כל המרחב האינסופי היה בעל סקאלה 0 ולכן טופולוגית הוא שקול לנקודה אחת. אתה שואל “איך” אבל זה בדיוק מה שהסברתי לך. אם ההסבר לא ממצה מספיק בשבילך אתה מוזמן לקרוא ספרות מקצועית יותר, אבל זה ידרוש פיתוח יכולות מתמטיות לא טריוויאליות. כפי שהערתי, זה שלפי יחסות כללית זה נכון לא אומר שזה נכון באופן עקרוני כי תקופת הזמן הזו (בה הסקאלה הייתה מאוד מאוד קטנה) היא בדיוק נקודת החולשה של תורת היחסות בה היא לא אמינה. לכן חוקרים מודרנים מסתכלים על תורות אחרות ומגיעים למסקנות אחרות.

    2. זה שהכל מתרחב ביחד לא בא על חשבון תנועה יחסית. אם שתי נמלים נמצאות על בלון, אחת עומדת במקום והשניה מתרחקת ממנה, והבלון מתנפח אז שני האפקטים קורים במקביל – הנמלים גם מרחקות כתוצאה מהניפוח וגם קיימת ביניהן תנועה יחסית. אני לא מבין מה הבעיה. וזה ששינוי בסקאלה הוא לא שינוי כי הכל משתנה זה לא נכון, כי יש זמן. בכל נקודה בזמן אם תעשה פריז-פריים ותסתכל על היקום אכן לא תוכל לדעת מה הסקאלה. אבל יש שיטות ומדידות שיכולות להגיד מה הסקאלה היום ביחס למה שהיה אתמול.

    3. זרימת הזמן אכן יחסית וניתן לשחק איתה. המספרים שנותנים בד”כ לזמן שעבר מאורע מסוים מתייחסים למערכת יחוס מסוימת (צופה שנע יחד עם היקום). לצורך העניין, אתה יכול לומר שזה הזמן שהיה מודד אדם שברגע המפץ התיישב במקום בחלל ריק ומאז לא זז או עבר בשדה כבידה משמעותי. צופים אחרים יכולים לראות זמן קצר יותר בהתאם למערכת היחוס שלהם.

  243. אלבנצו,

    אכן דיברנו הרבה בעבר סביב בנושא זה. אבל עכשיו אני מבקש לחדד ענין עקרוני. אם היקום התחיל מנקודה סינגולרית שהיא אפס ממש (ולא כמו שהסביר ניסים) אז עדיין צריך הסבר פשוט איך היקום יכול להיות אפס ואינסוף בעת ובעונה אחת. ואם כן – מה ההבדל בין אפס לאינסוף?

    דבר נוסף שאני רוצה לשאול עקב המשל שלך על המרחק ביני ובינך שהתכווץ בחצי. אתה אומר שבמקרה כזה הכל התכווץ בחצי. כלומר שבעצם לא היה שום שינוי כי גם אני ואתה התכווצנו. אבל ביקום לא רואים שהכל מתרחב ביחד כי אחרת לא היינו יכולים להבחין בשום תנועה, לדוגמה – לא היינו יכולים להבחין בהתרחקות הגלקסיות זו מזו כיוון שגם הגלקסיות עצמן גדלות באותו יחס.

    דבר אחרון – אנא התיחסותך לענין ה- 13.7 מיליארד שנים. האם גם יצורים החיים על כוכב לכת אחר בעל כח משיכה גדול בהרבה מכוכב הלכת שלנו היו מסכימים שעברו 13.7 מיליארד שנים מאז שהיקום החל להתרחב?

  244. רפאל,

    אני משוכנע שכבר ניהלנו את השיחה הזאת פעם ושכבר הסברתי לך את כל זה.

    תחילה צריך להבין מה זה סקאלה. אם היום אני ואתה עומדים זה ליד זה ובינינו מפריד סרגל שכתוב עליו “1 מטר”, אז המרחק בינינו הוא מטר. לאורך מגרש כדורגל אפשר להכניס 100 סרגלים כאלה, אז אורכו 100 מטר. בין חיפה לתל-אביב נכנסים בערך 80000 סרגלים כאלה, אז המרחק ביניהן הוא 80 ק”מ.

    אתמול הייתה סקאלה אחרת. כמו במחשב שניתן לפעמים למתוח תמונה או לכווץ אותה מבלי לשנות פרופורציות (לתפוס את הפינה ולגרור), ככה גם היקום. אז אם אתמול הסקאלה הייתה 1/2 ביחס להיום, אז אני ואתה – למרות שעדיין מפריד בינינו אותו סרגל – היינו במרחק חצי מטר זה מזה. גם הסרגל התכווץ, הכל התכווץ כי המרחב עצמו התכווץ. במגרש הכדורגל נכנסים 100 סרגלים אך אורכו 50 מטר, ובין חיפה לתל-אביב נכנסים 80000 סרגלים אך המרחק הוא 40 ק”מ. שלשום המצב היה אף גרוע יותר, והסקלאה הייתה 0.1. כלומר, המרחק בין חיפה לתל-אביב היה 8 ק”מ למרות שעדיין נכנסו ביניהם 80000 סרגלים, וכו’. שים לב שבכל אחד מהימים, למרות שהסקאלה שונה, היקום עדיין אינסופי. עדיין ניתן להתקדם בכל כיוון שתבחר בכל מרחק שתבחר. אם משה נחשד בשוחד ורוצה לברוח 3000 ק”מ ממדינת ישראל, אז היום הוא יכול לברוח לשוויץ. אתמול היה צריך לברוח עד החוף המזרחי של אמריקה. שלשום היה צריך לברוח לחלל החיצון כדי להתרחק 3000 ק”מ מרשויות אכיפת החוק, אבל בכל מקרה זה אפשרי. ליקום אין סוף באף אחד מהמקרים.

    ניתן לשאול מה קורה אם חוזרים אחורה בזמן עד לנקודה שבה הסקאלה היא ממש 0. אז המרחק ביני לבינך הוא 0, למרות שמפריד בינינו סרגל. אורכו של המגרש הוא 0 והמרחק בין חיפה לתל-אביב הוא 0. מכיוון שהמרחק בין כל שתי נקודות במרחב הוא 0, המרחב הוא למעשה נקודה (הוא מרחב 0 מימדי ששקול לנקודה). זו הכוונה בנקודה הסינגולרית – הזמן שבו הסקאלה הייתה בדיוק 0 ולכן כל שני מקומות ביקום הופרדו זה מזה במרחק 0 והיו למעשה אותו מקום. המונח נקודה סינגורלית נגזר מהביטוי המתמטי שמשמעותו שהטרנספורמציה הקונפורמית (שמשנה את הסקאלה) היא סינגורלית – אינה מוגדרת ביחידות. היא ממפה את כל הנקודות ביקום לאותו המקום.

    בניגוד למה שכתבת, לפי תורת היחסות היקום כן החל מנקודה זו. ניתן לחזור אחורה אחרי התנפחות היקום שלנו עד לזמן שבו הסקאלה הייתה ממש אפס. אבל, כמו שכתבתי לך בעבר, לפני שמגיעים לזמן זה, מגיעים לזמן שבו אפקטים קוונטים נהיים שקולים בחשיבותם לאפקטים הגרביטציונים בהתפתחות היקום, ולכן לא ניתן להזניח את מכניקת הקוונטים וחייבים לתאר את התהליך בעזרת תורה שמשלבת גם כבידה וגם קוונטים. יחסות כללית אינה כזו ולכן הפרדיקציה שלה (שהיקום היה ממש נקודה) אינה אמינה בהכרח. מה כן אמין? זה קצת ארוך ודיברנו על כך הרבה בעבר (אני זוכר שהבאת סרטון של חוקרים ממכון פרימטר שדיברו על מה באמת היה במפץ ומה קדם לו, על ידי כך שהסתכלו על תורות כבידה קוונטיות במקום על יחסות כללית בזמן שקרוב למפץ).

  245. נסים

    כלומר אתה אומר היקום כפי שרואים אותו הפיסיקאים היום לא התחיל מנקודה שהרי נקודה אינה תופסת מקום כלל אלא מגוף ראשוני תלת מימדי קטן מאד שהתנפח. אם כן 13.7 מיליארד שנים זה לא הזמן שעבר מאז שהיקום נוצר אלא זה הזמן שבו הוא התחיל מסיבה כלשהי להתנפח.

    כמו כן מזכיר לך שעדין לא מובן איך מחשבים 13.7 מיליארד שנים מאחר והזמן אינו ענין גלובלי אלא ענין מאד אישי ושונה מגוף לגוף בהתאם למהירות בה נע הגוף ובהתאם לכח הגרביטציה המופעל עליו.

  246. רפאל
    תחשוב על זה ככה: אתה נמצא בנקודה הסינגולרית ובזמן 0 מתחיל לנוע. הנקודה “מתנפחת” הרבה יותר מהר ממה שאתה מסוגל לנוע. לכן מבחינתך, אין “סוף” ליקום הזה.

    הנקודה אינה בגודל 0, אלא בגודל מינמלי כלשהו, מאד קטן.

  247. אלבנצו

    תוכל להסביר לאנשים פשוטים חובבי מדע איך יקום אינסופי מתחיל מנקודה סינגולרית שהיא סופית בהחלט?

  248. יוני,

    אני קורא שוב ושוב את התגובות שלך ולא מצליח להבין איפה אתה רואה סתירה. אתה יכול לכתוב באופן מפורש שני סעיפים ולהראות שהם מביאים למסקנות מנוגדות, כלומר שקיימת סתירה ביניהם?

    ודרך אגב, לא חשוב אם תכתוב “ראה ויקיפדיה”, זה לא יהפוך שום דבר שאתה אומר ליותר נכון או פחות נכון. היקום תמיד היה אינסופי וכל העניין הוא רק בשינוי של הסקאלה של המרחב. ליקום אין ולא היה גודל (לפי תאוריות קוסמולוגיות המקובלות כיום).

  249. זו בדיוק הסתירה, מצד אחד לפי תאורית המפץ הגדול הכל התחיל בנקודה סינגולריות ראה ויקיפדיה, לאחר מכן היתה אינפלציה שהגדילה את היקום לגודל של גלקסיה בינונית ראה ויקיפדיה, ומאז הכל הולך לפי הגדלים הפיזיקליים המוכרים לנו כיום. מצד שני טוענים שיש כוכבים שרחוקים מאיתנו אפילו יותר מ13 מליארד שנות אור, או כפי שמגדיר עם זאת “היקום הנצפה”.
    כמו כן אני מסכים שכשהאור יצא מהכוכב הוא היה יותר קרוב אך בגלל שהמרחב עצמו נמתח, לקח לו יותר זמן להגיע. אבל אנחנו יודעים שהמרחב בינינו לבין הקוואזרים הכי רחוקים נמתח בערך במהירות של חמישית ממהירות האור, וזה מה שגורם להסחה לאדום, ולכן אפשר לחשב בהערכה גסה שאם לקח לאור להגיע אלינו 13 מליארד שנה אז הקוואזר הזה היה במרחק של שקטן בכחמישית ממרחקו היום, שזה בערך מרחק של 10 מליארד שנות אור, משתמע שהוא הספיק להתרחק מאז המפץ הגדול תוך 690מליון שנה כ10 מיליארד שנות אור.

  250. יוני,

    מה שאתה כותב לא נכון. היקום (לפי הידוע לנו כיום ולפי התיאוריות הקוסמולוגיות המקובלות) תמיד היה אינסופי. מה שמשתנה זו רק הסקאלה היחסית של מרחק. היקום מעולם לא היה בגודל של גלקסיה, או בכל גודל אחר. אולי אתה מתבלבל עם היקום הנראה, שזה דבר אחר לגמרי? כך או כך, השאלה הרלוונטית היא כמה זמן לוקח לשני עצמים לעשות אינטראקציה (לצורך העניין, להחליף ביניהם אור). לפני האינפלציה, הסקאלה הייתה כל כך קטנה שפחות או יותר כל שני עצמים יכלו לתקשר כמעט מיידית והשמיים היו הומוגניים. אחריה זה כבר לא נכון.

    היקום מתרחב, ולכן אור שנע במשך 13 מיליארד שנה ומגיע אלינו היום, לא התחיל במרחק של 13 מיליארד שנות אור מאיתנו אלא קרוב יותר, והעצם שפלט אותו לא נמצא היום 13 מיליארד שנות אור מאיתנו אלא יותר. זו בדיוק המשמעות של השינוי בסקאלה. אבל אני לא מבין מה מכל זה מפריע לך.

  251. יוני
    אתה מסתכל על המפץ הגדול כעל פיצוץ בעולם תלת-מימדי. בעולם כזה, באמת היה לוקח זמן יחסית קצר לאור להגיע.
    אבל היקום הוא 4 מימדי, ומה שקורה אינו דומה לפיצוץ רגיל שאנחנו מכירים. בפיצוץ כזה יש מרכז, נקודה במרחב בה קרה הפיצוץ.
    במפץ הגדול, המצב שונה. המרחב עצמו מתפשט. התוצאה היא שלאור לוקח הרבה יותר זמן להגיע. יש אפילו טווח שממנו אור לעולם לא יגיע אלינו ואף פעם לא נדע מה קרה שם (או אז….).

    ודרך אגב – להבנתי, אותו חור שחור שמתואר במאמר נמצא הרבה יותר רחוק, משהו כמו 45 מיליארד שנות אור, או שגילו הרבה פחות מ-13 מיליארד שנה…

  252. נכון שלפי תאוריית המפץ הגדול היקום נוצר לפני 13.7 אך מופיע באתר שראו חור שחור מתקופה בה היקום היה בן690 מליון שנה כלומר לפני 13 מליארד.
    לעצם העניין: לפי תאורית האינפלציה, היקום גדל תוך כמה חלקיקי השניה לגודל של גלקסיה בינונית מה שאומר שהאור היה אמור להגיע עלינו תוך זמן קצר (לכל היותר מאה אלף שנה), גם אם הוא המשיך להתרחק בקצב גבוה מאוד אפילו לצורך העניין במהירות של חמישית ממהירות האור (שזו לגמרי מהירות עצומה) במשך 13 מליארד שנה, לאור היה לוקח להגיע אלינו חמישית מהזמן הזה.

  253. יוני
    המפץ הגדול היה לפני 13.7 מיליארד שנה, ולא 13. כמו שהסביר לך אלבנצו, היה שלב של ניפוח בו היקום התרחב הרבה יותר ממהירות האור, אבל הזמן שזה ארך (אם בכלל אפשר להשתמש במושג של זמן) הוא שבריר קטן של שניה.

    הפסקה האחרונה שלך זה בכלל לא מה שאלבנצו כתב. קרא את זה שוב.

  254. אז אם אני מבין נכון, אתה טוען שהוא היה במרחק של 13 מליארד שנות אור מאיתנו תוך פחות מכמה שניות. אני קראתי על המפץ הגדול די הרבה והבנתי שהגיעו למסקנה שהכל היה בנקודה אחת בגלל שאנחנו רואים שהכוכבים והגלקסיות מתרחקים מאיתנו, כך שאם מגלגלים את הזמן אחורה מגיעים לכך שלפני 13 מליארד שנה הכל היה באותה נקודה, אך לפי טענתך לפני 13 מליארד כבר היו כוכבים שרחוקים מאיתנו מרחק של 13 מליארד שנות אור.

  255. חובב,

    גודלו פחות או יותר זהה. כמות הקרינה שמשחרר חור שחור נמצאת ביחס הופכי למסתו. כלומר, חורים גדולים כמעט לא קורנים. אפילו בחור שחור בגודל של מסת שמש הקרינה ממש זעומה. סדרי הגודל של הזמן שיקח לחור שחור כזה גדול לקרון מספיק אנרגיה כדי שנוכל לשים לב לירידה במסתו גדולים בהרבה מזמן החיים של היקום.

    יוני,

    נכון שכשהיקום היה צעיר מספיק, הכל היה נורא קרוב ופחות או יותר כל שני אובייקטים יכלו לעשות אינטראקציה בזמן קצר, אבל תקופה זו היא טרום אינפלציה. האינפלציה הקוסמית ניפחה את היקום בהמון סדרי גודל במהירות גדולה ממהירות האור, ואחריה כבר לא הייתה הומוגניות כזו. החור השחור המדובר אמנם מאוד רחוק ומאוד זקן, אבל לא מדובר בקבועי זמן רלוונטים לאינפלציה. במילים אחרות, כאשר הוא נוצר היקום כבר היה די גדול ובשלב של התרחבות איטית, בדומה לתמונה שאנו רואים ביום כשאנו מסתכלים על היקום.

  256. לכאורה יש סתירה: מצד אחד טוענים שהאור יצא לפני יותר מ13 מליארד שנה עד שהגיע אלינו מה שאומר שבזמן שיצא הוא כבר היה רחוק מאיתנו מאד (אינני יודע לחשב כמה, כי ברור שהתרחקנו תוך כדי שהאור “נוסע” לכיוננו) , מצד שני לפי תאוריות המפץ הגדול, כל היקום היא בתחילתו מאד קטן כך שאמור להיות שהאור יגיע אלינו ממש מהר. אשמח אם מישהו יעשה לי סדר עם החישוב.

  257. מעניין מה גודלו של החור השחור הזה כיום לאור קרינת הוקינג שנפלטה ממנו ב 13 מיליארד השנים האחרונות.

כתיבת תגובה

האימייל לא יוצג באתר. שדות החובה מסומנים *

אתר זה עושה שימוש באקיזמט למניעת הודעות זבל. לחצו כאן כדי ללמוד איך נתוני התגובה שלכם מעובדים.